Содержание

Как распознать планеты в ночном небе?

Когда вы смотрите на ночное небо, некоторые из «звезд», которые вы видите, не являются звездами. Это планеты.

Из восьми планет нашей Солнечной системы пять видны невооруженным глазом, исключая, конечно, Землю: Меркурий, Венера, Марс, Юпитер и Сатурн.

Планеты выглядят как далекие звезды, но если мы знаем, что ищем, мы можем легко их обнаружить. Кроме того, поскольку эти планеты вращаются вокруг Солнца, их положение перемещается по небу в течение нескольких дней, месяцев или лет.

Фактически, их слежение и отслеживание на протяжении тысячелетий осуществлялось астрономами Рима и Древней Греции, которые видели в них богов.

Меркурий (у римлян, или Гермес у греков), планета, наиболее близкая к Солнцу и наиболее быстро движущаяся, была посланником богов, бродившим по небу между звездами.

Сатурн (у римлян, или Хронос у греков), видимая планета, наиболее удаленная от нас и, следовательно, также самая медленная для прохождения по небу, считалась божеством времени.

Марс (у римлян, или Арес у греков), заметно красный, был связан с войной (и две его луны, обнаруженные гораздо позже, были названы Фобосом, по Божеству страха, и Деимосом, олицетворяющим ужас).

Юпитер (у римлян, или Зевс у греков) был отцом и царем всех богов.

В этой статье мы дадим вам несколько советов, чтобы попытаться обнаружить планеты невооруженным глазом. Для некоторых планет это действительно не сложно!

Меркурий

Меркурий — самая близкая к Солнцу планета. Это иногда затрудняет наблюдение: Меркурий виден только ночью (обязательно), но, находясь близко к Солнцу, он виден и на небе. Поэтому он виден только сразу после захода солнца, но и не слишком рано, иначе свет солнца под горизонтом может замаскировать его. Также его можно увидеть прямо перед восходом солнца.

Когда он виден, Меркурий появляется относительно ярко, причем в плоскости эклиптики, то есть плоскости, содержащей все остальные планеты. Если вам удастся обнаружить Венеру (самую простую для обнаружения), у вас будет плоскость, и Меркурий тогда будет где-то на этой плоскости, рядом с Солнцем.

Как мы уже говорили, самая узнаваемая планета — Венера, о которой мы сейчас поговорим.

Венера

Венера, помимо Солнца и Луны, является самой яркой звездой на небе. Она настолько яркая, что ее регулярно путают с НЛО. Это потому, что Венера — самая близкая к нам планета, а ее поверхность очень хорошо отражает солнечные лучи.

Когда она видна на небе, Венера обычно является первой «звездой», видимой вечером, или последней, которая уходит утром, над горизонтом и на той стороне, где находится Солнце. Иногда это видно через полчаса после восхода солнца, когда солнце еще низко.
Длительность, в течение которой она видна, остается, тем не менее, значительно большей, чем у Меркурия: Венера видна до 3 часов после наступления темноты (или за 3 часа до восхода солнца), но вряд ли больше, потому что она также ложится спать.

И Венера, и Меркурий ближе к Солнцу, чем Земля. Поэтому их позиции на небе всегда близки к Солнцу. С заходом солнца на западе, если вы думаете, что видите Венеру на востоке вечером наблюдения, вы наверняка ошибаетесь: это не Венера. Если это особенно яркая звезда, есть шанс, что вы заметили Юпитер, который тоже очень яркий.

В любом случае если ночью на закате солнца или утром на восходе солнца вы видите необычайно яркую «звезду», то это определенно Венера.

Марс

Марс дальше от Солнца, чем Земля. Он всегда находится на плоскости эклиптики, которая содержит планеты и, следовательно, по оси, пересекающей небо, но эту планету можно увидеть в любом месте неба: как на стороне Солнца, так и на противоположной стороне. Поэтому его можно увидеть и посреди ночи, а не только во время захода и восхода нашей звезды, например, Венеры и Меркурия.

Марс прозван Красной планетой, и это не без оснований: Марс явно появляется на небе красным!
Этот цвет ему придает оксид железа, ржавчина, которая в целом красно-оранжевая и присутствует в больших количествах на поверхности планеты.

Марс — тоже небольшая планета, но близость к Солнечной системе означает, что он всегда хорошо виден, если, конечно, не скрыт Солнцем или за горизонтом.

Поскольку все планеты находятся на оси, пересекающей небо, нередки случаи, когда Марс иногда находится близко к Юпитеру, а иногда близко к Венере, иногда даже к Сатурну, а иногда в группах с Луной:

Планетарное соединение Марса, Юпитера и Сатурна с Луной, 20 марта 2020 года

В этих условиях легко увидеть эту светящуюся звезду рядом с очень яркой Венерой или Юпитером.

Наконец, и только для поэтической стороны, знайте, что когда вы наблюдаете планету Марс, есть небольшой шанс, что вас будут наблюдать обратно с Марса. Конечно, не марсиане или люди, а один из немногих марсианских роботов, которые годами пересекают его поверхность.

Эти роботы сделали несколько снимков и передали их на Землю, на которых изображена наша голубая планета на марсианском небе, также окрашенная в синий цвет из-за ее прекрасной атмосферы, в которой много CO2:

Земля сфотографирована с поверхности Марса

Марс на данный момент и по сей день является одним из двух миров, из которых была сделана фотография Земли с ее поверхности, первым из которых является Луна.

Юпитер

Огромная планета Юпитер (в 11 раз больше диаметра Земли, в 317 раз больше по массе) и ее относительная близость означают, что он всегда очень хорошо виден. Часто это четвертая яркая звезда на небе (после Венеры, Луны и Солнца). Юпитер не мерцает в небе, в отличие от звезд, и поэтому относительно узнаваем. Его видимый размер также является самым большим из всех видимых планет.

Как и Марс, Юпитер находится дальше от Солнца, чем от Земли. Поэтому Юпитер виден почти везде на оси планет и в любое время.

Наблюдая с помощью телескопа или даже хорошего бинокля, можно увидеть его большое красное пятно (если оно обращено к нам) и, возможно, четыре его самые большие луны: Ио, Ганимед, Европа и Каллисто.

Это галилейские луны, которые Галилею удалось наблюдать с помощью первого телескопа, который он изобрел (первоначально для военного флота) 400 лет назад. С тех пор вокруг Юпитера было обнаружено более 60 других лун, хотя они слишком малы, чтобы их можно было увидеть из дома.

Каждый из спутников — это отдельный мир. Особенно Европа, которая была бы хорошим кандидатом для приюта жизни, будучи согретой Юпитером. Ио, с другой стороны, настолько близко к своей планете, что он достаточно приливно-отливный, что является самой вулканически активной луной в Солнечной системе.

Если вы наблюдаете это в течение нескольких часов с помощью астрономического инструмента, вы сможете увидеть, как планета вращается, ее большое красное пятно появляется или исчезает, а ее спутники движутся по своей орбите.

Сатурн

Сатурн находится дальше, чем другие планеты, и также намного менее яркий. В зависимости от времени года он может быть даже затемнен или ослеплен Солнцем, что сделает невозможным его видеть в течение нескольких недель.

Сатурн наиболее яркий, когда его кольца также видны и освещены Солнцем, что увеличивает яркость всей планеты от Земли.
Это происходит, когда Сатурн и Земля находятся в оппозиции, т.е. по обе стороны от Солнца.

Как и другие планеты, Сатурн не излучает свет напрямую, а отражает свет Солнца. Поэтому он сияет иначе, чем другие звезды, не мерцая. Если вы привыкли к этому, то это относительно надежная подсказка, чтобы заметить планету.

Учитывая, что эта планета не является ни особенно окрашенной (она бледно-желтая), ни особенно яркой, обнаружить ее не так просто, как другие. Поэтому гораздо удобнее проверить его местоположение по (актуальной) небесной карте или специализированному приложению, а затем найти его.

А как же Уран? Нептун? А как же Плутон?

Уран слишком далеко, чтобы его можно было увидеть невооруженным глазом. Если мы знаем, где он находится, мы можем увидеть его с помощью телескопа, но он слишком далеко, чтобы что-то видеть.

Для Нептуна это еще сложнее. Эта планета, кстати, была обнаружена в результате вычислений до того, как была обнаружена с помощью телескопа: наблюдения Урана, похоже, показали нарушенную траекторию. Затем астрономы измерили возмущение на протяжении многих лет, выдвинули гипотезу о существовании новой планеты и начали вычисления ее положения (чтобы соответствовать аномалиям на орбите Урана). Он был обнаружен в 1846 году под одним углом от расчетного положения!

Наконец, для Плутона … эта бывшая планета — теперь «карликовая планета» — настолько далека от Солнца и от нас, что для его обнаружения требуется поисковый телескоп в несколько метров, и даже при этом его трудно различить. Его открытие датируется только 1930 годом.

КАК ВЛИЯЮТ НА НАС ЗВЕЗДЫ И ПЛАНЕТЫ

КАК ВЛИЯЮТ НА НАС ЗВЕЗДЫ И ПЛАНЕТЫ

Астрология. Рисунок Ганса Гольбейна, XVI век.

Сатурн и Марс — «злотворные планеты». Гравюра 1519 года. Астрологи до сих пор считают, что они приносят несчастье тем, кто родился под этими знаками.

По представлениям астрологов, планета, имеющая наибольшее влияние на человека, определяет его профессиональные склонности.

Астрология тесно связана с хиромантией — «искусством»предсказаний характера и судьбы человека по линиям ладони его руки.

Победа христианства над языческими небесными силами. Гравюра 1513 года.

Открыть в полном размере

В последние годы космическое влияние на Землю и ее биосферу стало «общим местом»: об этом пишут, снимают фильмы, его боятся. Сейчас человеческий страх эксплуатируют многие, в том числе и те, кто имеет отношение к изучению космоса. Некоторые научные коллективы, лишившись финансирования со стороны военных, пытаются разными способами привлечь к себе внимание и обеспечить свою работу. Речь не идет о продаже населению звезд — этим заняты откровенные проходимцы. Я имею в виду настоящих ученых, искренно болеющих за свое дело и порой перегибающих палку в общении с публикой исключительно из желания привлечь ее внимание к своим безусловно важным исследованиям.

Но в результате появляется раздутая до неприличных размеров астероидная опасность (кто не видел по телевидению, как бедный динозавр удирает от метеоритного дождя!), закрытые от солнца лица австралийских детей из страха перед озоновой дырой, ежедневные прогнозы геомагнитных бурь (на которые удобно списывать нарушения связи), долговременные прогнозы солнечной активности (непременно с драматическими нотками в голосе).

Все это делает нашу жизнь похожей на путешествие в утлом суденышке через бурный океан: того и гляди разнесет его в щепки «земное эхо солнечных бурь».

Разумеется, Земля живет не в вакууме; на нее падают метеориты и космические частицы, ее освещают Солнце, планеты и звезды. Их влияние на биосферу изучается. Если оставить в стороне очевидную связь жизненных процессов с солнечным светом, то все остальные «влияния» носят слабовыраженный, непредсказуемый или даже недоказанный характер [9].

Наиболее грамотные из астрологов уже поняли, что лучше не говорить о прямом влиянии звезд и планет на Землю — настолько оно незначительно. Теперь они предпочитают заклинания типа «космических ритмов», «звездных часов» и прочих указаний на непрямые и нефизические связи между биосферой и звездным небом. Однако я хочу вернуться к теме физического влияния планет и звезд на Землю, чтобы у читателя не осталось на этот счет сомнений.

Из всех видов физических взаимодействий сколько-нибудь серьезно можно говорить лишь о гравитации; остальные поля, потоки частиц и излучения от звезд и планет в окрест-ности Земли так слабы, что их регистрация даже чуткими современными приборами требует немалых усилий.

Чтобы ощутить гравитационное влияние Луны на Землю, нужно измерить разницу лунного притяжения в разных точках Земли. Она невелика: ближайшая к Луне точка земного шара притягивается к ней на 6% сильнее, чем наиболее удаленная. Эта разница сил растягивает нашу планету вдоль направления Земля-Луна. А поскольку Земля вращается относительно этого направления с периодом около 25 часов, по нашей планете с таким же периодом пробегает двойная приливная волна — два «горба» в направлении растягивания и две «долины» между ними. В твердом теле планеты и в открытом океане высота этих «горбов» невелика, всего около полуметра. Поэтому мы не замечаем приливов ни в океане, ни на суше. И только на узкой береговой полосе можно заметить приливы_отливы благодаря подвижности океанской воды, которая, набегая приливной волной на берег (скорость-то немалая, сотни метров в секунду!), может по инерции подняться на высоту до 16 метров.

Подобным же образом действует на Землю и Солнце, более массивное, но и более далекое, чем Луна. Высота солнечных приливов вдвое меньше, чем лунных. В новолуние и полнолуние, когда Земля, Луна и Солнце лежат на одной прямой, лунные и солнечные приливы складываются. А в первую и последнюю четверти Луны эти приливы ослабляют друг друга, поскольку «горб» одного приходится на «впадину» другого. Лунно-солнечные приливы — явление весьма заметное и важное в жизни Земли [10]. Например, под их влиянием Земля постепенно замедляет свое вращение; продолжительность суток увеличивается. Еще сильнее действует земная приливная сила на Луну: она уже давно замедлила свое суточное вращение настолько, что постоянно обращена к нам одной стороной.

Гигантские приливные эффекты, влияющие на движение планет, рождают иллюзию того, что малые живые тела уж и подавно должны управляться ими. В результате мы слышим от творцов «научной астрологии» наивные утверждения: «Луна вызывает приливные явления во всех жидкостных системах Земли — в океане, в полужидком ядре Земли, в каждой клетке организма, во всех межклеточных жидкостях» [11]. На основе подобных утверждений пытаются объяснить явление лунатизма, очень популярное в астрологии; предлагают «биологическую теорию приливов». При этом уровень аргументации таков: «Луна вызывает приливы на море, а человек также почти целиком состоит из воды, значит, и он должен испытывать родственное влияние» [12, 13]. Разумеется, вода здесь не при чем: земная поверхность, как мы уже знаем, деформируется приливом точно так же, как морская, разница лишь в том, что суша не может перетекать, поэтому приливная волна набегает на берег. Ну а в целом, с точки зрения физики, «биологическая теория приливов» выглядит просто смешно: ведь любой находящийся рядом с вами человек, например сосед по парте, оказывает на вас гравитационное приливное влияние приблизительно в миллион раз более сильное, чем Луна.

Еще менее серьезно выглядят утверждения о прямом приливном влиянии планет на Землю; для этого достаточно взглянуть на приведенную ниже таблицу. Суммарное действие всех планет не может вызвать на Земле прилива выше 0,045 миллиметра. А их влияние на конкретное живое существо исказит его форму не более чем на размер одного атома!

Теперь мы затронем несколько более сложный вопрос — опосредованное влияние планет на биосферу Земли, где в качестве «усилителя» используется Солнце. В 1920-х годах пионер гелиобиологических исследований в нашей стране А. Л. Чижевский писал: «Мы знаем, что периодическая деятельность Солнца —
процесс не вполне самостоятельный. Есть веские основания думать, что он находится в определенной зависимости от размещения планет Солнечной системы в пространстве, от их констелляции по отношению друг к другу и к Солнцу… Таким образом, и земные явления, зависящие от периодической деятельно сти Солнца, стоят, так сказать, под контролем планет… Исследования, проведенные с целью выяснения влияния планет на деятельность Солнца, дали вполне положительные результаты: в периодах солнечной активности обнаруживаются периоды планетных движений» [14]. По прошествии многих лет мы понимаем, что Чижевский проявил необоснованный оптимизм: неоднократные попытки связать солнечную активность с расположением планет так и не привели к ожидаемому результату.

Каково же реальное влияние планет на Солнце? Из приведенной выше таблицы видно, что даже если все планеты выстроятся в цепочку и их приливное влияние сложится, все равно высота приливного «горба» на поверхности Солнца составит не более 3 миллиметров. Несмотря на ничтожность этой величины, журналисты регулярно пугают «парадами планет» легковерную публику.

В 1974 году в США вышла книга Дж. Р. Гриббина и С. Х. Плэйжмана «Эффект Юпитера». В ней говорилось, что в 1982 году все планеты окажутся по одну сторону от Солнца, и этот «парад планет» вызовет на нем возмущения, убийственные для Земли. Минуло 10 марта 1982 года — момент наибольшего сближения всех планет. И, разумеется, ничего страшного не случилось — ни на Земле, где стихийные бедствия происходили обычным порядком, ни на Солнце: его активность под действием планет не изменилась. Новый апокалипсис обещали 11 августа 1999 года, когда «парад» совпадал с солнечным затмением. Затем «конец света» намечался на май 2000-го: «Когда Меркурий, Венера, Марс, Юпитер, Сатурн, Солнце и Луна выстроятся в одну линию, Земля содрогнется», — сообщала газета «Известия» от 29 мая 1998 года (N 97) со ссылкой на «The Sunday Times».

Мы еще не раз услышим такие прогнозы, эксплуатирующие основные инстинкты человека, один из которых — страх.

Для любителей физики, имеющих обыкновение ловить на безграмотности журналистов, отметим наивность самого понятия «парад планет». Приливная деформация растягивает тело вдоль одной оси, а вдоль перпендикулярных к ней направлений сжимает. Поэтому к такому же эффекту приводит и выстраивание планет на одной линии по разные стороны от Солнца (помните — лунные и солнечные приливы складываются в новолуние и полнолуние). Но современные астрологи не замечают этого. Около 70% приливного влияния на Солнце оказывают Юпитер и Венера. Максимальная высота прилива достигается, когда они лежат на одной прямой с Солнцем. Это повторяется примерно через каждые четыре земных месяца, однако никаких изменений солнечной активности с таким периодом не отмечено.

Да и трудно было бы ожидать заметного эффекта от приливного воздействия на Солнце: ведь энергия деформаций, которая ежесекундно рассеивается в его недрах, в тысячу раз меньше его термоядерной мощности. Но даже это не означает, что каждый «парад планет» увеличивает светимость Солнца на 0,1%, поскольку тепловая инерция солнечного тела составляет миллионы лет и сглаживает все подобные колебания светимости.

Наконец, обратив внимание на космические тела, расположенные за пределами Солнечной системы, мы не станем утруждать читателя упражнениями по физике, а просто скажем, что влияние звезд на нашу биосферу настолько мизерно, что никакие привычные масштабы с ним вообще не сопоставимы.

КАК РАЗОБЛАЧИТЬ АСТРОЛОГИЮ?

Для человека, воспринимающего рациональные аргументы, разоблачение астрологии не представляет труда: достаточно познакомиться со статистикой оправдываемости ее предсказаний. Вот результаты некоторых работ [15].

Психолог из Мичиганского университета Б. Силверман изучил влияние зодиакального знака, соответству ющего рождению каждого из супругов, на вероятность их бракосочетания или развода. Были использованы данные о 2978 свадьбах и 478 разводах, зарегистрированных в Мичигане в 1967-1968 годах. Ученый сравнивал реальные данные с предсказаниями двух независимых астрологов относительно благоприятного и неблагоприятного сочетания зодиакальных знаков для супружеских пар. Оказалось, что никакого совпадения между предсказаниями и реальностью нет, поэтому Б. Силверман заключил: «Положение Солнца на зодиаке в момент рождения не оказывает влияния на формирование личности».

Астрологи утверждают, что с помощью гороскопа можно определить предрасположенность человека к той или иной профессии. Если так, это сулит немалый экономический эффект. Вероятно, поэтому Дж. Беннет и Дж. Барт — экономисты из Университета Дж. Вашингтона — попытались выяснить, влияет ли положение планет относительно зодиакальных знаков на профессиональные склонности людей, в частности на частоту поступления юношей на военную службу. Особо тщательно изучались знаки, «управляе мые» Марсом. Это исследование астрологических предсказаний не подтвердило. Американский физик Дж. Мак-Джерви исследовал распределение дат рождения 17 тысяч ученых и 6 тысяч политических деятелей относительно зодиакальных знаков. Оно также оказалось совершенно случайным.

Проверялось и качество комплексного предсказания астрологами характера людей. С этой целью психолог из Чикаго Дж. Мак-Гру обратился в Федерацию астрологов штата Индиана. Участвовать в его экспериментах вызвались шесть опытных специалистов звездочтения. По просьбе Мак-Гру, 23 добровольца ответили письменно на анкету, содержащую как астрологические, так и традиционные вопросы о качествах их характера, работе и т. д. Затем время и место рождения добровольцев сообщили астрологам и шестерым членам контрольной группы, незнакомым с астрологией. После этого указанные в анкете характеристики добровольцев сопоставили с предсказаниями группы астрологов и контрольной группы. Результат получился следующий: предсказания астрологов оказались ничуть не точнее, чем предсказания членов контрольной группы, причем и те и другие совершенно не коррелируют с истинными качествами тестирован ных добровольцев. Самое же любопытное — характеристики одних и тех же добровольцев, данные разными астрологами, сильнейшим образом расходятся между собой.

Нужно заметить, что проверкой предсказательной силы «звездочтения» занимаются не сами астрологи, а «люди со стороны». Большинство ученых считают, что астрология, как прототип всех псевдонаук, вообще не заинтересована в точном обосновании своих основ. Ученых это не столько злит, сколько расстраивает: им просто непонятно, как может лженаука вроде астрологии процветать в самом технически развитом обществе за всю историю человечества?

Профессиональные ученые, пытающиеся найти в астрологии рациональное зерно, считают [16], что наиболее интересные результаты в этой области получены парижским статистиком М. Гокленом [17]. Гоклен изучил архивные данные, содержащие дату, время и место рождения 41 тысячи жителей Европы; среди них 16 тысяч
известных ученых, артистов, писателей, спортсменов и т. д., а также 25 тысяч «простых» людей. Он сопоставил положение планет и созвездий в момент рождения человека с типом его личности и родом занятий. Оказалось, что гороскопы совершенно лживы: нет никакой связи между характером и деятельностью человека и его знаком зодиака и расположением планет в момент рождения. Поэтому Гоклен отнес астрологию к разряду химер. Однако ему удалось подметить некоторые любопытные закономерности, дающие, как он полагает, право считать свою работу краеугольным камнем новой науки — космобиологии.

Оказалось, что у «простых» людей моменты рождения не зависят от конфигурации планет, а у знаменитых — зависят. Учтя известные демографам закономерности частоты рождения людей в разные дни года и в различное время суток, Гоклен установил, что выдающиеся представители своей профессии рождаются преимущественно при определенном положении некоторых планет относительно линии горизонта. Он показал, что положение Солнца, Меркурия, Урана, Нептуна и Плутона не влияет на профессию, а Луны, Венеры, Марса, Юпитера и Сатурна — влияет. Так, в группе из 2088 известных спортсменов многие родились, когда Марс восходил или был вблизи верхней кульминации. Для известных военных справедливо то же самое, но только в отношении Сатурна.

Выводы Гоклена неоднократно перепроверялись: одни исследователи частично подтверждали их, другие опровергали. Сам же Гоклен ищет возможность объяснения найденных им закономерностей на уровне генетической информации, которая, по его мнению, может управляться ритмами, едиными как для биологических объектов, так и для Вселенной. Ну что же, поиск — благородное дело; однако серьезных результатов на этом пути пока нет.

НУЖНО ЛИ «ВОЕВАТЬ» С АСТРОЛОГИЕЙ?

Итак, с точки зрения естествознания, астрология — пустоцвет, мыльный пузырь, лишенный рационального содержания. Там, где это возможно, наука создает методы прогноза и не окутывает их мистикой. А там, где невозможно, прямо об этом заявляет, не суля пустых надежд, как астрологи. Науке с астрологией не по пути. И если бы астрологи бессовестно не присваивали себе высокую репутацию, заработанную наукой, в частности астрономией, то и не было бы статей, подобных этой, и не обращали бы мы на них особого внимания, не выделяли бы из ряда других проявлений массовой культуры. Но когда диктор телевидения заявляет, что «сегодня по астрологическому календарю будет самый короткий день и самая длинная ночь», а бородатый астролог «назначает» на завтра солнечное затмение, хочется закричать: «Люди, при чем же здесь астрология? Это результаты нормальных научных расчетов, сделанных астрономами (покажите мне астролога, который самостоятельно может рассчитать хотя бы продолжительность дня, не говоря уже об обстоятель ствах солнечного затмения!). Люди, неужели вы думаете, что если астролог смог прочитать в Астрономичес ком календаре о завтрашнем затмении, то он так же легко сможет прочитать книгу вашей судьбы? Ведь эту книгу, в отличие от Астрономического календаря, не купишь в магазине».

Случается, противников астрологии ее адепты называют «догматиками и схоластами, не способными ощутить зарождение новой науки». Предоставляю читателю самому судить о справедливости этих обвинений.

То, что по привычке мы называем «борьба с астрологией», вовсе не равносильно стремлению искоренить ее. В данном случае позиция ученого состоит в желании оградить науку, ее «авторское право», ее честно заработанный авторитет от посягательства «незваных гостей», жаждущих эксплуатировать этот авторитет для своей корысти.

Как известно, ученые — скептики, а верующие — догматики. Именно поэтому наука и вера несовместимы. Они могут дополнять одна другую, но не вправе диктовать друг другу свои принципы. Эта мысль, очевидная теперь уже и для нас, россиян, казалось бы, разводит науку и веру (в широком смысле, а не только религиозную) в разные стороны, не оставляя им точек соприкосновения. Но это не так.

Дело в том, что положение науки и веры существенно различается. Наука практически не имеет на своем поле конкурентов: она с полной очевидностью доказала свою способность решать поставленные задачи. Попытки провозглашения «альтернативных», «неофициальных» наук — уфологии, парапсихологии и иже с ними — практически не задевают Большую науку.

В области веры совсем иная ситуация: на этом поле наблюдается жесточайшая конкуренция. А то, что бытующая в обществе астрология относится именно к этой области, признают даже весьма благожелатель но относящиеся к ней ученые: «Далеко не всем людям нужна истина, как она понимается в науке. В астрологии с древнейших времен присутствуют течения оккультно-мистического толка. Если человек чувствует себя комфортно в пределах такой идеологии и она помогает ему достойно нести тяжести жизни, то такая идеология имеет права на существование (коль скоро она не содержит в себе явных элементов антиобщественного)» [16].

Не будучи наукой, астрология ищет свою нишу, свой оригинальный образ и находит его на пути мимикрии, рядясь в ученые одежды, окружая себя компьютерами и наукообразной терминологией, но при этом полностью не признавая научный метод.

Трудно согласиться с высказыванием А. Л. Чижевского о том, что «астрология, если отбросить все ее мистические заблуждения, учит о связи всех вещей и явлений» [14]. Астрология без мистики уже не астрология, а нечто иное — космобиология, гелиобиология, ритмология, наконец, философия. Если же постоянно менять содержание какого-либо понятия, то в конце концов оно становится вообще бессодержательным. Сегодня, как и всегда, под астрологией понимают методику предсказания судьбы объекта по относительному расположению звезд и планет в момент его рождения. Иное содержание требует иных терминов.

Западная астрология зародилась в древнем Шумере, когда люди, не понимавшие причин происходящих вокруг них явлений, стали впервые нащупывать связи между, казалось бы, случайными событиями. Эта мотивация, вообще говоря, и в наше время стимулирует занятие как наукой, так и ее суррогатами (если человек не хочет или не может «играть по правилам» науки).

С этой проблемой лицом к лицу оказались педагоги: научные знания не создают надежного иммунитета к лженауке. Очевидно, следует уделять часть учебного времени критическому анализу псевдонаук. Путем простых опытов любой сможет сам легко убедиться, что гороскопы не способны предсказывать события на уровне выше случайных совпадений. Преподаватели должны попытаться понять причины увлечения астрологией, если хотят эффективно бороться с этой лженаукой, которая претендует на звание науки, не будучи таковой.

Литература

9. Владимирский Б. М., Темурьянц Н. А. Влияние солнечной активности на биосферу-ноосферу. — М.: Изд-во МНЭПУ, 2000.

10. Сурдин В. Г. Приливные явления во Вселенной. — М.: Знание, 1986.

11. Величко Ф. К. Астрология конца ХХ века // Астрология: за и против. — М.: Знание, 1990.

12. Lieber A. The lunar effect: biological tides and human emotions. — Anchor Press, 1978.

13. Антонов В., Ахмедов А. Гадание или предвидение // Наука и религия, 1981, № 7.

14. Чижевский А. Л. Земное эхо солнечных бурь. — М.: Наука, 1973.

15. Сурдин В. Г. Глупая дочь мудрой астрономии // Вестник АН СССР, 1990, № 11.

16. Владимирский Б. М. Мысли об иррациональном и рациональном в современной культуре, или Что делать астрофизикам с астрологией // Вселенная и мы, № 4, в печати.

17. Gauquelin M. Dreams and illusions of astrology. — Prometheus Books, 1979.

Ответы астрономов на вопросы | Большой новосибирский планетарий

Жанбуршинов Темирлан (Костанай Казахстан)

Вопрос: 08 января 2021 года примерно 07 часов 50 минут стал свидетелем падающей звезды, она была настолько яркой, что я подумал метеорит. Она пролетела продолжительное время. хотелось бы узнать название звезды?

Ответ: Здравствуйте. Вы верно пишете, что это мог быть метеор – то, что также литературно называется «падающей звездой» (сами звезды, как космическое тело, не имеют свойства «падать», внутри них происходят термоядерные реакции, но перемещение звезды это не вызывает). С конца декабря по начало января можно наблюдать метеорный поток Квадрантиды, представители которого «вылетают» из созвездия Волопаса (утром оно расположено в южном направлении) и отличаются средней скоростью пролета. Своему возникновению этот метеорный поток обязан астероиду Фаэтон 2003 ЕН, через шлейф остаточных частиц которого в этот период проходит Земля (именно эти частицы, оказываясь в атмосфере Земли, вызывают визуальный эффект «падающей звезды»). Такое название поток получил от названия неиспользуемого сейчас в астрономии созвездия Стенного Квадранта. Возможно представителя именно его Вы и наблюдали.

Куляшов Дмитрий (Анапа)

Вопрос: Подскажите, почему все планеты в Солнечной системе вращаются практически в одной плоскости? Если предположить, что материал для создания Солнечной системы в какой-то момент времени выглядел как диск, то видимо это осколок общего материала для создания Вселенной? И тогда возникает предположение, что во Вселенной все созвездия относительно друг друга параллельны (почти)? Тогда возникает вопрос — а не был ли самый первый взрыв направленным в определенной плоскости?

Ответ: Рождение Солнца и планет происходило не одновременно с рождением Вселенной. Наша Солнечная систем младше Вселенной почти на 10 млрд лет. Если придерживаться теории Большого Взрыва, благодаря которому, как считается, образовалась наша Вселенная, то согласно современным представлениям, он не был однонаправленным, а вещество разлеталось в разных направлениях. Строительным материалом будущего Солнца и планет (а возможно, что вместе с Солнцем родилась еще одна звезда) послужило вещество из оболочек более древней, взорвавшейся звезды. Законы распределения масс в протопланетном облаке, при его стремительном вращении вокруг молодого Солнца, заставляли вещество распределиться в виде диска с общим вращением против часовой стрелки, если представить, что мы смотрим на него с позиции земного шара, у которого северный полюс находится в верхней части. Позднее, когда из газопылевого протопланетного диска появились планеты, они продолжили движение в том же направлении.

Лебедев Артём (Аксу, Казахстан)

Вопрос: Здравствуйте! Я хочу поступить на астронома, мне сейчас 14 лет и я в 8 классе, мне хочется узнать какие предметы, науки нужно изучать, чтобы быть астрономом, физика знаю точно нужна, а вот какие ещё нет, спасибо!

Ответ: Артем, у Вас интересная и благородная цель. Астроном – редкая специальность. В России всего около 1,5 тысяч человек, имея диплом астронома, исследуют необъятные космические дали, а каждый пятый из них посвятил себя изучению Солнца. Вы правы, физика, как и математика, верные помощники астронома. Однако, астрономия – наука с ярко выраженным мировоззренческим характером, и находит отражение во многих других науках. Не зря многие ученые древности, изучавшие звездное небо, были еще и мыслителями (Пифагор, Эрастофен, Гиппарх и др.). Вы станете блестящим специалистом, если помимо математики и физики будете расширять свой кругозор среди других наук: химии, геологии, климатологии, экологии, биологии, истории и даже философии. Возможно к своему удивлению, вы обнаружите, как многое из привычного для нас интересно и логично связано воедино Вселенной.

Латынцева Динара (Новосибирск)

Вопрос: Здравствуйте. Почему принято изображать звезду пятиконечной? Когда я смотрю на звёзды, я отчётливо вижу 5 лучей или это моё воображение рисует такую форму и на самом деле звезда не пятиконечная? А какая она тогда по форме? Спасибо.

Ответ: Здравствуйте. «Лучистость» ярких звезд зависят не от воображения, а от особенностей восприятия света нашим органом зрения. Зрачок, как отверстие, через которое лучи попадают внутрь глаза, имеет не ровные, а волнистые края. Свет от точечных ярких источников, которыми являются и звезды, проходя через зрачок, приобретает изображение лучей. Чтобы убедиться в этом, можно провести небольшой эксперимент. Проколите отверстие с ровными краями в небольшом кусочке картона и посмотрите через него на яркую звезду. Если отверстие меньше, чем размер зрачка, то лучи, не касаясь стенок зрачка, будут падать на глазное дно, и звезда «потеряет» свои лучи. В репертуаре Большого новосибирского планетария есть полнокупольный фильм «Зрение», в котором интересно и увлекательно рассказано о движении света, работе зрения человека и астрономической оптике.

Дощанов Берик (Алматы)

Вопрос: Добрый день, с детства увлекаюсь астрономией и хочу приобрести в скором времени телескоп с хорошим разрешением. Вопрос: Как вы знаете, очень сложно найти на небе такие объекты как астероиды (имеются ввиду крупные Церера, Паллада и т.д ). Каким образом можно будет определить нахождение таких объектов на небе и существуют ли каталоги или программы, помогающие определить текущее местоположение объекта на определенную дату? Заранее спасибо!

Ответ: Здравствуйте. Карликовую планету Цереру в любительский телескоп увидеть вполне реально. В ее лучших условиях для наблюдения, которые можно отследить в астрономических календарях, она выглядит в телескоп, как крошечная звездочка. Однако, трудность может возникнуть с поиском этого объекта. Если телескоп не оборудован автоматической системой наведения на объекты (большинство любительских телескопов такой системы не имеют), то придется искать ориентиры и использовать их как привязки, чтобы попасть в объект, невидимый в небе (особенно в городском) невооруженным глазом. Чтобы узнать, какие объекты можно наблюдать в данное время, советуем воспользоваться Школьным астрономическим календарем или интерактивными программами-путеводителями по звездному небу. Среди них программа Stellarium, которая представлена в открытом доступе на сайте www.stellarium.org

Максим Калинин (Москва)

Вопрос: Как называется удаление планеты от Солнца на небесной сфере? 1.противостояние 2.стояние 3.квадратура 4.элонгация

Ответ: Термин «элонгация» применим к внутренним планетам (Меркурий и Венера) и обозначает удаление планеты на небесной сфере от Солнца. При этом различают западную элонгацию и восточную. Также элонгация может быть максимальной. Остальные «противостояние» и «квадратура» указывают на конфигурацию планет относительно Солнца. Термин «стояние» не является астрономическим.

Исакова Ксения (Курган)

Вопрос: Добрый день! Сегодня, 31.10.20, наблюдала рано утром на юго-востоке яркую «звезду». Она была высоко над горизонтом, других звезд видно не было, как и Луны. Это Венера?

Ответ: Добрый день. Вы определили все верно, самым ярким объектом утреннего неба сейчас выступает планета Венера, которую можно наблюдать на утренних сумерках до самого рассвета.

Суходольская Татьяна (Черкассы)

Вопрос: Добрый день! Справа от созвездия Орион находится яркая звезда (не совсем близко,ориентировочно), хотелось бы узнать, что это? Наблюдается после 5-ти утра. Спасибо!

Ответ: Добрый день. Орион — крупное созвездие, поднимающееся осенью во второй половине ночи и включающее в себя несколько ярких звёзд (астеризм созвездия формируют следующие звёзды: левая вершина — звезда Бетельгейзе с желтоватым оттенком, справа от нее более слабая, но достаточно яркая звезда Беллатрикс, нижняя правая вершина астеризма — яркая звезда Ригель, левее и слабее нее — звезда Сайф; три другие яркие звезды образуют Пояс Ориона). Немного выше и правее Ориона увидим яркую звезду Альдебаран, которая является главное звездой созвездия Тельца и, подобно Бетельгейзе, имеет желтоватый оттенок. Правее Ориона на заметном расстоянии сейчас располагается планета Марс, которая очень выразительно смотрится на ночном небе, имеет характерный рыжеватый оттенок и наблюдается на юго-востоке уже с наступлением темноты. Левее Ориона во второй половине ночи можно увидеть еще несколько ярких звезд ночного неба. Немного ниже и левее астеризма Ориона поднимается самая яркая звезда ночного неба — звезда Сириус в созвездии Большого Пса. Левее и выше Сириуса различим яркий Процион в созвездии Малого Пса, который вместе с Сириусом и Бетельгейзе образуют почти равносторонний треугольник, который представляет собой, так называемый, астеризм Зимнего треугольника, который в северном полушарии лучше всего наблюдается в зимний период. Выше и левее Проциона хорошо различимы две яркие звезды-соседки, расположенные словно друг над другом — это Поллукс и Кастор в созвездии Близнецов. Яркая звезда заметно выше них и примерно посередине расстояния до Альдебарана — это Капелла в созвездии Возничего. Все перечисленные звёзды являются одними из самых ярких на ночном небе северного полушария, они хорошо различимы в черте города и легко привлекают внимание наблюдателя.

Русакова Ксения (Корсаков)

Вопрос: Здравствуйте! Сейчас 15. 10.20 наблюдаю лунное затмение. Справа от Луны примерно под 45 градусов светит ярко звезда. Это Марс? И ещё, видела на юге сегодня три звезды в ряд. Это пояс Ориона? Вопросы не дают покоя, с утра облазил весь интернет. Спасибо за ответ.

Ответ: Здравствуйте! Ночью с 14 на 15 октября (фактически раннее утро 15 октября) правее и выше Луны, как Вами указано «под 45 градусов», находилась планета Венера, которая действительно очень яркая. Стоит сделать поправку, что Вы наблюдали не лунное затмение, а последние стадии фазы убывающей или иначе «стареющей» Луны, когда мы видим только от нее узкий серпик. Лунные затмения возможны только в полнолуния, но не каждый раз, и при этом диск Луны для наблюдателя остается виден целиком, но будто затемненным. 17 октября будет новолуние, после чего Луна будет «расти» и достигнет фазы полнолуния уже 31 октября. Три яркие звезды в ряд, которые Вы наблюдали на юге (если это было во время описанного наблюдения Луны и Венеры, которые располагались на востоке), действительно Пояс Ориона. Выше и ниже него Вы могли видеть другие яркие звёзды, которые вместе с Поясом напоминают фигуру большого повернутого банта — это основные звезды созвездия Ориона, которые формируют весь его астеризм (узнаваемую на небе фигуру). Левее Ориона можно встретить еще пару одних из самых ярких звёзд ночного неба — Процион в созвездии Малого Пса и ниже него самый яркий Сириус в созвездии Большого Пса.

Завьялов Александр Петрович (Новосибирск)

Вопрос: Сейчас, в конце сентября над городом Новосибирском на юго-восточном направлении недалеко от Луны видна крупная красноватая звезда- не мерцает. Хотелось бы узнать что это? Марс или допустим станция, которую вроде писали, что будет видно.

Ответ: Здравствуйте. Вы верно определили объект – это действительно «красная» планета Марс. Сейчас её хорошо видно в юго-восточном направлении с наступлением темноты, а 14 октября Марс окажется в противостоянии: таком положении, когда планета располагается примерно на одной линии с Солнцем и Землей и на достаточно близком расстоянии от Земли, которое в этот раз составит порядка 62,5 млн км. Этот период считается наиболее благоприятным для наблюдения Марса в телескопы. Но даже для наблюдателя с «невооруженным глазом» планета предстанет третьим по яркости объектом ночного неба (после Луны и Венеры), достигнув блеска -2,6m звездных величины.

Акопов Александр Александрович (Подольск)

Вопрос: Доброго Здравия! Проживаю в г. Подольск Московская области. Наблюдаю сегодня 23. 09.20 г. утром 5-40, перед восходом Солнца яркую звезду, практически в зените. Подскажите, что это за звезда или планета? Благодарю!

Ответ: Здравствуйте. К концу ночи высоко в небе, как Вы указали «практически в зените», можно наблюдать яркую звезду Капеллу в созвездии Возничего. Но также почти до самого рассвета значительно ниже в восточном направлении можно наблюдать планету Венеру, в этот промежуток времени она представляется самым ярким объектом звёздного неба.

Матвеева Вера Сергеевна (Воронеж)

Вопрос: Добрый день! Отец интересуется, что за звезду видно на востоке Воронежской области примерно в 3 часа ночи. В середине сентября.

Ответ: Добрый день. В восточной части неба в это время можно наблюдать несколько ярких объектов. Например, Альдебаран в созвездии Тельца, который имеет своеобразный оранжеватый оттенок. Выше и левее него встретим яркую Капеллу в созвездии Возничего, она же сияет белым светом. Так же чуть выше и на большем расстоянии справа от Альдебарана можно наблюдать планету Марс с ее характерным рыжеватым цветом. Чуть позже под Альдебараном также поднимается созвездие Ориона с его яркими представителями, например, звездой Бетельгейзе, которая тоже имеет оранжеватый оттенок.

Зайцева Елизавета (Калуга)

Вопрос: Добрый вечер! Сегодня, 12.09.20, примерно в 00:20 я находилась на улице и, посмотрев на небо, увидела непонятное мне явление: пару десятков каких-то светил, в виде «тучи», похожих на заезды, быстро летели в одном направлении и беспрерывно поочередно мерцали. До этого дня никогда такого не замечала. Хотелось бы узнать об этом поподробнее. Заранее спасибо.

Ответ: Здравствуйте. Описанное Вами «явление» напоминает пролетающие воздушные шары со светодиодами. Обычно у них прозрачная оболочка, невидимая в темном небе. А так как шары запускаются массово, то и на воздушных потоках они летят словно «роем», что для наблюдателя представляются маленькими сияющими точками, плавно и дружно куда-то летящими.

Семикозова Ольга Евгеньевна (Кодинск)

Вопрос: Здравствуйте. Ночью с 3 на 4 июля Луна была ярко оранжевого цвета, но за не сколько минут стала бордового цвета. А потом то исчезла в течение 2-3минут (как будто её ластиком стирали), то опять появлялась. И так было 4 раза за час. Объясните почему. Заранее спасибо.

Ответ: Здравствуйте. Цвет светил в небе зависит не только от свойств самих объектов, но и от свойств атмосферы. Если цвет Луны меняется быстро (на глазах), то скорее всего это атмосферный эффект. Вероятно, между Луной и наблюдателем в атмосфере появилась дополнительная среда (облако, дым, реактивный след самолета), проходя через которую свет мог изменяться, придавая объекту необычный вид. Временное исчезновение и появление объекта так же могло быть чисто визуальным эффектом и вызвано переменной облачностью.

Нартикоева Адриана (Владикавказ)

Вопрос: Здравствуйте! Много вопросов себе задаю, смотрю разные ролики и возникают вопросы. 1) Правда что, если есть где-то в конце нашей галактики разумная форма жизни, то чтобы добраться до нас при наличии беспилотных кораблей со скоростью 20% от скорости света, может понадобиться больше 20 лет. А возможность транспортировать живые организмы на такой скорости опасно для жизни. И поэтому развитые цивилизации осознали что это бесполезно и просто сдались? 2) В фильме «Интерстеллар» нам показывают Кротовую нору, которая позволяет переместиться из Солнечной системы в отдаленные участки Вселенной. Возможно ли такое ? 3) Если у нас есть приборы, которые могут увидеть приближение астероидов на нашу планету, то что мы сможем сделать если какой-нибудь астероид сможет столкнуться с Землей, есть ли у нас что-то , что сможет сбить с курса астероид, если нет то создается ли? Заранее спасибо.

Ответ: Здравствуйте. 1) Вероятно, что до сих пор мы не встретились с развитыми цивилизациями по простой причине: в окрестностях нашей Солнечной системы на десятки световых лет их просто нет. Уже несколько десятилетий с развитием радиоастрономии человечество имеет возможность принимать радиосигналы из глубин космоса. Сигналов искусственного происхождения до сих пор не обнаружено. Это значит, что вопрос о существовании наших «братьев по разуму» пока остается открытым. 2) Гипотеза о существовании Кротовых нор дополняет представления о влиянии на вещество экстремальных условий в Черных дырах. Однако существование Кротовых нор не имеет четкой математической модели и не подтверждено наблюдениями. 3) Большинство астероидов (каменных тел имеющих размеры 100 метров и более), угрожающих нашей планете своими близкими орбитами уже выявлено и каталогизировано. Отслеживать тела меньшего размера пока нет возможности. Однако, вероятность столкновения с телами менее 100 метров в нашу эпоху минимальна. Тем не менее, вопрос астероидно-кометной опасности периодически поднимается, но решить его под силу только объединенными усилиями целого ряда государств. Насколько нам известно, в настоящее время не существует систем, упреждающих столкновение нашей планеты с малыми телами Солнечной системы, а работы в данном направлении сводятся к выявлению потенциально опасных астероидов и комет и их каталогизации.

Мария Владимировна Дюзенли (Екатеринбург)

Вопрос: Здравствуйте! Вчера (05.09.2020) я видела в 23:00 красно-оранжевую точку слева от Луны. Что это?

Ответ: Здравствуйте. Вы наблюдали планету Марс.

Фильченкова Ярослава (Кропивницкий)

Вопрос: Добрый день. 09.06.20 около 23:20 я видела одну, достаточно яркую, что бы её можно было заметить, точку. Она двигалась достаточно быстро, но ровно, я думала что это самолёт или что-то в этом духе, пока вдруг она резко не изменила свою траекторию, выгнувшись зигзагом. Я подумала, что свихнулась, но позвала отца, и оказалось, он видит тоже самое. Далее я увидела ещё одну точку, двигающуюся так же само, как и выше названая. Они летели клином, пересекаясь в одной точке. Далее одна из них исчезла из моего поля зрения, но вторая продолжала двигаться так же странно и замысловато, как и в начале. Что это может быть?

Ответ: Здравствуйте. Одними из наиболее вероятных вариантов наблюдаемых Вами объектов могли быть летающие фонарики со свечками. Эти объекты хорошо различимы в небе, в зависимости от высоты могут иметь вид точки или маленького шарика. Траектория их движения может неожиданно меняться в зависимости от ветра. Если пламя приглушается ветром, то для наблюдателя объект может «исчезать», а потом неожиданно появляться в другом месте. Также уменьшение яркости пламени от ветра может восприниматься нашим сознанием как удаление объекта и тем самым придавать объекту загадочный образ.

Звездинская Стелла Аркадьевна (Южный, Одесская область)

Вопрос: Здравствуйте! Спасибо за проект и возможность задать вам вопрос! Очень хочется узнать, что за яркая звезда сверкает перед рассветом точно на юго-востоке, как раз посредине между прекрасными Венерой и Марсом. Объект большой, яркий и очень сильно сверкает, иногда и цветными лучами, движется по небосклону как и все тела и с рассветом, когда остальные звезды уже не видны, а красуются лишь две планеты, он тоже горит почти так же ярко, пока не скроется в дневном свете. Очень интересно, что это за звезда, но не могу найти информацию. Буду очень признательна за ответ, заранее благодарю.

Ответ: Здравствуйте. Благодарим за Ваш интерес к нашей рубрике! Звезда, которая сейчас располагается посередине между Венерой и Марсом примерно на одном уровне с ними — это Альдебаран, главная звезда созвездия Тельца. Он имеет слегка оранжевый оттенок и вместе с другими яркими звёздами этого созвездия, расположенными неподалеку, образует крупное рассеянное звёздное скопление Гиады, которое напоминает по форме букву «V», лежащую на боку. Чуть выше и правее Альдебарана хорошо различимо еще одно компактное, но выразительное звёздное скопление Плеяды, напоминающее по своему звёздному рисунку маленький ковшик. Помимо Альдебарана в этой же юго-восточной стороне неба ближе к горизонту встретим еще несколько красивых ярких звёзд, которые могут привлечь внимание наблюдателя. Под созвездием Тельца увидим большое созвездие Ориона, которое во второй половине ночи полностью поднимается над горизонтом. Одна из его ярчайших звёзд находится примерно на одной прямой под Альдебараном — это Ригель, который сияет белым светом; слева от этой воображаемой линии и примерно посередине встретим главную звезду Ориона — яркий Бетельгейзе, который является красным сверхгигантом и имеет характерный рыжеватый оттенок. Видимая фигура созвездия Орион представляется большой бабочкой или бантом, стоящим на боку. Между Бетельгейзе и Ригелем хорошо виден, так называемый, астеризм Пояса Ориона из трех звёзд, который отмечает как раз середину этой бабочки. Под Орионом и у самого горизонта уже ближе к концу ночи поднимается самая яркая звезда ночного неба — Сириус в созвездии Большого Пса, сейчас он виден недолго, но уже с середины осени будет украшать наше ночное небо более длительный промежуток времени. Еще одна яркая звезда в этом направлении — Процион в созвездии Малого Пса — его найдем на некотором удалении слева примерно на одинаковом расстоянии от Сириуса и Бетельгейзе. А чуть левее и выше Проциона сейчас как раз располагается самый яркий объект актуального ночного неба — планета Венера. Для будущих удачных наблюдений и быстрой ориентации в объектах звёздного неба рекомендуем веб-версию программы Stellarium: www.stellarium-web.org Желаем чистого неба и увлекательных наблюдений!

Гостев Максим (Челябинск)

Вопрос: В 23:00 29.08.20 наблюдал яркое оранжевое свечение на востоке по размерам на много больше звезды. Что это было?

Ответ: Здравствуйте. В это время на востоке в Челябинске уже поднимается планета Марс, которая выглядит как достаточно крупная звезда и имеет оранжевый или желтовато-алый оттенок.

Диля Холмуратовна (Лиссабон)

Вопрос: Здравствуйте, я хотела узнать что за яркая звезда мигающая на северо-востоке над горизонтом 22-30 часа 25. 08.2020 Португалия, Лиссабон.

Ответ: Здравствуйте. В 22:30 в северо-восточном направлении недалеко от горизонта можно видеть главные звёзды созвездия Персея: более яркий Мирфак и правее и ниже его сосед Алголь; выше Персея хорошо видны звёзды созвездия Кассиопеи, напоминающие своим астеризмом букву «W», а правее Персея можно рассмотреть яркие звёзды созвездия Андромеды. Но наиболее примечательной из звёзд на северо-востоке является главная звезда созвездия Возничего — Капелла, она восходит ближе к 00 ночи, является самой яркой звездой в том направлении и для наблюдателя как будто переливается или мигает.

Денисова Галина Виссарионовна (Козьмодемьянск)

Вопрос: Очень яркое небесное тело, по карте созвездий не смогла определить, там нет на данном месте никаких ярких звёзд, почитала в разных источниках, какие сейчас планеты видны, тоже не нашла… В общем, если проводить линию от Кастора до Бетельгейзе, что-то очень яркое почти по середине, и с Капеллой и Альдебараном образует равносторонний треугольник. Подскажите, пожалуйста это? Заранее спасибо)

Ответ: Добрый день! Вы все верно определили, что иных ярких звёзд там нет. Сейчас в этой части неба располагается планета Венера — именно ее Вы и наблюдаете. Она словно космический прожектор сияет ярче любых звёзд, поднимаясь во второй половине ночи, и видна почти до восхода Солнца.

Васильева Ирина (Москва)

Вопрос: Здравствуйте! Сегодня, 18 августа 2020 года, над Москвой в южном направлении, в ночном небе видна очень яркая звезда. Подскажите пожалуйста, как она называется. Спасибо.

Ответ: Здравствуйте! В южном направлении сейчас с наступлением темноты недалеко от горизонта можно наблюдать планету Юпитер — он виден яркой крупной точкой, может быть со слегка желтоватым оттенком. Недалеко от него слева также можете различить Сатурн, но он уже представляется более слабой «звёздочкой». Выше от горизонта также в южном направлении можно увидеть одни из ярких летнего ночного неба: Альтаир в Орле (чуть выше и чуть ниже него на небольшом расстоянии еще видны две звезды почти на одной линии, образующие астеризм «Коромысло весов»), а еще выше почти в зените Вегу — в созвездии Лиры.

Нотвин Иван Константинович (Новосибирск)

Вопрос: Сегодня в четыре ночи в Нск, если смотреть на восток с района около планетария, видна невероятно яркая немигающая точка. Что это может быть?

Ответ: Здравствуйте. В это время на востоке уже хорошо видна планета Венера, которая в тот момент выступает самым ярким объектом звёздного неба, и именно ее Вы и наблюдали.

Санжинимаев Батор Викторович (Чита)

Вопрос: Здравствуйте, хотел бы спросить, какое явление мог видеть? Вышел на улицу, глядя в ночное небо заметил что три объекта, очень похожие на звёзды, двигались по небу, и движение было прямо по линейки, а сами ярко горели, за ответ заранее большое спасибо)

Ответ: Здравствуйте. Вероятнее всего Вы видели какие-то искусственные спутники, которые в большом количестве работают и летают на околоземной орбите. Скорее всего это спутники единой связанной системы, возможно, из группы спутников «Starlink». Определить какие именно это были спутники с общего описания маловероятно. Более точно отследить или проверить пролеты разных спутников над своим городом можно в программе «Stellarium» (www.stellarium.org), задав конкретную дату и время наблюдения, а также на сайте heavens-above.com. Желаем удачных наблюдений!

Тарабрина Наталья (Петрозаводск)

Вопрос: Добрый день, отдыхаем в Крыму, п. Песочное. 16.08.2020 около 22.30 в ночном небе над Черным морем наблюдали необъяснимое явление: как будто стайка-большая группа звёздочек (2 яркие и около 30 менее ярких) двигалась с юга над морем, меняя форму. Далее этот двигающийся объект или группа объектов постепенно замедлило движение, пока совсем не остановилось. При этом первоначальное положение звёздочек относительно друг друга постоянно менялось. Мы прекратили наблюдение, когда звёздочки побледнели и долгое время оставались неподвижными. Все продолжалось около 5 минут. Наблюдали вшестером. Поясните, пожалуйста, что мы видели?

Ответ: Здравствуйте. По представленному описанию можно судить о том, что Вы наблюдали не космическое явление. Яркие астрономические объекты, такие как звёзды и планеты, являются статичными на ночном небе и меняют свое видимое расположение лишь за счет осевого вращения Земли. Космические аппараты, такие как искусственные спутники Земли, Международная космическая станция, плавно двигаются по заданным им траекториям и не могут резко менять своего положения в ходе полета. Скорее всего Вы могли наблюдать какой-то массовый запуск небесных фонариков или воздушных шаров со светодиодами. Последние обычно имеют прозрачную оболочку, не видимую в тёмном небе, а свечение диодов напоминает россыпь мелких, хорошо различимых звёзд, которые за счет движения в воздушных потоках легко меняют свое положение на небе и относительно друг друга, представляя внимательному наблюдателю необычное красивое зрелище.

Терешина Дарья Викторовна (Кировск, Ленинградская область)

Вопрос: Доброго времени суток! Одним теплым вечером (15 июля этого года) с семьей решили покататься по Неве на лодке. Дело уже близилось к закату. Вдруг на небе стало видно слегка заметное свечение. Исходило оно от уже ушедшего солнца и напоминало зеленый столб света. Со временем этот столб менял свой угол относительно горизонта. Поискав информацию в интернете, мы пришли к выводу, что это был солнечный столб, но все еще сомневаемся в своих догадках. К удивлению, на фотографии, сделанной в этот вечер, было видно это свечение. Никогда подобного не видели, поэтому хотелось бы получить ответ, что же это было. Заранее спасибо за ответ)

Ответ: Здравствуйте! По указанному описанию Вы действительно наблюдали, так называемый, зелёный луч – оптическое явление, возникающее в самый последний момент захода Солнца или перед самым его восходом. Происходит это явление в связи с преломлением солнечных лучей и их разложением на спектр в атмосфере. Когда Солнце уже находится у горизонта, коротковолновые лучи из спектра — фиолетовые, синие, голубые — успевают рассеяться, не добираясь до Земли. В итоге последний луч заходящего Солнца имеет зеленый цвет. Среди условий наблюдения зеленого луча необходим открытый горизонт, благоприятная погода и чистый воздух. Явление может представляться в форме столба, сегмента или зеленого края на солнечном диске. Обычно это явление достаточно короткое, около нескольких секунд, но при определенном удачном перемещении наблюдателя можно видеть его чуть более продолжительное время.

Рахымов Гурбансахет Юсупович (Туркменистан, Лебап)

Вопрос: Здравствуйте! Сотни звезд выстроились по Туркменскому времени около 4:15 утра 16 июня этого года. Они как цепи, их движения как поезда. Что это может быть?

Ответ: Здравствуйте. В указанное время можно было наблюдать пролет группы спутников «Starlink-8», которые ранее 13 июня были запущены и выведены на околоземную орбиту компанией «SpaceX» в количестве 58 штук. Первые дни спутники хорошо видны и двигаются вереницей друг за другом. Для наблюдателя с Земли пролеты этих только запущенных спутников представляют необычное яркое зрелище. Со временем каждый из них расходится на собственные заданные траектории, постепенно формируя единую сеть интернет-доступа с ранее запущенными спутниками.

Семёнов Сергей Анатольевич (Белореченск)

Вопрос: Я сейчас нахожусь в Воронежской области. Сегодня ночью в 3 часа наблюдал интересный объект. Он двигался с запада на восток. Состоял из множества маленьких объектов, расположенных на одной линии (как бусы). Когда начал приближаться к восточной стороне неба постепенно начал терять яркость. Пожалуйста, подскажите, что это было.

Ответ: Здравствуйте. В указанных данных наблюдений (место и время) 8 августа можно было наблюдать пролет очередной группы спутников «Starlink-9», запущенных на околоземную орбиту накануне 7 августа. Первое время спутники предстают выстроенной линией плавно плывущих белых точек, чем привлекают внимание наблюдателя. С мая 2019 года спутники «Starlink»регулярно запускаются компанией «SpaceX» группами около 60 штук с целью формирования единой глобальной системы обеспечения высокоскоростного широкополосного доступа к сети интернет.

Пищулин Ярослав Валерьевич (Киев)

Вопрос: Почему круглый год на небе видны одни и те же звезды? Разве, если планета вращается вокруг Солнца, они не должны меняться?

Ответ: Здравствуйте. Вы совершенно правы: в соответствии с движением нашей планеты по своей орбите, то есть её вращения вокруг Солнца, звёздное небо для наблюдателя должно меняться. И оно действительно меняется, но не полностью, а только его часть. Все созвездия, видимые в нашем северном полушарии, делятся на две группы. Одна из них — это околополярные незаходящие созвездия, те созвездия, что находятся над полюсом и никогда не опускаются ниже горизонта. Эти созвездия мы видим в течение всего года, к ним относятся Большая и Малая Медведицы, Дракон, Кассиопея, Цефей и Жираф. В южном полушарии есть свои незаходящие созвездия, всегда видимые там, но никогда не видимые у нас. Все остальные созвездия являются заходящими, иначе называемые также сезонными созвездиями. Они, в ходе годичного вращения Земли и наклона ее оси относительно Солнца, меняют свое положение на небе (высоту расположения) и, в соответствии со временем года, видны те или иные созвездия. Например, крупное созвездие Ориона для нас считается зимним и имеет лучшую видимость зимой, а летом в северном полушарии его не видно, в то же время такие созвездия как Лира, Лебедь и Орел, яркие звёзды которых составляют астеризм Летне-осеннего треугольника, лучше видны в летний период. Некоторые яркие звёзды заходящих созвездий могут наблюдаться почти в течение всего года, однако, при этом они меняют свое положение относительно горизонта, располагаются ближе к нему, а, соответственно, меняется и продолжительность их видимости.

Мотренко Сергей (Таганрог)

Вопрос: Добрый вечер! Сегодня, 10 августа, в 1⁰⁰ ночи наблюдал звезду на 2 часа от Луны. Подскажите, пожалуйста, как она называется. Спасибо.

Ответ: Здравствуйте. В день Ваших наблюдений справа от Луны можно было наблюдать Марс, который представляется достаточно яркой немигающей точкой со слегка алым или желтоватым оттенком.

Смирнова Марина (Кострома)

Вопрос: Здравствуйте! Помогите, пожалуйста, настроить телескоп Synta Protostar 50AZ. В какой последовательности и какие окуляры ставить, чтобы увидеть хотя бы Луну?

Ответ: Марина, здравствуйте! Вы обладаете простым, но надежным телескопом. Он сможет показать Вам целый набор основных астрономических объектов, доступных для широкого круга астрономов-любителей. Ознакомиться с ними и временем их наблюдения можно в литературе для начинающих или в очередном выпуске Школьного астрономического календаря. Однако, после выбора места и времени наблюдения, телескоп, как Вы верно заметили, нужно настроить. Для начала стоит изучить инструкцию, прилагаемую к телескопу, чтобы знать его устройство и назначение элементов. Для того, чтобы найти объект в небе, нужно воспользоваться искателем. Однако, каждый раз перед наблюдениями его необходимо настраивать. Телескоп и искатель должны «смотреть» в одном и том же направлении. Для этого установите в телескоп окуляр с самым большим значением миллиметров, указанном на нем. Кратность приближения при этом будет минимальной из возможного. Направьте телескоп на далекий наземный объект (макушка дерева, далекая труба, кран, рекламная вывеска) таким образом, чтобы объект оказался в центре поля зрения. Путем вращения настроечных винтов искателя, добейтесь того, чтобы его перекрестие оказалось на объекте, а картинка в искателе в миниатюре повторяла картинку в поле зрения телескопа. Теперь искатель и телескоп согласованы и без труда, найдя в искателе объект, Вы сможете его наблюдать в телескоп. Это правило настройки искателя общее для многих телескопов. Однако найти объект еще не достаточно для наблюдений. Его надо еще сопровождать (поворачивать за ним телескоп). Ваш телескоп оборудован азимутальной монтировкой, о чем свидетельствуют буквы «AZ» в названии. Это значит, что всякий раз, когда объект будет уходить из поля зрения, Вам нужно будет перемещать за ним телескоп по двум плоскостям: вверх и вправо или вниз и вправо. К этому надо быть готовым и удобней устроиться возле телескопа во время наблюдений. Рекомендуем также познакомиться со статьей нашей рубрики «Шпаргалка астронома» по сходной теме «Сборка и настройка телескопа»: https://vk. com/@nebonsk-shpargalka-astronoma-2-sborka-i-nastroika-teleskopa Желаем ясного неба и интересных наблюдений!

Киселева Анастасия Александровна (Москва)

Вопрос: Добрый день. Сегодня 10.08.2020 года, в 2-45 над Москвой наблюдала интересное и загадочное для меня явление: в ряд, с одинаковой скоростью и по одной и той же траектории летели более 25 точек…синхронно, на одинаковом расстоянии друг от друга…попыталась заснять видео на телефон — но, к сожалению, он не справился с темнотой…прошу вас, расскажите об этом явлении. Заранее благодарю вас!

Ответ: Анастасия, здравствуйте. Вы наблюдали пролет спутников Starlink, запущенных ранее 7 августа компанией SpaceX. Первые несколько дней спутники двигаются друг за другом вереницей, постепенно расходясь по заданным им рабочим орбитам. Вся система спутников Starlink предполагает создание единой связанной сети высокоскоростного широкополосного интернет-доступа. С мая 2019 года это был уже десятый массовый вывод спутников на околоземную орбиту, каждый раз включающий порядка 60 штук. Отследить грядущие видимые пролеты этих спутников можно в открытом доступе на сайте: www.heavens-above.com Отметьте свой город, выберите соответствующую группу спутников и планируйте следующие наблюдения!

Кочергин Павел Валерьевич (Мурманск)

Вопрос: Добрый день. Подскажите, пожалуйста, сейчас нахожусь в Белгородской области и в южном направлении, на данный момент, на одном уровне с луной видны две звезды, права от луны если точнее, появляются в вечернем небе одни из первых. Правая звезда более яркая. Как называются эти звезды? Заранее спасибо.

Ответ: Здравствуйте! В южном направлении сейчас располагаются планеты Юпитера и Сатурн. Именно их вы и наблюдали рядом с Луной в начале августа. В июле эти планеты прошли свои точки противостояния и условия для их наблюдения стали наболее благоприятными. Даже в черте города эти планеты с наступлением темноты хорошо видны невооруженным глазом, более яркий из них — это Юпитер, а слева от него — Сатурн.

Бугаков Максим Вадимович (Воронеж)

Вопрос: Добрый день. 20.07.2020 я выбрался на вело прогулку в Воронеже, так как наслышан о пролетающей комете весь вечер старался высмотреть ее в небе, но около 23.00 заметил другой объект который и вызвал у меня вопрос. В очередной раз посмотрев на небо я увидел 3 очень яркие звезды, образующие треугольник, почти в зените, удивило меня то, что до этого я их там не видел весь вечер, а по яркости они не уступали планетам солнечной системы во время парада, еще больше меня поразило то, что в течение каких-то секунд эти звезды начали затухать, я уже было подумал что мне показалось и глаза просто с непривычки так ярко увидели эти звезды, но в итоге они пропали совсем. Я бы решил что мне показалось, но конец этого действия наблюдал я уже не один успев обратить внимание друзей. Размером этот треугольник был не маленький и сопоставим с размером ковша Медведицы Большой примерно. Вероятнее всего это был какой то искусственный объект, но как я не высматривал его еще раз больше не увидел, потому решил поинтересоваться, что же это могло быть?

Ответ: Здравствуйте, Максим. По Вашему описанию можно достаточно точно ответить, что Вы наблюдали известный астеризм Летне-осеннего треугольника — хорошо узнаваемую фигуру большого треугольника из ярких звёзд, который виден в летний и осенний период. Звёзды, которые его образуют, действительно одни из самых ярких в северном полушарии: правую вершину треугольника занимает наиболее яркая звезда Вега в созвездии Лиры (вторая по яркости звезда летнего неба в нашем полушарии после Арктура в созвездии Волопаса — его можем наблюдать слева от ручки ковша Большой Медведицы, как будто скатившись с нее), левую вершину треугольника обозначает Денеб в созвездии Лебедя, а нижний угол треугольника занимает Альтаир в созвездии Орла (который вместе с двумя другими соседними звёздами — выше (Туразед) и ниже (Альшаин) самого Альтаира — формирует еще один небольшой астеризм «Коромысло весов»). Все эти звёзды и астеризм треугольника легко различимы при хорошей погоде даже в черте города. Неожиданное «исчезновение» звёзд, вероятнее всего, объяснимо заходящей на небе облачностью. Рекомендуем провести повторные наблюдения в ближайшую ясную ночь, проверить расположение этих звёзд и убедиться в их стабильности на нашем небе.

Тимофей Максимов (Тамбовская область)

Вопрос: Здравствуйте. У меня такой вопрос: я живу в Тамбовской области и уже три дня наблюдаю странный луч, но звезду нет, точнее он находится правее Большой Медведицы. Спасибо заранее!

Ответ: Здравствуйте. Вы наблюдаете комету C/2020 F3 (NEOWISE), точнее ее хвост, который выглядит достаточно рассеянным лучом, расширяющимся кверху. Внизу нее при достаточно темном небе или при наличии приближающего оптического прибора можно рассмотреть и голову кометы, представленную более яркой точкой. Подробнее о комете, ее видимости и иную полезную информацию можно почитать здесь: vk.com/nebonsk?w=wall-29943951_13814%2Fall

Махмутова Лилияна (Нурлат Татарстан)

Вопрос: Ночью я увидела звезду, над которой идёт полоса будто она падает, но сама стоит на месте. Что это может быть?

Ответ: Здравствуйте. Вы наблюдаете комету C/2020 F3 (NEOWISE), которая была обнаружена в марте 2020 года и в начале июля прошла свой перигелий (ближайшую точку относительно Солнца), благодаря чему на протяжении некоторого времени была очень хорошо видна невооруженным глазом. К концу июля яркость кометы будет постепенно снижаться и в августе она станет различима только с помощью оптических приборов. Эта комета признана самой яркой за последние 7 лет и представляет действительно удивительное зрелище, особенно, при наблюдении в отдаленных от городской засветки местах.

Громенко Игорь Евгеньевич (Челябинск)

Вопрос: Здравствуйте! Вчера увидел звезду, от неё будто идёт луч. Видел всего 2 раза. Вчера (16.07) и позавчера (15.07.20). Как-будто прожектор подняли высоко в небо и начали им светить. Подскажите пожалуйста, что это может быть? Как называется звезда? И почему я не видел ее до этого столько лет?

Ответ: Здравствуйте! Вы наблюдали самую яркую за последние 7 лет комету C/2020 F3 (NEOWISE), обнаруженную 27 марта 2020 года на снимках космического инфракрасного телескопа WISE. Весной комета была неразличима невооруженным глазом. 3 июля она прошла точку перигелия (максимально близкое положение к Солнцу), на некоторое время становясь очень яркой и хорошо различимой на небе невооруженным глазом. Постепенно, удаляясь от Солнца, она будет достаточно быстро терять свою яркость и уже в начале августа станет различима только при помощи оптических приборов. Больше интересной и полезной информации о комете можно почитать здесь: vk.com/nebonsk?w=wall-29943951_13814%2Fall

Саитов Николай Викторович (Щучье)

Вопрос: Здравствуйте, ночью с 16 на 17 июля примерно в 1:20 наблюдал непонятный объект, вроде бы звезда, но почему то с хвостом как метеорит и мерцает как будто горит, что это может быть?

Ответ: Здравствуйте! Как и многие внимательные наблюдатели, Вы видели комету C/2020 F3 (NEOWISE), на тот момент хорошо различимую на небе невооруженным глазом. К концу июля яркость кометы ослабеет и постепенно ее все сложнее будет находить на небе. Подробнее о комете рекомендуем почитать в статье: vk.com/nebonsk?w=wall-29943951_13814%2Fall

Гумаров Руслан Улиханович (Палласовка)

Вопрос: Что значит, если на небе видно звезду с хвостиком?

Ответ: Здравствуйте. «Звезда с хвостиком», продолжительное время наблюдаемая на звездном небе, является кометой. В данном случае — это обнаруженная в марте 2020 года комета C/2020 F3 (NEOWISE), которая в июле достигла максимума своей яркости и хорошо наблюдалась невооруженным глазом даже в черте города. Эта комета выступает самой яркой за последние семь лет, которая в момент пика своей яркости представляла очень красивое впечатляющее зрелище.

Ключников Владимир Владимирович (Калининск)

Вопрос: В ночь на 12.07.2020 18 градусов северо-восточной части виден объект, который падает с неба, а за ним тянется расходящийся хвост. Что это? Первый раз вижу!!!

Ответ: Здравствуйте. Наблюдаемый Вами объект — комета C/2020 F3 (NEOWISE), обнаруженная весной 2020 года и в июле набравшая свою максимальную яркость. Комета является самой яркой за последние семь лет и продолжительное время легко была различима даже случайным взглядом на небе. Подробнее почитать о комете рекомендуем в статье: vk.com/nebonsk?w=wall-29943951_13814%2Fall

Татевосян Илья (Черновцы, Украина)

Вопрос: Здравствуйте! В ночь с 01.07 на 02.07 в небе очень ярко светилось что-то красное, так как рыбачил всю ночь? смотрел за её передвижением аж до утра, звезда (я предполагаю) сделала четверть круга с Юга по часовой стрелке до рассвета! Подскажите что это, я устал листать интернет, ничего не могу найти! Заранее спасибо.

Ответ: Здравствуйте. Вероятнее всего Вы наблюдали одну из планет Солнечно системы, которые сейчас хорошо видны в течение всей ночи. По характерному красноватому оттенку можно определить Марс, но он поднимается во второй половине ночи на востоке и пропадает на рассвете ближе к южному направлению. Гораздо ранее почти сразу с наступлением темноты на юго-востоке поднимается Юпитер, видимый яркой жирной точкой (самой крупной в этом направлении), которая может иметь легкий желтоватый оттенок, а следом за ним на некотором расстоянии слева поднимается Сатурн, выглядит слабее, но также хорошо различим на небе. Обе планеты видны всю ночь до рассвета, «проходят» для наблюдателя через южное направление. Для более точного определения наблюдаемого объекта рекомендуем воспользоваться симулятором звездного неба Stellarium (stellarium-web.org).

Сапрыгин Роман (Новосибирск)

Вопрос: Добрый день! Если я не ошибаюсь, сейчас можно наблюдать парад планет, очень редкое явление. Можно ли его увидеть в данной обсерватории? С телескопами я ранее никогда не работал.

Ответ: Здравствуйте, Роман. Да, с 4 июля и до середины августа действительно можно наблюдать, так называемый, «парад планет». Подробнее о нем можно почитать здесь: http://nebo-nsk.ru/news/3057. Наблюдать планеты в обсерватории можно только каждую по отдельности, так как угловое расстояние достаточно большое и время восхождения планет разное. Наблюдения в обсерватории Большого новосибирского планетария невозможны до возобновления деятельности учреждения с посетителями.

Дюкарева Татьяна (Новосибирк)

Вопрос: 7 мая в 23-00 наблюдала за Венерой. . Почему то видела только ее часть.. Не подскажите в какое время можно увидеть ее полным шаром?

Ответ: Венера является внутренней планетой, ее орбита находится внутри орбиты Земли. Это значит, что для земного наблюдателя Венера всегда находится в одном направлении с Солнцем, располагаясь то в стороне от него, то на одной линии с Солнцем и Землей. Кода Вы наблюдали Венеру 7 мая, она для земного наблюдателя была слева от Солнца (восточней) и, соответственно, была освещена только с одного бока, для наблюдателя отображаясь серпиком. 3 июня Венера догнала нашу планету и оказалась на одной линии между Солнцем и Землей (так называемое, нижнее соединение), но уже 28 мая Венера перестала наблюдаться с Земли, так как ее свет оказался перекрыт ярким солнечным светом. Такая комбинация планет повторится только спустя 588 суток. Однако двигаясь по своей орбите быстрей Земли Венера скоро окажется с другой стороны от Солнца (с западной) и с 22 июня мы будем наблюдать ее по утрам, но опять же в виде серпика, освещенного со стороны, обращенной к Солнцу. Чтобы увидеть полный диск Венеры придется подождать несколько месяцев, когда Венера окажется относительно нашей планеты «за Солнцем». В этом случае Солнце для земного наблюдателя осветит большую часть этой планеты. Такие условия сложатся к концу декабря, когда на предрассветном небе можно будет в телескоп наблюдать почти весь диск этой планеты.

Анатолий Третьяков (Краснодар)

Вопрос: Здравствуйте. Скажите, пожалуйста, если черная дыра искривляет пространство-время, то можно ли теоретически путем его «выправления» высвободить из дыры материю? Спасибо.

Ответ: Здравствуйте. «Выпрямить» область пространства-времени, значит уменьшить массу Черной дыры. Если бы это удалось, то мы остановили бы (или замедлили) процесс падения материи в Черную дыру, но не вернули бы содержимого. Черная дыра – не яма, в которой накапливаются упавшие предметы. Если в Черную дыру что-то попадает, то находится оно уже совсем в другом, неизвестном пока виде. Оказавшись в «плену» Черной дыры, материя становится совсем не похожа на ту, которую мы знали до ее «падения». Следовательно возникают уже другие вопросы: в каком варианте предстанет эта материя и что с ней делать? А на эти вопросы, как бы они ни были интересны, у современной науки достоверных ответов пока нет.

Мазурова Ольга (Нижний Новгород)

Вопрос: Земля вращается вокруг своей оси. Так? Так. Звёзды вместе с Землёй не вращаются (понимаю, что звучит тупо, но это важно). Так? Так. Свет от любой звезды доходит до нас за там миллионы лет. А теперь вопрос: а как же так получается, что мы видим одни и те же звёзды в одно и то же время в одном и том же месте?!

Ответ: Земля вращается вокруг оси и обращается вокруг Солнца. Значит, всякий раз, когда наша планета оказывается в одних и тех же точках своей орбиты, мы будем в небе наблюдать те же звезды, в том же расположении, что и год назад в это же время суток. Звезды, которые мы видим невооруженным глазом, на самом деле удалены от нас не дальше, чем на 1,5 тысячи световых лет. Это совсем рядом. Подавляющее большинство звезд из наблюдаемых будут светить еще не один миллиард лет и имеют такое малозаметное для нас собственное движение, что увидеть изменения очертания созвездий можно только через несколько тысяч лет. Это значит, что всю свою жизнь мы «обречены» видеть одни и те же звезды и созвездия.

Мария (Мариуполь)

Вопрос: Здравствуйте, сегодня вечером на небе увидела яркую красно-жёлтую полосу. Чем-то она напоминала полосу от самолёта, но тоньше. Выглядело как шлейф от чего-то, но сам объект видно не было, возможно он был тёмный и слился с ночным небом. Так, оно продолжало двигаться вверх, а после чего, начиная с хвоста (если это можно так назвать), начало исчезать. Очень интересно узнать, что это было.

Ответ: Здравствуйте. Красно-желтый цвет объекта в вечернем небе означает, что он находится высоко и подсвечивается Солнцем, которое для наблюдателя уже зашло. С большей вероятностью, это был так называемый инверсионный след, оставленный реактивным самолетом. Если бы это было облако (по форме – это не облако) оно начало бы исчезать, как вы заметили, не с одного края, а по всей протяженности одновременно. Иногда, следы после пролета реактивных самолетов в вечернем небе выглядят эффектно и даже таинственно.

Чумачков Даниил (Ставрополь)

Вопрос: 27-28 апреля 2020 года немного левее Венеры я наблюдаю какие-то полосы, наблюдаю через телескоп VEBER 350×60. Полосы небольшие, наблюдаю их в одном и том же месте . Что это может быть?

Ответ: Скорее всего, это постороннее явление, которое не относится к наблюдаемому объекту. Многое зависит от состояния атмосферы, качества и состояния оптики. В Вашем случае, вероятно, свет от постороннего источника попал в телескоп и создал такую картину. 27 и 28 апреля 2020 года, восточнее Венеры («слева» как Вы описываете), находилась Луна. Возможно ее свет и создал блик, который Вы видели, наблюдая Венеру. Проверьте также чистоту оптических элементов телескопа, пыль и отпечатки пальцев также могут влиять на качество изображения.

Липай Анастасия (Псков)

Вопрос: Поздно вечером посмотрела на небо и заметила объект похожий на спутник, как только он скрылся за облаком чуть выше по той же траектории появился другой, и так раз 10 и каждый из них был выше предыдущего.

Ответ: Анастасия, Вы действительно наблюдали искусственные спутники, которых сейчас очень много на околоземной орбите. По Вашему описанию, вероятнее всего, это был пролет группы спутников Starlink, ранее запущенных компанией SpaceX. Их траектории схожи и для наблюдателя они движутся друг за другом на некотором расстоянии. Определить какую именно группу спутников Starlink Вы наблюдали (с начала 2020 года было осуществлено 5 запусков) можно на сайте www.heavens-above.com, указав географическое местоположение и дату наблюдения, где отразятся траектории пролетов разных групп спутников как прошедших, так и предстоящих.

Амбарцумян Эльмира Робертовна (Siauliai, Литва)

Вопрос: Здравствуйте! 9 мая 2020 г. в 23:30 я вышла на балкон и посмотрела на небо. В юго-восточной части неба почти в зените я увидела яркую неподвижно висящую звезду. Посмотрела в театральный бинокль и увидела, что это не одна звезда, а несколько, близко к друг другу выстроившиеся в ряд. Но первая из них светилась очень ярко и как бы испускала рассеянный яркий свет. Я стояла минут 30. Звезды никуда не двигались, висели неподвижно. Я ушла домой, вернулась через 10 минут. Звезды висели там же, но, неожиданно линия стала растягиваться, как будто плоский объект оказался диском, меняющим положение в пространстве и поворачивающийся к наблюдателю своей круглой стороной. При этом по краям диска стали видны огни разной интенсивности свечения. Но объект по-прежнему висел неподвижно. Пока писала этот текст, светящийся объект немного сместился к западу и изменил форму. Из диска превратился в яркую звезду, вокруг которой горели более слабые огни и всё это было в светлом ореоле. Что это может быть? Спутники Илона Маска 22 или 23 апреля я видела ночью с балкона. Было страшновато. А на следующий день мне позвонила невестка из Вильнюса и сказала, что это видели многие и делились увиденным в фейсбуке. Пришлось залезть в инет и всё разъяснилось. Но в этот раз ответа на вопрос что там такое на небе, я не нашла. Может вы скажете , что это может быть.

Ответ: Здравствуйте. Примерно в описываемом Вами направлении и времени наблюдения находится одна из ярчайших звезд Северного полушария — звезда Вега из созвездия Лиры. Вероятно, она и привлекла Ваше внимание. Если Вы для наблюдения использовали театральный бинокль, который обладает широким полем зрения, то в поле зрения могла попасть широкая двойная звезда (эпсилон Лиры). Это знаменитая Двойная Лиры. В бинокль наблюдается как широкая пара (две звездочки рядом), но каждая звезда в свою очередь – тоже состоит из двух компонентов, которые различимы уже в небольшой телескоп. Возможно Вега, двойная звезда и еще какие-нибудь звезды, попавшие в поле зрения бинокля «выстроились» для Вас в подобия объекта. По мере того, как небо темнело, в поле зрения стали появляться новые звездочки и «очертания» воображаемого объекта стали меняться. И все они двигались в западном направлении, как и должно быть в соответствии с суточным вращением нашей планеты. Так это или нет, можно проверить проведя повторные наблюдения в тоже самое время. Для того, чтобы получить точное представления о наблюдаемых объектах советуем пользоваться программой Stellarium, которую в общем доступе бесплатно можно скачать с сайта www.stellarium.org

Кочева Оксана Валерьевна (Новосибирск)

Вопрос: Здравствуйте, подскажите, пожалуйста, что было за небесное тело на небе в 1996-1997 годах? Его называли кометой, но точно не помню. Наблюдать это тело можно было в юго-западном направлении летом по направлению на Алтай и видно его было даже днём. На небе оно стояло неподвижно, в течении полугода, а потом я не помню. Началось обучение и я забыла о нем.

Ответ: Здравствуйте. С большой вероятностью Вы, как и все мы, наблюдали комету Хейла-Боппа. Это долгопериодическая комета, ставшая самой «наблюдаемой» кометой XX века, и одной из ярчайших за несколько последних десятилетий. Была видима невооружённым глазом рекордный срок – почти 18 месяцев. Иногда её называют «Большой кометой 1997 года».

Ширабокова Елена Александровна (Курск)

Вопрос: Скажите пожалуйста 24.05.2020. На небе я увидела очень яркий объект ярче полярной звезды, он очень медленно летел по небу в сторону юга через некоторое время он начал тускнеть и вскоре совсем потух. Что это могло быть?

Ответ: Здравствуйте. По заданному описанию можно с уверенностью сказать, что Вы наблюдали пролет Международной космической станции, которые с определенной периодичностью можно наблюдать в своем городе. 24 мая пролет станции проходил с 21.47 до 21.55 с западного на восточное-юго-восточное направление и яркость станции выла достаточно высокая. Стоит отметить, что в некоторые пролеты МКС предстает самым ярким объектом звёздного неба, превышая по яркости не только Полярную звезду, но и самые яркие звезды ночного неба и даже планету Венеру. Посмотреть предстоящие пролеты МКС для своего населенного пункта можно на сайте www.mks-online.ru 21.47 — 21.55

Алижан Ислам (Алматы)

Вопрос: Объект появляется примерно в 12 вечера и исчезает в 4 утра мигает, светится белым светом выглядит как обычная звезда. Рядом с ним похожий объект только меньше и мигает чуть реже.

Ответ: Скорее всего речь все-таки идет о каких-то ярких звездах ночного неба. Определить более точно объект наблюдения в соответствии с направлением наблюдения рекомендуем при помощи программы виртуального планетария «Stellarium», которая на сайте www.stellarium.org предоставляется бесплатно, а также есть online версия.

Анна Зайцева (Тбилиси)

Вопрос: Добрый день! Второй вечер наблюдаю на небе ярчайшую звезду, светит прям как свеча. Это Венера? И еще, как найти Юпитер невооружённым глазом? Спасибо заранее!

Ответ: Анна, здравствуйте. Если Вы наблюдаете эту звезду в западном направлении, то да, это Венера. Она появляется самой первой на вечернем небе в западном направлении и заходит за горизонт на северо-западе после 23 часов для Вашего местоположения. А Юпитер сейчас поднимается после 2 часов ночи в направлении юго-востока и также очень яркий и заметно крупнее среди остальных звезд. Кстати, следом за Юпитером немного левее движется и Сатурн, а после 3 часов ночи из-за горизонта поднимается Марс, заметно меньшего размера, но с характерным красноватым оттенком. Чтобы не ошибиться во всех ярких объектах рекомендуем воспользоваться программой Stellarium (в свободном доступе на сайте www.stellarium.org), где можно посмотреть расположение каждого объекта в соответствии со временем и спланировать свои наблюдения.

Орквасова Арианна (Нальчик)

Вопрос: Какое-то небесное тело находится на западной части Звёздного неба, появляется раньше всех вместе со звездой справа сверху. Хочу узнать их названия. Заранее спасибо

Ответ: Аринна, здравствуйте. Первым объектом, который можно сейчас наблюдать в западном направлении на вечернем небе, является планета Венера. Яркая звезда выше и правее Венеры — Капелла в созвездии Возничий.

Шагов Владимир (Могилёв)

Вопрос: Здравствуйте, ночью 19.04.20 на небе появился летающий объект, потом за ним плыли ещё такие самые объекты, они держали дистанцию, их было около 100, замыкающий объект, когда приблизился на половину неба очень сильно свернул светом и он как бы отстал от других потому что летел далеко от них, что это?

Ответ: Владимир, здравствуйте. Подобным образом сейчас выглядят пролеты искусственных спутников Starlink, запущенных в недавнее время, когда 60 объектов с одного запуска первое время следуют по схожей траектории, постепенно расходясь на свои орбиты, чем обусловлено «отставание» некоторых спутников. По указанной дате наблюдения в небе над Могилёвом проходила группа спутников Starlink-5, запущенная на орбиту 18 марта 2020 года, наблюдать пролеты которой можно будет до начала мая, при этом количество спутников постепенно будет уменьшаться. Для уточнения времени наблюдения рекомендуем воспользоваться ресурсом www.heavens-above.com Удачных наблюдений!

Эвита Черногузова (Латвия)

Вопрос: Здравствуйте. Второй раз уже наблюдаем что летит в одну колонну звезды, не меньше чем 100 штук (считали) Самолёти и спутники они мигают, но тут летят выглядят как звезды. Что это может быть?

Ответ: Эвита, здравствуйте. Как и наблюдатель из Могилёва и Курска, Вы видели пролет группы искусственных спутников Starlink-5, запущенных компанией SpaceX 18 марта 2020 года — очередного запуска с целью создания единой системы глобального доступа к интернету. Постепенно спутники будут расходиться по своим заданным орбитам и уже не будут выглядеть такой единой чередой. 22 апреля также на орбиту была выведена очередная группа спутников Starlink-6, которую над Латвией можно было наблюдать 23 и 24 апреля. Запуск следующих 60 спутников Starlink-7 запланирован на май 2020 года.

Вовк Денис (Караганда)

Вопрос: Здравствуйте. Хотел уточнить, что за звезда появилась, или уже была, но настолько яркой я ее заметил вчера, 16.04.2020 г. ночью около полуночи правее от ковша Большой Медведицы и недалеко от горизонта. Она выглядела как сильно пульсирующий разными цветами пучок с ответвлениями, похожими на взрыв сверхновой. В любительский бинокль приблизил и увидел белую сферу, на поверхности которой активно что происходило. Не подскажете, что именно это такое?

Ответ: Здравствуйте, Денис. Вероятней всего речь идет об одной из ярких звезд неба северного полушария — Капелле в созвездии Возничий. Она как раз располагается правее ковша Большой Медведицы и в указанное время находится относительно недалеко от горизонта. Капелла — шестая по яркости звезда ночного неба и третья по яркости среди звезд северного полушария. Она представляет собой кратную систему звезд, состоящую из двойной системы звезд-гигантов (Capella Аа и Аb) и ее спутника двойной системы красных карликов (Capella Ha и Hb). Ощущение некоего «движения» на «поверхности» объекта при наблюдении его в оптический прибор обычно вызвано атмосферными помехами нашей планеты, которое может быть дополнительно усиленно еще и восходящими потоками при наблюдении объекта у горизонта. Кстати, под Капеллой можно наблюдать еще более яркий объект — планету Венеру, которая светит ровным белым светом и при наблюдении ее даже в бинокль сейчас предстанем маленьким серпиком.

Necto (Сочи)

Вопрос: В небе появляются точки, которые быстро двигаются и мигают, через неск. секунд они исчезают.

Ответ: Чаще всего мигающие объекты на ночном небе — это обычные самолеты с их сигнальными навигационными огнями красного, зеленого и белого цветов. В зависимости от высоты, расположения и направления движения самолета, наблюдателю на Земле могут быть видны как все, так и только часть этих сигнальных огней. Есть вероятность, что так могут наблюдаться и какие-то из искусственных спутников на околоземной орбите. Однако, обычно они выглядят более тусклой, белой точкой с равномерным движением, при этом скорости перемещения разных спутников отличаются, и их движение может быть как спокойно-плавное, так и стремительно-быстрое. На сайте www. stuffin.space представлена визуализация траекторий движения всех основных групп спутников, а на сайте www.heavens-above.com можно посмотреть периоды пролетов различных спутников для своего географического положения.

Валентина Белкина (Амурская область)

Вопрос: Здравствуйте! Когда-то, когда мне было лет 18, я видела потрясающее явление на небе, всю жизнь его помню и хочу узнать, что это было. Расскажите пожалуйста. Это был примерно 2003 год, май или какой-то летний месяц, примерно 23-00, город Искитим, Новосибирская область. Тогда я любила смотреть на звезды, знала все созвездия , которые видела, и их главные звезды. Была очень крупная звезда желтоватого цвета на небе, сначала я, занятая разговором с подругами и глядя на нее отвлеченно, подумала, что это Арктур, но потом сообразила, что она находится не там, намного западнее. И она начала увеличиваться в размерах и вращаться, менять цвета и, во время вращения, удлиняться, как запятая, и превратилась в красивую большую спираль, которая вращалась, именно так выглядят на фотографиях галактики.

Ответ: Валентина, здравствуйте. По представленному описанию можно предположить, что Вы наблюдали пролет ракеты, запущенной с космодрома Байконур, который достаточно хорошо виден в Новосибирской области и всегда интересен для наблюдателя. По данным Роскосмоса с конца апреля по середину августа 2003 года было произведено несколько космических запусков. Пролет разных ракета-носителей отличается друг от друга, и для наблюдателя может выглядеть следующим образом. Поднимающееся с юго-запада облачко со звездочкой в центре (если это ракета-носитель «Протон») или конусообразное облачко в виде кометы, также со звездочкой в центре (если это ракета-носитель «Союз»). Во время работы второй ступени, облачко оставляет за собой большой длинный шлейф. Движение по небу происходит, примерно, в два раза быстрее самолета. Достигнув положения почти над югом, происходит быстрая вспышка, после чего из облачка или конуса вылетает облачко/конус поменьше — в этот момент произошло отделение второй ступени и начала работать третья. Оставшаяся часть начинает затухать, а отделившаяся начинает набирать яркость. Далее космический аппарат продолжает полет на восток, теряя яркость и уходя из виду. Менее чем через минуту после отделения на горизонте на юго-юго-востоке появляются падающие обломки в виде хорошо видимых ярко-желтый звездочек, 5-7 секунд падающие и затухающие. При пусках ракета-носителя «Протон» также можно увидеть сгорающие обломки второй ступени. В южном направлении в момент, совпадающий с отделением аппарата от третьей ступени, появляются разгорающиеся звездочки ярко желтого цвета, среди которых одна более яркая. Они быстро уходят по небу на юго-восток и затухают. Источник описания: astrodrome.ru/we/index.php?topic=97.0 Стоит также отметить, что в соответствии с географическим положением наблюдателя, траекторией пролета и ракурсом наблюдения видимая картинка естественным образом может несколько отличаться от описанной.

Проць Александр Александрович (Краснодар)

Вопрос: Здравствуйте, подскажите, пожалуйста, где-то посередине между юго-западом и западом видна звезда еще при свете солнца. Это самая яркая звезда или может быть созвездие на вечернем небе которое можно наблюдать не выезжая за город. 10 апреля 2020 ее было видно еще с 18:40 висит достаточно высоко, путь ее по небу выше солнечного, и уходит она за горизонт дальше на север чем заходит солнце. Свечение ярко синего цвета.

Ответ: Здравствуйте. По заданному описанию можно точно сказать, что Вы наблюдаете Венеру — вторую планету Солнечной системы, которая по своему блеску превышает самые яркие звезды ночного неба. Благодаря своей яркости Венера появляется первой «звездой» на еще светлом вечернем небе сразу после захода Солнца. Вечерняя видимость Венеры продлится примерно до середины мая, после чего во второй половине июня ее можно будет наблюдать уже в восточном направлении в предрассветное время.

Ливина Мария (Курск)

Вопрос: Здравствуйте. Каждый день на протяжении нескольких недель в одно и тоже время, с запада на восток двигается колонна звёзд, расстояние между объектами не одинаковое. И примерно в одном месте они исчезают. Что это может быть? Заранее спасибо.

Ответ: Здравствуйте, Мария. С наибольшей вероятностью Вы наблюдаете пролёты одной из групп спутников Starlink — системы глобального доступа к интернету. 18 марта 2020 года ракета-носителем Falcon 9 был осуществлен вывод на орбиту очередных 60 спутников Starlink-5. С мая 2019 года компанией SpaceX были осуществлены уже пять таких запусков, этот стал шестым. Сразу после запуска спутники для наблюдателя выглядят стройной чередой двигающихся друг за другом «звёзд»; в последствии спутники расходятся по своим заданным орбитам, некоторые из которых относительно совпадают и часть спутников могут продолжать»следовать» друг за другом. Проверить точнее пролёт какой из групп спутников Starlink Вы наблюдали в соответствии со временем наблюдения рекомендуем на сайте www.heavens-above.com. В разделе «Пролёты спутников Starlink определённого запуска» введите свое геоположение, расписание пролётов можно будет просмотреть на разные календарные даты. Например, на конец марта для Курска пролёты Starlink-5 приходились на более позднее ночное время, а вот в вечернее время можно было наблюдать объекты Starlink-4, запущенные 17 февраля 2020 года. Также на сайте доступно Динамичное 3D-отображение всех объектов из недавнего запуска спутников Starlink. Ближайший запуск новой группы спутников Starlink компанией SpaceX запланирован на 22 апреля 2020 года.

Любова Татьяна (Набережные Челны)

Вопрос: Здравствуйте, уже в течение 10 дней я наблюдаю очень яркую звезду в ночном небе в районе 21-23.00 на юго-западе. Прочла, что вероятнее всего это Венера. Успокоилась. Но вот загадка: звезда резко меняет свое месторасположение. Когда я начала за ней наблюдать, то первые 4 ночи она появлялась в одном и том же месте, постепенно смещаясь вправо. И , вдруг, на пятую ночь она значительно смещается к северу и так продолжается 2 дня, затем она вновь оказывается на прежнем месте — на юго-западе. Может ли такое быть? На ум приходит лишь одно объяснение — это разные звезды. НО мы живем в городе и со звездами у нас, мягко говоря, напряженка. Их попросту не видно. Вследствие чего я все же полагаю, что это один и тот же объект. Или я не права? И звезда ли это вообще? Может, это метеозонд или еще что-то в этом роде. Спасибо.

Ответ: Татьяна, здравствуйте. Вероятнее всего речь идёт о разных объектах. В одной области неба недалеко от Венеры сейчас можно наблюдать несколько ярких звёзд, хорошо видимых в черте города. Конечно, Венера является самым ярким объектом среди них и видна сразу после захода Солнца в западном направлении. На более темном небе выше Венеры можно наблюдать другую яркую звезду — Капеллу в созвездии Возничий. Левее Капеллы можно наблюдать две звезды созвездия Близнецы — в этой паре более яркий Поллукс и менее яркий Кастор. Ниже и левее Поллукса расположен Процион в созвездии Малый Пёс, а ниже него почти у горизонта в начале апреля можно было наблюдать ярчайшую звезду ночного неба — Сириус в созвездии Большого Пса. Также примерно посередине между Проционом и Венерой можно наблюдать другую яркую звезду — Бетельгейзе в созвездии Орион, которая имеет характерный желтоватый оттенок и в начале апреля была видна в юго-западном — западном направлении. В начале апреля, на момент Вашего обращения, в указанном юго-западном направлении более очевидным объектом мог оказаться как раз Процион, а вот Венера весь апрель наблюдается в западном направлении, которая к моменту своего захода за горизонт оказывается уже ближе к северо-западу. Для более точного определения объектов прошлых и будущих наблюдений рекомендуем воспользоваться программами виртуального планетария, например, удобной в пользовании «Stellarium», которая есть в свободном доступе на www.stellarium.org. Удачных наблюдений!

Фролова Валентина Сергеевна (Волгоград)

Вопрос: Здравствуйте. 12.03.20 возле Полярной звезды летели звёзды одна за одной ровно в ряд их было около 100 (может больше или меньше, точно не скажу) это было около 22:00 . Подскажите, пожалуйста, что это могло быть?

Ответ: Здравствуйте! Скорее всего Вами наблюдалась группа околоземных спутников Starlink-3, которые в количестве 60 штук двигаются по схожей орбите словно друг за другом. Данная группа спутников была запущены компанией SpaceX 29 января 2020 года и является уже не первой в системе данных спутников, которые выводятся на орбиту Земли с мая 2019 года с целью создания единой сети высокопроизводительного спутникового интернет-канала связи. Для знакомства с траекториями движения этих и иных искусственных спутников нашей планеты рекомендуем ресурс: www.heavens-above.com

Алексей (Новосибирск)

Вопрос: Здравствуйте, интересует звезда в южной части небосвода, очень яркая, или это комкта? Спасибо.

Ответ: Добрый день! В южной части весеннего небосвода Новосибирска Вы можете наблюдать Сириус в созвездии Большого Пса — ярчайшую звезду ночного неба. Совместно с другими яркими звёздами — Проционом в созвездие Малый Пес (расположенном выше и левее от Сириуса) и Бетельгейзе в созвездии Орион (расположен правее от Проциона) — Сириус образует, так называемый, зимний треугольник — хорошо различимый астеризм, лучше всего видимый в Северном полушарии в течение всей зимы, ранней весной, а также под утро в период осени.

Вася Пушкин

Вопрос: Какую планету или звезду я могу наблюдать с начала февраля этого года на западе или чуть севернее. Висит как полярная звезда, на одном месте, может быть изменяется высота.

Ответ: Здравствуйте! Возможно, речь идет о Венере — ярчайшем объекте вечернего неба, видимым сейчас как раз в юго-западном/западном направлении. Для более точного определения рекомендуем воспользоваться программой Stellarium, в которой можно посмотреть расположение всех объектов в разный промежуток времени в соответствии с географическим расположением наблюдателя: www.stellarium.org

Колесникова Ольга Николаевна (Ростов)

Вопрос: 2 марта 2020 в 5:00 выстроились 7 звёзд подряд, двигались чётко друг за другом. Что это может значить?

Ответ: Добрый день! Скорее всего Вами был отмечен пролет серии околоземных спутников Starlink, в частности: STARLINK-1122, STARLINK-1109, STARLINK-1126, STARLINK-1091, STARLINK-1082, STARLINK-1100, STARLINK-1083. Спутники достаточно яркие и действительно двигаются друг за другом по одной траектории. Проверить пролет их и других искусственных спутников Земли можно на сайте www.heavens-above.com (необходимо отметить свое местоположение и время наблюдения).

Козловская Елена Ивановна (Дубно)

Вопрос: Здравствуйте! У меня два вопроса. Может ли астероид или комета лишь приблизившись к планете вызвать землетрясения? Может ли комета или астероид приблизившись к какой-либо планете расколоться так, что одна её часть упадёт на планету, а другая улетит в космос, изменив свою орбиту и период обращения? Большое спасибо.

Ответ: Добрый день! Размеры и масса астероида или кометы не сопоставимы с планетными, а потому эти малые тела не способны вызвать землетрясение один лишь сближением с планетой. Раскол астероида или кометы при сближении с планетами возможен в том случае, когда тело оказывается в сфере действия тяготения этой планеты и может быть разорвано под действием гравитацией, а все образовавшиеся осколки тела смогут продолжить только дальнейшее падение на планету.

Никулин Алексей Владимирович (Новосибирск)

Вопрос: Здравствуйте. Как найти Уран на звездном небе Новосибирска сейчас?

Ответ: Алексей Владимирович, здравствуйте. Уран сейчас доступен для наблюдений в вечернее время в юго-западном направлении в созвездии Рыбы. Но никаких ярких объектов для ориентира рядом не находится, поэтому рекомендуем воспользоваться программой Stellarium для лучшей координации на звездном небе (www.stellarium.org). Также приглашаем на программы вечерних астрономических наблюдений, которые пройдут в Планетарии 19 и 31 января, где при благоприятных погодных условиях удастся понаблюдать и Уран.

Старук Ольга Васильевна

Вопрос: Добрый вечер! Подскажите, пожалуйста, какие две звезды (или планеты) видны очень ярко и очень низко на юго-западе ночью, остальные звезды гораздо выше. Если этой информации достаточно… Или где посмотреть посоветуете. Спасибо. С уважением, Ольга Васильевна

Ответ: Ольга Васильевна, достоверно ответить о каких звездах идет речь сложно, понятие «гораздо выше/ниже» достаточно размыто. Тем не менее, можно предположить, что если в указанном направлении эти звезды – одни из самых ярких на небосводе, то вероятней всего это Альтаир и Вега (в созвездии Орла и Лиры соответственно). Альтаир располагается левее и немного ниже Веги. В то же время заметно выше этих двух звезд можно отметить еще одну яркую звезду – Денеб в созвездии Лебедя. Все вместе эти звезды образуют так называемый «Летне-осенний треугольник», видимый в летний и осенний период вплоть до наступления зимы. Для более точного изучения звездного неба на любую дату, время и место рекомендуем воспользоваться программой Stellarium – это можно сделать бесплатно на сайте программы http://www.stellarium.org/ru. Желаем приятных наблюдений и интересных исследований!

Вячеслав Евгеньевич Тригбович

Вопрос: Здравствуйте! Подскажите, Плутон планета или нет и кто это решает? Буду ждать ответ.

Ответ: С момента открытия и до 2006 году Плутон считался девятой планетой Солнечной системы, но в конце XX – начале XXI века в области за орбитой Нептуна, так называемом поясом Койпера (был открыт в 1992 году) было обнаружено много новых малых космических объектов, схожих по параметрам с Плутоном. В 2006 году Международный астрономический союз впервые дает определение термину «планета». Одной из его характеристик – способность расчистить район своей орбиты от других объектов – Плутон не соответствует. В связи с этим Плутон причисляют к новой категории «карликовых планет», к которой также относятся Церера, Макемаке, Хаумеа, Эрида и Седна.

Анна Миронова (Новосибирск)

Вопрос: Хочу посмотреть суперлуние в большой телескоп 14 ноября 2016 года. Есть ли такая возможность в планетарии?

Ответ: Анна, добрый день! Суперлуние подразумевает под собой явление, когда Луна пребывает в фазе полнолуния и одновременно с этим, вследствие эллиптической орбиты Луны, находится на минимальном расстоянии с Землей. На самом деле, это астрономическое явление происходит достаточно часто. В этот же раз Луна достигнет минимального расстояние в 351 781 км в ночь с 14 на 15 ноября в 00.20 по-местному времени. Благодаря данному явлению с Земли можно видеть чуть более крупный размер лунного диска, чем обычно. Однако оптические наблюдения Луны в фазу полнолуния не особенно удобны и в этот день в Планетарии проводиться не будут.

Марченко Егор Александрович

Вопрос: Здравствуйте, я хочу задать такой вопрос: Я слышал что телескоп Kepler в 2015 году обнаружил планету очень похожую на землю Kepler-452 b, и я хочу узнать какие примерные шансы, что человек приспособится к этому месту. (И ещё, расскажите об этой планете Kepler-452 b) Заранее СПАСИБО!

Ответ: Действительно, 23 июля 2015 года американское космическое агентство NASA опубликовало информацию о планете Kepler-452b b, которая имеет схожие общие характеристики с Землей. Обнаружена она была космическим телескопом Kepler транзитным методом — фиксированием изменения яркости ее материнской звезды, вызванного прохождением перед ней планеты. Наблюдения за Kepler-452 b велись с мая 2009 года по июль 2014, после чего ученые обрабатывали полученные данные различными способами. Планета Kepler-452 b вращается вокруг похожей на Солнце звезды в созвездии Лебедь, расположенной на расстоянии более 1400 световых лет от Солнечной системы (примерно 13,2 квадрильона км). Период обращения планеты вокруг звезды составляет около 385 земных суток. Kepler-452 b больше на 63 % и старше на 1,5 млрд лет, чем Земля, и также находится на орбите в зоне обитаемости своей звезды. Подробно исследовать физические характеристики этой экзопланеты не представляется сейчас возможным, тем не менее, согласно индексу подобия Земли, планета Kepler-452 b занимает сейчас шестое место в списке планет, похожих на Землю.

Васюкова Мария Романовна

Вопрос: Здравствуйте. У меня есть один вопрос: Почему метеориты, когда летят через атмосферу, стираются и становятся маленькими, а космические корабли нет?

Ответ: Здравствуйте! У космических кораблей (ракетоносителей, посадочных модулей) делается специальная оболочка из термостойких и тугоплавких материалов, которая как раз и предназначена для сохранения аппарата и защиты от горения при прохождении плотных слоев атмосферы.

Семенюк Яна Игоревна

Вопрос: Добрый день, будет ли 27.08.2016 видно «две луны»? Будет ли Марс очень близко проходить от Земли?

Ответ: Добрый день! Данная информация из года в год освещается в разных источниках, но отображает некорректные данные. Даже при максимальном приближении Марса к Земле, так называемом противостоянии, когда планета находится на небе в направлении, противоположном Солнцу, Марс никогда не сможет выглядеть на нашем небе так, как Луна. Подобные противостояния повторяются каждые 26 месяцев в разных точках орбиты Марса и Земли, а раз в 15-17 лет случается, так называемое, великое противостояние, когда Марс приближается на самое близкое расстояние к Земле – около 55-60 млн.км. В то время как расстояние от Земли до Луны составляет менее 400 тыс.км. Таким образом, даже при самом близком расположении Марса, он все равно будет виден яркой звездочкой, но никак не лунным диском. Дата же 27-28 августа всплывает каждый год в новостях после 2003 года, когда действительно было великое противостояние Марса. Следующее ближайшее противостояние Марса пройдет достаточно скоро — 27 июля 2018 года. Сейчас же во второй половине августа Марс располагается практически в самой южной части эклиптики и очень быстро заходит за горизонт, виден только в сумерках и очень непродолжительное время.

Орлов Виктор Семенович

Вопрос: Что за звезда на западе под углом 35-40 градусов? Звезда большая похожа на шар или это двойная звезда. Два месяца назад она была на юго-западе.

Ответ: Вероятней всего речь идет об Арктуре – самой яркой звезде в созвездии Волопас. Найти его можно легко – проведя мысленно взгляд влево и вниз с ручки ковша Большой Медведицы.

Лукина Татьяна Витальевна

Вопрос: Добрый день! 06.08.2016 около 23.00 в небе над Бердском мы с сыном увидели нечто потрясающее и в тоже время непонятное. По небу летел очень яркий поток звезд, очень большой в ширину и нескончаемый в длину. По времени это продолжалось около двух минут. Что это было я даже предположить не могу. Может комета, может еще что-то (прошу прощения за свою безграмотность в сфере астрономии). В интернете никакой информации не нашли. Будьте добры, подскажите, пожалуйста, что это могло быть. Уж очень завораживающее было зрелище. Спасибо.

Ответ: Татьяна, добрый день! Исходя из данного описания, скорее всего Вы стали свидетелем массового запуска воздушных шаров со светодиодами. Запущенные одновременно, они двигаются в одном воздушном потоке, а поднимаясь выше, очертания самих шаров быстро становятся не различимы в ночном небе, но светодиоды – маленькие яркие светящиеся точки — как будто летят сплошным плавным потоком, словно светящиеся бабочки высоко в небе. Это действительно красивое и завораживающее зрелище, но не имеет никакого отношения к астрономии.

Ефименко Андрей

Вопрос: Добрый день. Были у Вас на замечательной экскурсии. Нам во время ее демонстрировали стеклянную колбу внутри которой вращался вентилятор. Тем самым показывая действие солнечных лучей. Этот же предмет продается у Вас в магазине. Вопрос как называется этот прибор? Заранее спасибо за ответ.

Ответ: Добрый день! Данный прибор называется радиометр Крукса.

Бразгин Роман Сергеевич

Вопрос: Возможно ли увидеть Млечный путь в окрестностях Новосибирска в ясную погоду, в период июль-август, с 23 до 03 часов?

Ответ: Действительно, в указанный период в ясную ночь Млечный путь хорошо виден, но, конечно, только за пределами города. Наблюдать его можно в южной части неба.

Бачинский Алексей Васильевич (32 года, Норильск)

Вопрос: Здравствуйте. Увлекаюсь звездами сравнительно недавно. В силу удаленности своего постоянного места жительства (г.Норильск). Раньше не имел возможности приобщиться к столь прекрасному занятию. Хотелось бы поближе познакомиться с оборудованием, пообщаться со знающими людьми. И хотя бы одним глазом заглянуть в глубины космоса. Скажите пожалуйста, есть ли возможность посетить Вашу обсерваторию? В данный момент нахожусь в г.Новосибирске. С уважением, Алексей.

Ответ: Алексей, 12 августа в Большом новосибирском планетарии будет открыт сезон астрономических наблюдений. В рамках этого мероприятия будут работать сотрудники с разными телескопами на уличной площадке планетария, а также будут открыты наши обсерватории, где можно будет познакомиться с оборудованием и пообщаться со специалистами. В начале следующей недели появится полная информация о мероприятии. Следите за новостями на сайте!

***** (17 лет, Москва)

Вопрос: Совсем недавно захотелось узнать побольше о космосе, подскажите, пожалуйста, какую спец.литературу можно почитать ничего еще не знающему о космосе человеку?

Ответ: Интересной литературы очень много сейчас — о Солнечной системе, о физике космоса, о зарождении Вселенной — можно изучать разные теории и работы разных современных ученых. Но если мы говорим об астрономии, то порекомендуем Вам дистанционный курс МГУ «Основы астрономии», который доступен всем желающим на сайте: https://openedu.ru/course/msu/BASTRO/

Уваров Виктор Петрович (29 лет, Иркутск)

Вопрос: Летел спутник на небе ночью (белая точка), потом стал ярче, увеличиваясь в размерах, и показалось, что он стал приближаться к Земле; потом через 15 секунд вернулся в первоначальное положение и продолжил двигаться дальше, не меняя траекторию полёта. Хотелось бы узнать что это за явление?

Ответ: Вероятней всего, вы столкнулись с достаточно распространенным явлением вспышки «Иридиумов» — явление, вызываемое отражением солнечного света от гладких поверхностей антенн спутников системы связи «Иридиум». Время от времени одна из антенн спутника отражает солнечные лучи на поверхность Земли, что для земного наблюдателя это выглядит как плавное появление и последующее плавное исчезновение ярчайшей звезды. Явление продолжается менее 10 секунд и в моменты отражения солнечного света становится самым ярким звездообразным объектом на небосводе, видимым невооруженным глазом. Положение каждого космического аппарата известно с высокой точностью, что позволяет вычислять время появления таких бликов для любой точки планеты заранее. Вы можете посмотреть предстоящие вспышки Иридиумов на сайте www.heavens-above.com, предварительно указав свое местоположение и временной период наблюдений.

Вовченко Оксана Николаевна

Вопрос: Добрый вечер! Сегодня 25.05.16г. в небе над нашим городом мы наблюдаем особенно большую красного цвета луну, подскажите, пожалуйста, как это объясняется? Заранее благодарим за ответ!

Ответ: Небесные светила у горизонта всегда окрашиваются в красный цвет (например, Солнце на закате или восходе). Это объясняется тем, что у красного цвета лучшие условия для проникновения через атмосферу к наблюдателю. Луна – не исключение и у горизонта она тоже будет иметь красноватый оттенок.

Иванов Владимир Александрович

Вопрос: Здравствуйте, решил купить себе телескоп, магазины предлагают множество вариантов, на форумах предлагают другие, Celestron AstroMaster 130 EQ, Levenhuk Skyline 120×1000 EQ, SkyWatcher 13065EQ2, SkyWatcher 1149EQ2, Meade Polaris 114, либо Synta BK909EQ2 Можете ли вы подсказать какой из них лучше взять и стоит ли брать какой то из них, может есть какой то другой вариант и вы порекомендуете.

Ответ: Если выбирать телескоп не для детей, а для серьезных наблюдений, то следует ориентироваться на то, что у такого телескопа должны быть окуляры в металлической оправе диаметром 1,25 дюйма. Именно такими окулярами комплектуются телескопы, подходящие для любительских и профессиональных наблюдений. Также лучше, чтобы оптика была изготовлена из стекла, а не из пластика. С фирмой-изготовителем, ценой и конструкцией телескопа стоит определяться по своему усмотрению.

Лямкин Иван Николаевич

Вопрос: Здравствуйте. Сегодня, 17 июня 2016г. на ночном небе видно три ярких звезды, очень похожи на планеты. Вопрос — какие планеты сейчас можно наблюдать без телескопа? Спасибо.

Ответ: Здравствуйте! Действительно вечернее небо сейчас очень богато на яркие космические объекты. Невооруженным глазом можно увидеть такие планеты как Юпитер, Марс, Сатурн. С вечерних сумерек и до 00 часов в западной части неба хорошо виден Юпитер. Справа и чуть ниже Луны можно наблюдать Марс, а на чуть большем расстоянии с левой стороны от Луны до самого рассвета хорошо виден Сатурн.

Вычегжанин Сергей Петрович

Вопрос: Посетил Планетарий с внучкой. Очень понравилось. Но, даже она в 10 лет знает, что в центре Черной дыры напряженность гравитационного поля равна 0 из-за симметрии, а не бесконечности. Я ей рассчитал распределение гравитационного поля внутри Земли с учетом изменения плотности достаточно точно. Но как оказалось гравитационную постоянную в Законе всемирного тяготения до сих пор уточняют и ускорение свободного падения то ли расчетная то ли экспериментальная. Где можно узнать g на поверхности Земли поточнее?

Ответ: Ускорение свободного падения, как и гравитационная постоянная, экспериментальные величины. Современное значение гравитационной постоянной составляет 6,67384·10−11 м3·с−2·кг−1 с погрешностью около 0,01%. Для практических нужд это вполне приемлемая точность. Ускорение же свободного падения на поверхности Земли (g) варьируется от 9,780 м/с² на экваторе до 9,832 м/с² на полюсах. Стандартное («нормальное») значение составляет g = 9,80665 м/с² и было определено как «среднее» на всей Земле, примерно равное ускорению свободного падения на широте 45,5° на уровне моря.

Столяров Игорь Александрович

Вопрос: Здравствуйте!! Побывал у вас в Планетарии. Очень понравилось!! Очень хочется посмотреть в телескоп!! Когда будет возможность?

Ответ: Добрый день! Регулярные астрономические наблюдения в Планетарии проводятся с сентября по май. В летний период темные ночи наступают достаточно поздно и они очень коротки, поэтому сейчас наблюдений не проводим. Вы можете принять участие в наблюдениях, которые периодически организует в центре города движение «Тротуарная астрономия» (vk.com/nsk_astronomy). Обо всех астрономических наблюдениях, которые будут проводиться в Планетарии, информация обязательно будет размещаться на официальном сайте и группе ВКонтакте. Следите за новостями.

Ксения

Вопрос: Добрый день, мне интересно знать допускает ли ученое общество возможность того, что открыты не все соединения и элементы и что звезды и планеты в других галактиках могут состоять из абсолютно неизвестных нам элементов. А так же что скорость и направление удаления звезд друг от друга не хаотичны, а определяются силой гравитации, как например солнце вокруг солнца, затем галактики вокруг галактик с большей массой и так до уровня вселенных? Извините за глупый вопрос, но действительно интересно узнать.

Ответ: Ксения, с ответом на Ваш вопрос нам помог доктор физико-математических наук, заведующий отделом физики и эволюции звезд Института астрономии РАН Дмитрий Зигфридович Вибе: 1. Допускает ли ученое общество возможность того, что открыты не все соединения и элементы и что звезды и планеты в других галактиках могут состоять из абсолютно неизвестных нам элементов. «Давайте разделим вопрос на два. Сначала об элементах. Как известно, химические элементы отличаются друг от друга количеством протонов в ядре (оно ещё называется атомным номером). Поскольку количество протонов не может быть слишком большим, число элементов ограничено. Сейчас нам известны элементы с количеством протонов в ядре до 118; новые, пока не известные нам элементы могут иметь лишь большее количество протонов. Далее, нужно учитывать, что у элементов тяжелее урана нет долгоживущих изотопов. Это означает, что существование каких-либо объектов из (пока) неизвестных науке элементов невозможно. Даже если ядра таких элементов и возникают в результате каких-то процессов (например, в земных лабораториях), они распадаются быстрее, чем из них может сформироваться какое-либо тело. Теперь о соединениях. Соединения элементов, то есть различные химические вещества нам, безусловно, известны не все. Ежегодно астрономы открывают в космосе по несколько новых молекул. Чаще это вещества, известные нам по земной химии, но иногда встречаются и молекулы, которые на Земле не синтезировались. Однако они всегда состоят из известных нам химических элементов. Могут ли звёзды и планеты в других галактиках состоять не из химических элементов, не из протонов и нейтронов, а вообще из какого-то совершенно нам не известного вида вещества? Вряд ли. Наши наблюдения проникли сейчас на колоссальные расстояния от Земли, и везде в звёздах и планетах мы видим признаки наличия только тех веществ и химических элементов, которые известны нам по нашей планете и её ближайшим космическим окрестностям.» 2. А так же что скорость и направление удаления звезд друг от друга не хаотичны, а определяются силой гравитации, как например солнце вокруг солнца, затем галактики вокруг галактик с большей массой и так до уровня вселенных? «В Солнечной системе нам привычно видеть именно систематическое вращение тел друг вокруг друга под действием силы гравитации. Однако эта сила способна приводить и к более хаотическому движению. Так движутся, например, звёзды в звёздных скоплениях. Да и Солнечная система не свободна от хаоса, что выражается, например, во временами очень быстрой эволюции орбит астероидов и комет. Поэтому ничего удивительного в хаотическом движении нет. В любом случае, если бы во Вселенной присутствовала описанная в вопросе иерархия вращения, мы бы её, конечно, увидели.»

Факты о Марсе | Температура на Марсе | Гравитация на Марсе | Расстояние до Марса

Планета Марс известна человечеству с древнейших времен. На Марс было отправлено множество космических миссий, и, вероятно, именно на эту планету в скором будущем высадятся первые астронавты. В этой статье мы расскажем всё, что вам нужно знать о Марсе и ответим на самые популярные вопросы о Красной планете.

Содержание

  • Факты о Марсе
  • Какого размера Марс?
    • Размер Марса
    • Что больше: Марс или Земля?
  • Орбита и вращение Марса
    • Сколько длится год на Марсе?
    • Сколько длится день на Марсе?
    • Есть ли времена года на Марсе?
  • Как далеко находится Марс?
    • Чему равно расстояние от Марса до Солнца?
    • Чему равно расстояние от Земли до Марса?
    • Сколько лететь до Марса?
  • Миссии на Марс
  • Из чего состоит Марс?
    • Формирование Марса
    • Структура Марса
    • Поверхность Марса
    • Атмосфера Марса
  • Сколько спутников у Марса
  • Предстоящие события
    • 1 августа: соединение Марс-Уран
    • 19 августа: соединение Луна-Марс
  • Часто задаваемые вопросы
    • Какая сила тяжести на Марсе?
    • Почему Марс называют Красной планетой?
    • Какая температура на Марсе?
    • Сколько марсоходов на Марсе?
  • Интересные факты

Факты о Марсе

  • Тип: планета земной группы
  • Радиус: 3 396 км
  • Масса: 6,417 × 10^23 кг
  • Афелий: 249,2 млн км
  • Перигелий: 206,6 млн км
  • Среднее расстояние до Земли: 225 млн км
  • Температура поверхности: от −143 °C до 35 °C
  • Солнечные сутки: 24 ч 39 м 35 с
  • Звездные сутки: 24 ч 37 м 22 с
  • Год: 686,98 земных дней
  • Возраст: 4,503 млрд лет
  • Назван в честь: древнеримского бога войны

Какого размера Марс?

Марс – одна из самых маленьких планет Солнечной системы; меньше Марса по диаметру только Меркурий. Давайте измерим Красную планету и сравним ее с Землей.

Размер Марса

Диаметр Марса – 6 792 км; длина окружности планеты вокруг экватора составляет 21 326 км. Таким образом, двигаясь со скоростью около 100 км/ч, вы сможете совершить путешествие вдоль экватора Марса примерно за девять дней.

Что больше: Марс или Земля?

Диаметр Марса составляет примерно половину диаметра Земли (который равняется 12 742 км). Также Марс всего в два раза больше нашей Луны, диаметр которой – 3 474 км.

Орбита и вращение Марса

У каждой планеты Солнечной системы есть определенный орбитальный период (определяющий продолжительность года) и период вращения (определяющий продолжительность суток). Давайте посмотрим, с какой скоростью Марс движется вокруг Солнца и вращается вокруг своей оси.

Сколько длится год на Марсе?

Так как Марс находится дальше от Солнца, чем Земля, Красной планете требуется больше времени, чтобы совершить один оборот вокруг Солнца. Год на Марсе длится примерно 687 земных дней или 1,88 земного года.

Сколько длится день на Марсе?

Марс вращается вокруг своей оси примерно с той же скоростью, что и Земля. Из-за этого продолжительность суток на этих двух планетах почти одинаковая. Один марсианский день (называемый “сол”) длится 24 часа 39 минут, что всего на 39 минут дольше земных суток.

Есть ли времена года на Марсе?

Как вы, вероятно, знаете, смена времен года на планете происходит из-за наклона оси ее вращения. Наклон оси вращения Марса весьма схож с земным: Красная планета наклонена под углом 25,2°, а Земля – под углом 23,5°. По этой причине на Марсе есть четыре времени года: весна, лето, осень и зима. Однако каждый сезон на Марсе длится примерно в два раза дольше, чем на Земле. Так происходит из-за того, что Марсу требуется почти два земных года, чтобы совершить один оборот вокруг Солнца.

Как далеко находится Марс?

Марс – четвертая планета от Солнца, а также один из двух ближайших соседей Земли (еще один сосед – Венера).

Чему равно расстояние от Марса до Солнца?

Из-за высокой эксцентричности орбиты Марса расстояние от Красной планеты до Солнца существенно меняется в течение марсианского года. Ближайшая к Солнцу точка орбиты Марса находится на расстоянии 206,6 млн км, а наиболее удаленная точка – на расстоянии 249,2 млн км. В среднем Марс находится на расстоянии 228 млн км от нашей звезды, что равняется 1,5 астрономическим единицам.

Чему равно расстояние от Земли до Марса?

Расстояние между Марсом и нашей планетой постоянно меняется. Максимальное расстояние от Земли до Марса составляет 401 млн км; минимальное – 54,6 млн км. Однако настолько тесное сближение этих двух планет ни разу не происходило за всю историю человечества. В 2003 году Марс и Земля подошли друг к другу на самое близкое расстояние за последние 60 тысяч лет; оно составило 55,7 млн км.

Сколько лететь до Марса?

Продолжительность полета на Марс сильно зависит от времени вылета. Лучшее время для запуска космического корабля на Марс – примерно за три месяца до максимального сближения Красной планеты с Землей. Такое сближение наступает каждые два года незадолго до противостояния Марса. Согласно информации от НАСА, в среднем полет на Марс занимает около девяти месяцев.

Два самых быстрых полета на Марс совершили космические аппараты “Маринер-6” (пять месяцев) и “Маринер-7” (четыре месяца). Однако оба эти аппарата изучали Марс, пролетая мимо планеты, а значит им не нужно было замедляться при подлете к ней, как это делают орбитальные станции и спускаемые аппараты. Последний марсоход, совершивший посадку на Марс – Персеверанс – достиг Красной планеты за семь месяцев.

Миссии на Марс

Из-за относительной близости к Земле, Марс был объектом множества космических миссий. С 1960 года на Красную планету было отправлено около 50 миссий, при этом только половина из них оказались успешными. Мы перечислим самые значимые из них.

Аппарат НАСА “Маринер-9” достиг орбиты Марса в 1971 году, став первым космическим аппаратом, вышедшим на орбиту другой планеты. “Маринер-9” провел подробную съемку около 85% марсианской поверхности и отправил на Землю более 7 000 изображений.

Первым искусственным объектом, достигшим марсианской поверхности, был советский аппарат “Марс-2”, запущенный в тот же год, что и “Маринер-9”. К сожалению, скорость с которой межпланетная станция “Марс-2” подошла к Красной планете была слишком высока: произошел сбой посадочной системы и спускаемый аппарат разбился о поверхность планеты.

Первую успешную посадку на Марс совершил “Викинг-1”. Этот аппарат НАСА работал на планете с 1976 по 1982 годы и отправил на Землю более 57 000 снимков.

Первым планетоходом, достигшим поверхности другой планеты стал “Соджорнер”. Он прибыл на Марс в 1997 в рамках миссии НАСА “Пасфайндер”. “Соджорнер” работал на планете в течение 83 солов, проводя научные исследования и делая фотографии местности.

Марсоходы НАСА “Спирит” и “Оппортьюнити” прибыли на Красную планету в 2004 году. В их задачи входили изучение климатической истории Марса и поиск воды на планете. Изначально миссия должна была продлиться 90 дней. Однако оба марсохода проработали на Красной планете намного дольше: “Спирит” функционировал до 2010 года, а “Оппортьюнити” – до 2018 года.

В 2012 году марсоход НАСА “Кьюриосити” высадился в районе марсианского кратера Гейла. Марсоход изучал климат и геологию Марса и обнаружил, что на планете раньше были условия, пригодные для жизни микробов. По состоянию на 2021 год “Кьюриосити” работает на Марсе уже более 3 000 солов.

В 2021 году Китай успешно посадил свой первый космический аппарат на Марс в рамках миссии “Тяньвэнь-1”. Аппарат “Чжужун” стал первым марсоходом, не принадлежащим НАСА, которому удалось достичь Красной планеты.

Также в 2021 году марсоход НАСА “Персеверанс” совершил посадку на Красной планете. Помимо марсохода, в миссии принимает участие вертолет “Индженьюити”. 19 апреля 2021 года “Индженьюити” совершил первый контролируемый полет на другой планете.

Из чего состоит Марс?

Как и три другие планеты земной группы (Меркурий, Венера и Земля), Марс представляет собой каменистое небесное тело. Давайте рассмотрим подробнее физические особенности этой планеты.

Формирование Марса

Марс сформировался одновременно с остальными планетами Солнечной системы. Около 4,5 млрд лет назад гигантское облако межзвездного газа и пыли сколлапсировало под воздействием собственной гравитации, образовав протопланетный диск. Марс и другие “каменные” планеты сформировались во внутренней части этого диска, а газовые планеты-гиганты – во внешней.

Структура Марса

У Марса есть ядро, мантия и кора. Ядро Красной планеты состоит из железа, никеля и серы. Оно окружено силикатной мантией и корой, состоящей из железа, магния, алюминия, кальция и калия.

Поверхность Марса

Поверхность Марса в основном состоит из базальта. Преобладание оксида железа в марсианской почве придает ей характерный красный цвет.

Как и на Земле, на Красной планете есть долины, пустыни, горы и полярные шапки. На Марсе даже обнаружены дельты древних рек, наличие которых говорит о том, что когда-то на этой планете была вода.

Атмосфера Марса

Атмосфера Марса намного тоньше земной. Она на 95% состоит из углекислого газа, в то время как атмосфера нашей планеты богата азотом и кислородом. Таким образом, свободно дышать на Марсе земляне не смогут.

Однако в апреле 2021 года марсоходу “Персеверанс” удалось получить небольшое количество кислорода из марсианской атмосферы. В будущем эта технология, вероятно, позволит производить кислород для марсианских колонистов.

Сколько спутников у Марса

У Марса два спутника: Фобос и Деймос. Они были обнаружены в 1877 году американским астрономом Асафом Холлом. Спутники Марса – одни из самых маленьких спутников в Солнечной системе. Для сравнения, диаметр Луны более чем в 100 раз превышает диаметр Фобоса – большего из двух спутников. Как и наша Луна, Фобос и Деймос находятся в приливном захвате и всегда повернуты к планете одной и той же стороной.

Предстоящие события

1 августа: соединение Марс-Уран

1 августа 2022 года в 12:22 по московскому времени (09:22 GMT), Марс встретится с Ураном в созвездии Овна. Красная планета будет самой яркой из двух: ее видимый блеск составит 0,2. Тусклый зеленоватый Уран (видимый блеск 5,8) будет на расстоянии 1°22′ от Марса. Лучше всего наблюдать соединение в бинокль. Через объектив телескопа можно будет рассмотреть планеты только по отдельности, так как расстояние между ними будет слишком велико.

19 августа: соединение Луна-Марс

19 августа в 15:16 по московскому времени (12:16 GMT), Луна пройдет на расстоянии 2°41′ от Марса; оба объекта будут в созвездии Тельца. Лунный диск (видимый блеск -11,7) будет освещен на 42%, а видимый блеск Красной планеты составит 0,0. Наблюдайте соединение невооруженным глазом или в бинокль.

Часто задаваемые вопросы

Какая сила тяжести на Марсе?

Сила тяжести на Марсе на 62% меньше, чем на Земле. Это значит, что человек массой 80 кг будет весить всего 30 кг, оказавшись на Красной планете. Хотя благодаря такой низкой гравитации людям будет значительно легче ходить по поверхности Марса, чем по земной поверхности, у нее есть и негативные последствия: например, у астронавтов могут развиться мышечная атрофия и остеопороз.

Почему Марс называют Красной планетой?

Преобладающий цвет марсианской поверхности – красный. Это объясняется содержанием большого количества оксида железа (также известного как ржавчина) в марсианской почве.

Какая температура на Марсе?

В целом Марс – очень холодная планета. Средняя температура на Марсе составляет -62 °C. Однако по словам ученого Майкла Мишна, температура воздуха на Марсе будет ощущаться людьми иначе, чем на Земле. На Марсе очень мало водяного пара и молекул воздуха, поэтому температура в -70 °C будет ощущаться примерно как -34 °C. Чтобы лучше представить себе температурные условия на Красной планете, посмотрите эту инфографику от НАСА.

Сколько марсоходов на Марсе?

По состоянию на октябрь 2021 года на Красной планете находятся шесть марсоходов. Пять из них (“Соджорнер”, “Спирит”, “Оппортьюнити”, “Кьюриосити” и “Персеверанс”) принадлежат НАСА и один (“Чжужун”) – Китайскому национальному космическому управлению.

Интересные факты

  • На Марсе находится крупнейший вулкан во всей Солнечной системе – Олимп. Высота вулкана составляет 21 км, что в 2,5 раза больше высоты Эвереста.

  • Спутник Марса Фобос сближается с Красной планетой со скоростью около 2 см в год. Через 50 млн лет Фобос либо врежется в поверхность Марса, либо распадется на части и образует кольцо вокруг планеты.

  • Миллиарды лет назад Марс был очень похож на Землю. На Красной планете были океаны с жидкой водой, в которых, вероятно, существовали примитивные формы жизни. Постепенно Марс лишился большей части своей атмосферы под воздействием солнечного ветра, и поверхность планеты высохла.

  • В конце 19-го и начале 20-го веков многие астрономы наблюдали на поверхности Марса сеть неких каналов. Некоторые считали, что это ирригационные каналы, созданные инопланетной цивилизацией. Однако в итоге оказалось, что это была не более чем оптическая иллюзия.

Надеемся, что вы узнали что-то новое о Марсе из нашей статьи. Если статья вам понравилась, обязательно поделитесь ей с друзьями в соцсетях. Вы также можете посмотреть наш познавательный мультфильм о Красной планете. Желаем вам ясного неба и успешных наблюдений!

Ночь с 29 на 30 июля 2022. Стояние Юпитера. Знакомимся с околополярными созвездиями (часть 2) / Хабр

Вечером предыдущего дня случилось состоялось новолуние. И теперь уже, как говорят люди иногда, зародилась новая луна. Увидим ли мы её этим вечером?

Вряд ли.

Новая Луна очень тонка, близка к солнцу, но — главное — она бежит впереди него по нисходящей части эклиптики, а значит зайдет за горизонт вскоре после захода Солнца. В момент касания горизонта центром солнечного диска Луна будет всего на высоте 6 градусов — в той части неба, где атмосфера чаще всего запыленная после жаркого дня. И уже через несколько минут Луна зайдет за горизонт.

В нескольких градусах от Луны будет располагаться Меркурий, но и он, будучи гораздо более контрастным и заметным объектом, вряд ли доступен наблюдениям в северных широтах.

Но следующим вечером самые опытные наблюдатели уже смогут заметить тончайший серп молодой Луны. Если им повезет с погодой, разумеется.

В сумерках на юго-востоке восходит Сатурн — первая из планет, которую можно наблюдать предстоящей ночью. Условия для наблюдения Сатурна сейчас наилучшие — он виден всю ночь, достаточно ярок, и расположен в восточной части созвездия Козерога, где много заметных звезд, что придает наблюдению планеты эстетическое удовольствие и помогает заметить, как от ночи к ночи планета смещается относительно окружающих её звезд. Сейчас Сатурн перемещается в попятном направлении, ведь скоро его противостояние, а вблизи противостояний все планеты перемещаются попятно. Астрологи такое движение планет называют ретроградным.

Вскоре после появления на небе Сатурна восходит и Юпитер — ярчайшее светило ночного неба и крупнейшая из планет Солнечной системы. Сегодня Юпитер остановился в своем видимом движении среди звезд и, подобно Сатурну, начал двигаться вспять — с этого дня Юпитер тоже двигается попятно (или — ретроградно, как сказал бы астролог). Из этого следует, что противостояние Юпитера тоже случится довольно скоро.

Стояние

Явление иллюзорной остановки планеты среди звезд в астрономии называется «Стоянием». Различают «Стояние по прямому восхождению» и «Стояние по эклиптической долготе». Отличие в системах отсчета, которые используются для рассмотрения видимого движения планеты. В одном случае система отсчета связана с небесным экватором, в другом — с эклиптикой. Первой чаще пользуются астрономы, второй — астрологи. Но это неточно.

Период времени вблизи противостояний некоторой планеты (примерно месяц в обе стороны по оси времени от момента противостояния) — лучшая пора для её наблюдений. Во время или вблизи противостояния планета наиболее близка к Земле, наиболее ярка, имеет наибольший видимый размер (что позволяет в телескоп рассмотреть те или иные подробности на её поверхности), и планету можно наблюдать всю ночь — от заката до рассвета.

Юпитер находится в созвездии Кита — вблизи его границы с созвездием Рыб. Куда бы он не двинулся с этого места, он в любом случае попадет в созвездие Рыб. В конце ноября Юпитер вновь замрет — размаха петли которую он до этого описывал, немного не хватит, чтобы оказаться в созвездии Водолея, и с началом календарной зимы величайшая из планет вновь устремится по звёздной карте вперед.

Лучше всего посмотреть движение Юпитера в динамике. В следующей анимации показано его движение за полтора года — с начала 2022 по середину 2023.

Музыка в ролике: “Звёзды водной стихии”

Восточнее Юпитера уже довольно хорошо виден красноватый Марс — его не спутаешь ни с одной звездой. И хотя неподалеку можно заметить красно-оранжевый Альдебаран — ярчайшую звезду созвездия Тельца, Марс имеет более пронзительный оттенок.

Немного восточнее Марс расположен совсем незаметный глазом Уран — эта планета доступна для наблюдений в бинокли, трубы, телескопы, но детали на её поверхности доступны лишь пролетающим поблизости космическим станциям. Напомню, всего один земной робот-исследователь посетил эту планету, и благодаря этому космическому аппарату, именуемому “Вояджер-2” мы теперь знаем, как Уран выглядит в тех или иных подробностях. Но американское агентство NASA уже готовит новую миссию к Урану.

Предполагается, что миссия к Урану стартует на ракете Илона Маска Falcon Heavy в 2031 году — не скоро. И кто знает, быть может к тому времени появятся новые, гораздо более современные ракеты. Сейчас все меняется очень стремительно.

А мы переведем взгляд еще восточнее и полюбуемся на красивые рассеянные звездные скопления Плеяды и Гиады в созвездии Тельца. Если вы заметите Марс на ночном небе последних дней июля, то так же легко найдете Плеяды с Гиадами.

За час до восхода Солнца над северо-восточным горизонтом взойдет удивительно яркое светило. Люди уже не придают ему особого значения, думая, что это — самолет. Но для самолета это светило слишком медленное и почти не меняет своего положение, пока пристально смотришь на него в течении нескольких минут. Потом, конечно, оказывается, что необычной яркости “Утренняя звезда” чуть сместилась, поднялась над горизонтом…

Это — не звезда. Это — планета Венера. Сейчас она видна по утрам за час или даже чуть раньше до восхода Солнца.

Венера продолжает путь по созвездию Близнецов, постепенно подбираясь к его ярчайшим звездам — Кастору и Поллуксу, которые пока трудно заметить в лучах утренней зари. Вместе с этим Венера догоняет и Солнце, сближается с ним. Поэтому, в середине осени видимость Венеры прекратится.

Во второй половине ночи есть смысл встать таким образом, чтобы Сатурн и Юпитер оказались по правую руку. При этом ваш взор будет устремлен на Восток. Подняв глаза на среднюю высоту между зенитом и горизонтом вы направите взгляд в сторону созвездия Персея.

Постойте так несколько минут блуждая взглядом среди звезд, при этом не фиксируя внимание ни на одной из них. Наверняка минут за 10 вы увидите один или несколько метеоров потока Персеиды. Сейчас самое время считать падающие звезды и загадывать желания.

Активность метеорного потока Персеиды продлится до конца следующего месяца, а пик активности придется на ночи с 11 по 13 августа.

В прошлом обзоре я начал знакомить Вас, мои читатели, с околополярными созвездиями. Искусство чтения звездных узоров начинается именно с них, ведь он видны круглый год и никогда не заходят за горизонт.

Сегодня я продолжу этот мастер-класс, и расширю звездное пространство, которое для кого-то из Вас станет привычным и понятным.

В прошлый раз мы познакомились с Ковшом Большой Медведицы, научились находить с его помощью Полярную звезду, Кассиопею и ярчайшую звезду созвездия Возничий — Капеллу.

Кто пропустил прошлый урок, найдет его по ссылке:

  • Знакомимся с околополярными созвездиями (часть 1)

А мы продолжим это звездное путешествие, но все же вновь вернемся к Медведицам и Ковшам.

Как мы уже знаем, две крайние звезды Ковша Большой медведицы указывают на Полярную звезду. И если опустить от Полярной отвесную линию — вниз — к горизонту, мы найдем точку севера. Двигаясь в её направлении достаточно долго, мы сможем попасть на Северный полюс нашей планеты. Но сегодня такого в планах нет.

Кстати, летними вечерами на точку Севера неплохо ориентирует и звезда Капелла — она видна низко над горизонтом именно над северным направлением.

Обратите внимание на звезды между Ковшом и Капеллой — не слишком яркие, и кажется, что — хаотично разбросанные. На самом деле можно усмотреть в них некоторый порядок и даже символизм. Это — голова и передние лапы Большой медведицы.

Как можно заметить, Большая медведица — очень обширное созвездие. Оно занимает на небесной сфере 1280 квадратных градусах, но конечно сами по себе эти цифры для большинства людей ничего не говорят. Становится понятнее, если сказать, что это третье по величине созвездие неба. Больше него только Гидра и Дева. То и другое сейчас не очень хорошо видно — не сезон. А Медведица всегда над горизонтом, и только её задние лапы иногда пытаются за горизонт зацепиться.

Пространство между Ковшом и Капеллой — это звезды головы Большой Медведицы. Нише головы расположены передние лапы, а задние — под ковшом.

Лапы Медведицы очень легко опознаются, потому что все они — и передние, и задние — заканчиваются звездными вилками — когтями.

А теперь изучим другое направление — левее Ковша.

Если продлить ручку Ковша с легким изгибом, мы найдем яркую оранжевую звезду Арктур — альфу созвездия Волопас. Созвездие Волопаса располагается на Арктуром — 5 звезд средней яркости, и еще несколько вблизи Арктура и чуть ниже него. Некоторые наблюдатели сравнивают фигуру Волопаса с парашютом, на котором спускается к горизонту Арктур. Это оправданное сравнение, ведь с течением времени Арктур опустится и даже зайдет за горизонт — это вполне заходящая в наших широтах звезда.

Кроме того, Арктур одна из самых быстрых звезд. И здесь я уже имею в виду его собственное движение, а не суточные перемещения по небесной сфере — восходы, закаты… Нет, Арктур довольно быстро смещается на фоне других — более далеких или просто медленных звезд. И это первая звезда, у которой это движение было замечено еще в античную эпоху — греческими астрономами.

Но смещение это для человеческого глаза совсем небольшое. Арктур смещается на видимый размер Луны (полградуса) за 800 лет.

Между Арктуром и “Черпаком” Ковша можно заметить еще одну звезду средней яркости — Сердце Карла. Это альфа созвездия Гончих псов. Столь нелепое название звезда получила по следам политических событий, когда был казнен король англии Карл I. Более того, когда-то именем этого короля, потерпевшего поражение в гражданской войне внутри своей же страны, называлось все созвездие. Астрономы — тоже люди, и тоже иногда поддаются на соблазн прославиться, получить покровительство или просто заявить о себе не вполне научным образом.

К счастью, ныне созвездие носит название связанное с древнегреческим мифом о Нимфе Каллисто, превращенной в Медведицу, её сыне Арктуре и сопровождающей Арктура своре охотничьих псов. Но главная звезда созвездия так и осталась “при дворе короля Карла I”, и уже никто не помнит, как она называлась прежде.

Сердце Карла является красивой и легкой для наблюдений двойной звездой, двойственность которой заметна в подзорную трубу или в телескоп при самом небольшом увеличении.

В следующие наши встречи я продолжу рассказ о том, как находить в звездных узорах неба знакомые очертания созвездий.

Следите за новыми публикациями в телеграмм-канале “Вселенная и Человек”, чтобы не пропустить очередную историю.

Поддержать автора можно здесь

Колонизация экзопланет, Венеры, Марса: разговор биолога и астрофизика

Поверхнось планеты Kepler-1649 c, звезды Kepler-1649 и планеты Kepler-1649 b. Фантазия художника. nasa.govМихаил НикитинБорис Штерн

Борис Штерн продолжает беседу с Михаилом Никитиным, науч. сотр. отдела эволюционной биохимии НИИ физико-химической биологии имени Белозерского при МГУ, автором научно-популярной книги «Происхождение жизни». На сей раз речь пойдет о поиске и заселении планет, пригодных для жизни. Видеозапись интервью: [1].

— В предыдущем интервью [2] Михаил рассказал о гипотезе Геи. Наша Земля обладает многими свойствами живого организма, и цивилизация в принципе может стать репродуктивной системой Геи: мы можем ей помочь обзавестись потомством, заселив необитаемые планеты. Сегодня мы эту тему разовьем. Начнем с того, что я расскажу — очень коротко, тезисно — о технической и астрофизической стороне дела, и потом уже буду в основном задавать вопросы Михаилу, касающиеся химии и биологии.

Итак, может ли человек перемещаться в межзвездном пространстве и достичь других планет? Ответ: сам человек — нет, но отправить посылку с живыми эмбрионами — может. Это крайне тяжело, но возможно на расстоянии около 10–15 парсек (30–45 световых лет). Его можно преодолеть где-то за 3–4 тысячи лет. Если когда-нибудь освоят термоядерный синтез (дейтерий + гелий-3), срок может сократиться в два раза. Зонд будет лететь раза в три быстрее, то есть около одной тысячи лет. Вот такие у нас возможности. Здесь ограничения очень суровые, очень жесткие — их задает природа. И новых законов физики, которые помогут нам природу обмануть, судя по всему, не существует.

Какие существуют планеты в радиусе 10–15 парсек? Есть оценки на основе данных космического телескопа «Кеплер», который наловил тысячи экзопланет — у него самый богатый урожай. «Кеплер» не мог детектировать земли. Он мог детектировать только либо очень крупные планеты, либо планеты, сравнительно быстро обращающиеся вокруг центральной звезды, с периодом меньше земного года — скажем, за сто дней. Планету с размером порядка размера Земли на орбите порядка орбиты Земли он не видел. Но благодаря статистике «Кеплера» мы знаем много более короткопериодических планет, и если мы их проэкстраполируем, то можем сделать такой вывод: приблизительно у 15% или 20% звезд типа Солнца есть планеты, пригодные для жизни — они находятся на нужном расстоянии от светила, будучи размером примерно с Землю. Есть и другие оценки, еще более оптимистичные, вплоть до единицы: якобы у каждой звезды есть одна, может быть, даже в среднем полторы планеты в зоне обитаемости. Я больше верю первым оценкам: их делали очень грамотные люди. Если эти оценки верны, то земель, пригодных для обитания, в нашей Галактике миллиард. Ближайшая находится на расстоянии где-нибудь 15 световых лет, а в радиусе 30 световых лет их уже несколько штук, может быть десяток. В общем, есть куда лететь за тысячи лет.

Как найти такую планету — отдельный вопрос. Если у нас останется время, мы поговорим, можно ли, находясь на Земле, узнать, какие планеты пригодны для жизни. А сейчас давайте поговорим вот о чем. Допустим, я прав, все эти оценки верны и с помощью ядерной энергии нам удалось послать корабль и мягко посадить на нужную планету. Теперь вопрос к Михаилу. Эта планета не будет похожа на Землю, даже раннюю. Допустим, мы привезли туда жизнь. С чем эта жизнь столкнется? Сможет ли она там нагенерировать кислород? На этой планете наверняка полно еще не окисленного железа. Это нечто совсем иное, чем наша современная Земля. Как по-вашему?

— Начну с количества воды на планетах, это важная характеристика. У нас в Солнечной системе есть Земля, покрытая океаном в среднем толщиной километра четыре; есть Марс, где воды на пару порядков меньше, чем на Земле; есть Венера, на несколько порядков более сухая, чем Марс. Модели образования планетных систем показывают, что твердые, землеподобные планеты могут иметь, в зависимости от очень небольших случайных отличий начальных условий, очень разное содержание воды. Диапазон — от сухих, подобных Венере, до планет-океанов, покрытых слоем воды в сотни километров. Планеты-океаны мы непосредственно не наблюдаем, а с сухими планетами можем легко ориентироваться на Венеру и Марс. Представим не точную копию Марса, а планету сухую, как Марс, но размером ближе к Земле и с магнитным полем… Мы же можем такой оптимизм проявить, да, Борис?

— Конечно-конечно.

— Тогда тут есть с чем работать. На поверхности такой планеты при условии магнитного поля, защищающего от космических лучей, вполне могут выживать экстремальные микробы, включая цианобактерии, которые будут использовать углекислотную атмосферу для фотосинтеза и выделения кислорода. Одних микробов, конечно, не хватит, чтобы сделать такую сухую планету комфортной для человека. В случае с терраформированием Марса, боюсь, придется ронять на него ледяные кометные ядра, чтобы приблизить количество воды к земному. И если такая работа в Солнечной системе проще, чем постройка межзвездного корабля, то в другой звездной системе эта задача явно гораздо сложнее. То есть заселить экзопланетный аналог Марса микробами и создать там какую-то биосферу мы вполне сможем, но эта биосфера вряд ли сможет легко и быстро поддерживать жизнедеятельность животных.

— Секунду, у меня вопрос по ходу. Какой диапазон количества воды примерно допустим для бурного развития жизни? Скажем, на порядок меньше, чем на Земле, или на порядок больше, чем на Земле?

— Это сложный вопрос. Количество воды на планете влияет на огромное разнообразие процессов. Например, на планетах-океанах, где нет суши и вся поверхность покрыта водой, может возникнуть серьезная проблема для биосферы: там нет выветривания горных пород и многие минеральные вещества, прежде всего фосфор, который живым организмам на Земле строго необходим, не попадают в морскую воду. Жизнь в наших океанах зависит от смыва фосфора, железа, цинка и некоторых других элементов суши, потому что морское дно выветриванию не подвергается; наоборот, эти элементы постепенно с океанскими осадками откладываются на него. То есть на планете-океане биохимические циклы фосфора, железа и других важных для жизни элементов могут быть очень сильно заторможены. Может оказаться, что все они лежат в донных остатках в темных глубинах океана. А поверхность океана, освещаемая звездой, вокруг которой вращается планета, даже при попытке заселять ее микробами может оказаться пустыней, лишенной фосфора, железа, цинка и других микроэлементов. Технические решения тут тоже возможны, но сами микробы с этим не справятся. В реальных условиях на Земле испытано устройство под названием апвелл — большая пластиковая труба, открытая с обоих концов, плавающая в море, длиной так в километр или полтора. В верхнем конце у нее большой поплавок, работающий клапаном, который от волнения колеблется вверх-вниз и подсасывает глубинную воду вверх. Это приводит к обогащению поверхностной воды фосфором, азотом и микроэлементами и к более бурному развитию водорослей. Апвелл был испытан, по-моему, в Тихом океане, около Гавайских островов, как средство повышения рыбопродуктивности моря. С планетой-океаном, боюсь, без таких устройств ничего не получится, потому что там будут проблемы с удобрениями. А сухие планеты, как я уже сказал, можно заселить микробами, но продуктивность этой микробной биосферы будет слишком мала, и кислородной атмосферы на них мы будем дожидаться миллионы лет.

Ну, и самый оптимистичный вариант — если у нас есть экзопланета, которая похожа на Землю по содержанию воды, то есть там есть площадь моря и площадь суши примерно одного порядка. Если заселить туда неких оптимизированных цианобактерий, они могут довольно быстро выйти на весьма высокую производительность кислорода. И тут, кстати, мы можем посмотреть на Марс, потому что у Марса, несмотря на отсутствие жизни и кислородного фотосинтеза, поверхность довольно окисленая из-за отсутствия магнитного поля: углекислый газ и вода в его атмосфере разлагаются космическими лучами. При этом водород улетает в космос, угарный газ CO, очень устойчивый, остается, а выделяющийся кислород окисляет поверхностные минералы, поэтому на поверхности Марса железо и сера находятся в таком же окисленном состоянии, как и на поверхности Земли, хотя фотосинтетиков там не было. Если планета потеряла магнитное поле, прожарилась космической радиацией, поверхность уже окислена, геологическая активность уже затухла — вот тут может очень хорошо сыграть сочетание заселения микробами и создания искусственного магнитного поля. Насколько я понимаю, чтобы создать на Марсе адекватное магнитное поле, сравнимое с Землей, достаточно построить электростанцию на два гигаватта и проложить по экватору сверхпроводящий кабель.

— Даже медный сойдет, только толстый. Правда, придется очень долго ждать. Надо изрядно погонять эти два гигаватта, чтобы энергию магнитного поля просто накопить, но это возможно, да.

— Если нам нужно быстрое создать кислородную атмосферу, то может оказаться полезна как раз планета относительно сухая, без магнитного поля и с высоким уровнем радиации. Одними микробами мы, конечно, не обойдемся, тут понадобится астроинженерная деятельность по добавлению воды и созданию магнитного поля, но при таком сочетании кислородную атмосферу можно сделать за тысячу лет, если повезет. Если мы берем планеты, более похожие на древнюю Землю по наличию магнитного поля и неокисленного железа на поверхности, то там, конечно, история с кислородной атмосферой может сильно затянуться.

— Интересно. Но с другой стороны, кислородную атмосферу делали совсем древние бактерии. У нас же сейчас прошла мощная эволюция и есть генная инженерия. Не можем ли мы вывести каких-нибудь термоядерных цианобактерий или что-то в этом роде, которые сделают всё гораздо быстрее и настолько быстро будут выделять кислород, что железо не успеет окислиться?

— Всё будет ограничиваться доступной им энергией, количеством света от звезды на этой планете. Если это система красного карлика, то там, скорее всего, будут проблемы.

— Там другие проблемы. Красных карликов очень много, они близко, но там всё плохо.

— Если звезда типа Солнца или даже более яркая, то надежды больше, но даже в этом случае, боюсь, набор нужной концентрации кислорода может затянуться на миллионы лет. На Земле это продолжалось два миллиарда лет; можно ускорить на несколько порядков, но несколько порядков от двух миллиардов лет — это миллионы лет. Гораздо дольше, чем межзвездные перелеты.

— Вопрос по динамике. Если не ждать, когда всё железо будет окислено, а ускорить темп… Если сейчас на Землю, такую, как есть, бросить столько восстановленного железа, сколько его было на ранней Земле, биосфера справится или нет?

— Зависит от того, как его бросить, насколько оно мелкодисперсно и где оно будет лежать, в каком климате, условно говоря: в Антарктиде или на экваторе, в низине или высоко в горах. Потому что под прикрытием, скажем, антарктических или горных ледников оно будет защищено от контакта с атмосферой. Если оно будет лежать монолитным базальтом, то будет окисляться довольно медленно — десятки и сотни тысяч лет. Если его вывалить в пустыне, то оно будет окисляться быстро. А если его вывалить в океан, оно может удачно лечь в глубинные районы океана так, что кислород кончится только в глубинах океана, а на поверхности ничего особенно не изменится. Если удачно угадать с океанской циркуляцией, то можно свалить его в неперемешиваемой глубинной области. Условно говоря, метеоритная пыль, равномерно посыпающая поверхность Земли, — это худший вариант, потому что она будет окисляться очень быстро. И вот если вывалить на Землю метеоритную пыль в количестве, соответствующем доступному двухвалентному железу на континентах архея, то, боюсь, содержание кислорода в атмосфере у нас просядет в несколько раз, и большинство сложных животных этого не переживут. Останутся какие-нибудь черви, останутся микробы, останутся растения.

— Понял. Теперь скажите: чего можно ожидать на планете, скажем, похожей на Землю, которой четыре миллиарда лет и на которой не было жизни? Всё железо там лежит в готовом состоянии, близко к поверхности, или оно куда-то могло деться?

— Конечно, вопрос очень гадательный, но я попробую. Значит, планета возрастом четыре миллиарда лет с количеством и процентом железа, сравнимым с Землей, да? Все четыре миллиарда лет она сохраняет магнитное поле, защищена от космической радиации. Это означает, что ее поверхность окислиться, скорее всего, не успеет. Там будут лежать сульфиды, базальты и другие соединения двухвалентного железа.

— Ну, значит, это одна из серьезных трудностей для колонизации экзопланеты. Я, в общем, подозревал, но не знал, что это действительно так тяжело.

— Да, получается, что одни микробы без астроинженерных работ, скорее всего, не справятся.

— Понятно. Человека туда не пошлешь. Человека там можно только вырастить. А для того чтобы там вырастить человека, нужны кислород, нормальная биосфера и много чего еще. И это — одно из серьезных препятствий для осуществления всей затеи, правильно?

— Да. Послать человека, если лететь несколько тысяч лет, конечно, проблематично. Проблемы начнутся уже с целеполаганием такой экспедиции, потому что люди живут меньше 100 лет, и они не очень склонны будут ставить такие долгосрочные цели и тратить на них ресурсы. Мне кажется, всё сдвинется с мертвой точки после серьезного продления человеческой жизни хотя бы в десяток раз, чтобы длительность межзвездного перелета стала обозримым временем, хотя бы сравнимым с длиной человеческой жизни.

— Согласен. Это одна из проблем. Но здесь есть несколько путей решения. Не только продление жизни, но и изменение психологии человека, способность работать на далеких потомков, своеобразный новый уровень альтруизма.

— Это уже тема для отдельного интервью. На этом пути разложены такие замечательные грабли, что мало не покажется никому. В сообществах любых альтруистов, как известно науке о поведении животных, обязательно встречаются паразиты. И вводя новый уровень альтруизма, мы открываем гигантские новые возможности для паразитов.

— Да. Действительно, тема для отдельного интервью. Гигантские сроки перелета — это действительно одно из препятствий в плане мотивации для этого безумно сложного, безумно дорогого проекта. Но все-таки продолжаем верить, что это возможно. Дальше вопрос стратегии. Если придется создавать кислородную атмосферу сотни тысяч или миллион лет, значит, колонизацию экзопланеты нельзя провести за один прием. Допустим, люди сделали для будущих поколений подарок — послали корабль. Если речь идет о сотнях тысяч лет для генерации атмосферы, значит, единственный шанс — в будущем послать другой корабль, уже более продвинутый, который будет создавать биосферу, потребляющую кислород. И сразу тяжелейший вопрос: живут ли цивилизации такое время?

— Ну, мы не знаем. Нет примеров. Наша цивилизация еще столько не прожила.

— И не факт, что собирается прожить столько, к сожалению.

— Если мы начнем хотя бы с Марса, это повышает наши шансы как цивилизации прожить дольше.

— Абсолютно согласен. Я бы добавил: повышает в двух отношениях. Во-первых, будет запасная цивилизация, пускай маленькая. А второе — созидательная деятельность сама по себе повышает устойчивость цивилизации.

Да. И объединяет людей.

— Поэтому я абсолютно согласен: надо начинать с Марса. Но все-таки остается много вопросов. С железом мы уже выяснили. Есть серьезная проблема с созданием кислородной атмосферы.

Другая проблема: как выяснить, что это за планета? Ведь от этого будет зависеть стратегия ее колонизации. Зонд, конечно, может выяснить всё, но он будет лететь многие сотни лет. А что можно увидеть с Земли? Эта планета, скорее всего, будет не транзитной, что сильно затрудняет ее изучение. Транзитные планеты проходят по диску звезд, свет звезды просвечивает атмосферу, и легко увидеть там спектральные линии — это уже сделано для некоторых планет. Если планета, о которой мы говорим, не транзитная, а вероятность этого 99%, то у нас нет техники для изучения ее с Земли. Нужно строить что-то вроде космического интерферометра, который увидит эту планету прямым образом. Такие проекты были: европейский «Дарвин», американский TPF (Terrestrial Planet Finder). Все они были зарублены еще где-то в 2000-х годах, к сожалению. Так что сейчас мы очень-очень далеки от понимания того, куда лететь, как действовать. Это целая область науки, которая ждет своего часа. Но тем не менее не хотелось бы заканчивать на пессимистических прогнозах…

— Давайте сначала на Марсе потренируемся. До него хотя бы ближе.

— Даже на Луне можно построить постоянно обитаемую базу, а Марс можно частично терраформировать. Там, как я понимаю, углекислоты хватит, чтобы удвоить атмосферу, что совершенно недостаточно. Но есть еще карбонаты, которых больше.

— Вот с количеством карбонатов там не очень понятно. Видимо, большинство карбонатов присыпаны какой-то летучей пылью, поэтому не видны с орбиты, и без множества посадочных миссий оценить, сколько там карбонатов, трудно.

— Значит, беда Марса в том, что он слишком легкий. Беда и в отсутствии тектоники, и в отсутствии магнитного поля. В принципе может ли рассматриваться дикая идея увеличить массу Марса? Я думаю, нет. Есть и другие проблемы. Первая — это перхлораты. Говорят, что Марс отравлен, и в этой почве ничего расти не будет. Что по этому поводу можно сказать? Это существенное препятствие?

— Действительно, сразу сажать в марсианскую землю картошку, конечно, действительно нельзя из-за перхлоратов. Для высших растений перхлораты ядовиты, но есть микробы, которые не только растут в их присутствии, но даже питаются ими, используя перхлораты как источник энергии, разлагая их до безопасных хлоридов. Такие микробы обнаружены на Земле, например, в пустыне Атакама, где перхлораты в почве встречаются наряду с нитратами. Используя таких микробов или пересадив их гены в другие виды микробов, вполне можно сделать марсианскую почву пригодной для высших растений. Эта задача решаема.

— Но растениям нужен кислород. Наверное, их придется высаживать в закрытых помещениях при низком давлении?

— Высших поначалу — да. Цианобактерии можно высаживать в открытый грунт, и они будут производить кислород, после того как будет решена проблема с радиацией — видимо, путем создания искусственного магнитного поля.

— Я думаю, что для цианобактерий радиация там не такая существенная, в два-три раза выше, чем на МКС.

— Если так, тогда для них это небольшая радиация.

— Вода для них нужна наверняка. Ее тоже мало, правильно? Надо, чтобы климат стал теплее, и температура нужна, то есть надо нарастить атмосферу, увеличить парниковый эффект.

— Боюсь, тут без забрасывания комет не получится, потому что, хотя на Марсе есть запасы водяного и углекислотного льда в полярных шапках, их не настолько много, чтобы сделать достаточно плотную атмосферу. Вроде бы в толще марсианского грунта есть жидкие рассолы, но они размещаются на несколько метров под поверхностью, куда свет не проникает.

— Льда очень много — обыкновенного водяного льда. Гораздо меньше, чем на Земле, конечно, но много по сравнению с ледяными шапками.

— Много по сравнению с Венерой, да. На Земле есть интересные микробные сообщества в сухих местах Антарктиды, например в долине Мак-Мердо: эндолитные микробы, которые вбуравливаются в толщу камня на несколько миллиметров, и толща камня защищает их от суровых внешних условий. Они, конечно, растут очень медленно, но ухитряются жить в экстремальных условиях. Из того, что можно найти на Земле, долина Мак-Мердо по условиям ближе всего к Марсу. Там похожие температуры и почти так же мало воды, разве что атмосферное давление выше и кислород уже есть.

— Цианобактерии вырабатывают кислород, но они в нем не нуждаются. Однако высшие растения потребляют кислород. Какое минимальное количество кислорода нужно высшим растениям, чтобы они росли и усиливали фотосинтез?

— Из физиологии растений известно, что как минимум корневые системы, особенно у болотных растений, переносят практически полное отсутствие кислорода, потому что бывает, что вся почва пропитана водой, и воздух к корням не проникает. Надземные части растений, листья и побеги, дышат кислородом ночью, когда не могут получать энергию путем фотосинтеза, но сколько кислорода им для этого надо — тут, к сожалению, я не смог сразу найти нижнюю оценку. Но точно в несколько раз ниже, чем процент кислорода в земной атмосфере, то есть снижение содержания кислорода с 21% до 5% рост растений в экспериментах даже ускоряло, а не нарушало. Будет ли проблематичным снижение кислорода в десять, сто или тысячу раз? Здесь надо копать литературу, я таких экспериментов быстро найти не смог. Но если у нас будет налажена генетическая модификация, то мы, наверное, сможем и ночной обмен веществ листиков и побегов приблизить к тому, что свойственно корням болотных растений, то есть они смогут обходиться ночью без кислорода.

— Ну, замечательно. Тогда есть шанс, что в принципе на Марсе даже при слабой, малокислородной атмосфере можно будет выращивать растения в открытом грунте. Это важно и для колонизации экзопланет, потому что развитие биосферы на землеподобной планете, если этой биосферы до сих пор не было, если полно неокисленного железа, — это вопрос времени.

Пора сделать некоторое промежуточное резюме из того, что я узнал. Впечатление, что теоретически размножение живых планет с помощью разума возможно. Этот процесс очень долгий. Я считал, что он займет тысячи лет, но, похоже, на самом деле — сотни тысяч.

— Возможно, миллионы.

— Возможно, миллионы. Это сроки, на которые не замахивается даже научная фантастика, и нам тоже очень сложно делать прогнозы. При современном состоянии человеческой цивилизации это невозможно. Будем надеяться, что наш вид когда-нибудь эволюционирует до такой степени, что сможет заняться подобными вещами. Кажется, всё. Михаил, хотите что-нибудь еще добавить?

— Я бы обратил внимание на относительно маленькие небесные тела вроде Цереры. Человеку там жить, скорее всего, будет совсем некомфортно из-за низкой гравитации, но какие-то локальные обитаемые купола с кислородом, микробами и растениями там в принципе можно сделать. И люди могут прилетать вахтовым методом. Кроме Цереры, есть всевозможные ледяные спутники планет-гигантов. В экзопланетных системах, кстати, часто бывают газовые гиганты в обитаемой зоне, достаточно близко к звезде, и спутники как раз — очень интересная цель для исследования и колонизации. Мы, к сожалению, еще не знаем, что и как там устроено. И есть планеты-океаны. Я говорил о нехватке на них микроэлементов и о том, что надо строить апвеллы. Но это неточно. Поскольку мы ни одной планеты-океана реально не знаем, возможно, мы не учитываем какие-то важные факторы.

— Но на планетах-океанах в роли апвеллов может выступить конвекция из-за вулканической активности.

— Или мощная вулканическая активность, или мощные приливы, перемешивающие воду на всю стокилометровую глубину. Еще можно подумать о терраформировании планет, подобных Венере. Венера при всех ее недостатках большая, как Земля. Поэтому если сделать зонтик от Солнца, блокирующий избыточный свет, понижать температуру и добавлять воду из кометного льда, можно получить более пригодную для жизни планету, чем Марс, хотя начальное вложение, конечно, будет большое.

— Ну, там надо как-то связать углерод, иначе ничего не получится наверняка. СО2 куда девать?

— Если затенить планету от Солнца процентов на 80, то температура там упадет до пригодной для жизни, и если добавить воды, в такой высокой концентрации СО2 цианобактерии будут счастливы до безумия; при наличии жидкой воды там начнется водный цикл, выветривание базальтов, отложение карбонатов и даже без цианобактерий углекислый газ будет связываться в карбонаты, а цианобактерии будут связывать его и в углерод тоже. Сочетание этих двух процессов (плюс бактерии еще ускоряют выветривание базальтов) может создать на Венере вполне пристойную кислородную атмосферу. Собственно, согласно моделям происхождения Земли, у нее первые десятки миллионов лет атмосфера по плотности была близка к современной венерианской: там тоже было очень много углекислого газа. Просто теперь он почти весь связан в карбонатных осадках. Карбонатных осадков, которые известны на Земле, хватит на три атмосферы Венеры, если их все разложить. То есть с Венерой, если мы начнем с астроинженерных действий, с зонтика, шансы на успех есть, и результат может оказаться даже лучше, чем с Марсом.

— Интересно. Да, я плохо себе представляю такой зонтик, потому что, скажем, альбедо в атмосфере повышать, наверное, не очень разумно — у нее и так высокое альбедо.

— Она и так довольно много отражает облаками.

— Да. И как сделать еще сильнее — я не знаю. Зонтик планетарных размеров, конечно, нам не под силу.

— Я вот не знаю, что сложнее: зонтик планетарных размеров или межзвездный корабль?

— Я тоже не знаю.

— Одного уровня сложность.

— Может быть.

— Еще я хотел сказать про задачи для космической техники, промежуточные по сложности между полетами на Марс и межзвездными полетами. Есть два очень интересных направления, и я не слышал, чтобы кто-то этим занимался.

Во-первых, ловля межзвездных астероидов и комет и взятие проб с них, которое позволяет узнать хотя бы химический состав других экзопланетных систем. Не уверен, что можно точно узнать, из какой системы прилетает тот или иной конкретный экзоастероид или экзокомета, но хотя бы усредненный состав комет в галактике мы таким способом изучать можем. Но как поймать межзвездную комету? Она пролетает через Солнечную систему обычно под большим углом к плоскости эклиптики, мы обнаруживаем ее буквально за несколько месяцев до перигелия. То есть нужен корабль с термоядерным двигателем, который быстро разогняется на сотни километров в секунду, может дежурить на околосолнечной орбите, перпендикулярно эклиптике, чтобы не тратить энергию хотя бы на поворот плоскости орбиты; но по какой траектории будет подлетать эта комета или межзвездный астероид, заранее не понятно, поэтому маневры потребуются большие и быстрые.

А второе — это телескопы. Есть такая точка — гравитационный фокус Солнца; вы, наверное, знаете это лучше меня. Она расположена довольно далеко за орбитой Плутона. Находясь в этой точке, можно использовать гравитационную фокусировку света Солнцем как линзу телескопа совершенно чудовищного размера. Если туда долетать, то получать прямые изображения экзопланет будет довольно легко.

Космический телескоп «Кеплер». Фантазия художника. NASA/Wendy Stenzel

— Проблема в том, что этот фокус — на расстоянии 550 астрономических единиц. Туда надо посылать целую армаду телескопов, то есть надо строить изображение объемное. Один телескоп увидит только размазанную дугу. Туда надо посылать целую армаду телескопов и строить объемное изображение. Это чудовищно сложная вещь.

— У меня к вам вопрос насчет межзвездных астероидов. В Сети упорно утверждают, что первый межзвездный астероид Омуамуа на самом деле был кораблем пришельцев (потому что он якобы маневрировал, как астероиды не могут), а власти это скрывают…

— Я тоже про это слышал, но деталей не знаю. Знаю только, что он был очень вытянутым, это определили по перемене блеска. Кстати, чтобы выяснить состав астероида, необязательно его догонять. Можно просто выстрелить наперерез и снять спектр того, что вылетело.

— А если это действительно окажется корабль внеземной цивилизации? Нехорошо получится!

— Получится нехорошо, но что же делать. Нечего летать без предупреждения. Кстати, что касается средств связи, — здесь вообще никаких проблем нет, любой радиосигнал, даже изотропный, с такого корабля легко ловится.

— Это понятно. Есть еще вариант, что корабль давно мертв и вышел из строя; выстрелив в него, мы его поломаем и получим меньше информации, но хотя бы не поссоримся с его хозяевами.

— А иначе мы вообще его упустим. Кстати, то, что это мертвый корабль, можно понять по спектру: там будет совершенно необычный химический состав.

— Да, был же замечательный прикол с околоземным астероидом, в спектре которого преобладал диоксид титана. Потом посчитали траекторию — оказалось, что это верхняя ступень одной из ракет «Сатурн-5», отправлявших астронавтов на Луну, которая просто вышла на гелиоцентрическую орбиту и через десятки лет вернулась.

— Такой веселый конец на фоне грустных размышлений о перспективах. Надеюсь, он немного скрасит общее впечатление. Мы прощаемся, но, конечно, будем продолжать дискуссии на эту тему. Большое спасибо!

1. youtu.be/watch?v=7u0TRCmMbRI

2. trv-science.ru/2022/05/nikitin-abiogenesis/

Если вы нашли ошибку, пожалуйста, выделите фрагмент текста и нажмите Ctrl+Enter.

См. также:

Марс — Исследование Солнечной системы НАСА

Марс — четвертая планета от Солнца — пыльный, холодный, пустынный мир с очень тонкой атмосферой. Марс также является динамичной планетой со сменой времен года, полярными ледяными шапками, каньонами, потухшими вулканами и свидетельствами того, что в прошлом он был еще более активным.

Марс — одно из наиболее изученных тел в нашей Солнечной системе, и это единственная планета, на которую мы отправили марсоходы, чтобы бродить по инопланетному ландшафту.

НАСА в настоящее время имеет два вездехода (Curiosity и Perseverance), один посадочный модуль (InSight) и один вертолет (Ingenuity), которые исследуют поверхность Марса.

Марсоход Perseverance — самый большой и самый совершенный вездеход, который НАСА отправило в другой мир — приземлился на Марсе 18 февраля 2021 года после 203-дневного путешествия, преодолев 293 миллиона миль (472 миллиона километров). Вертолет Ingenuity летел на Марс, прикрепленный к животу Perseverance.

Perseverance — один из трех космических кораблей, прибывших к Марсу в 2021 году. Орбитальный аппарат «Надежда» из Объединенных Арабских Эмиратов прибыл 9 февраля 2021 года. Китайская миссия Tianwen-1 прибыла 10 февраля 2021 года и включает в себя орбитальный аппарат, посадочный модуль и марсоход. У Европы и Индии также есть космические аппараты, изучающие Марс с орбиты.

В мае 2021 года Китай стал второй страной, успешно приземлившейся на Марсе, когда его марсоход Zhurong приземлился.

Международный флот из восьми орбитальных аппаратов изучает Красную планету с высоты, включая три орбитальных аппарата НАСА: 2001 Mars Odyssey, Mars Reconnaissance Orbiter и MAVEN.

Эти роботы-исследователи нашли множество свидетельств того, что Марс был намного влажнее и теплее, с более плотной атмосферой миллиарды лет назад.

Иди дальше. Подробное исследование Марса ›

Десять вещей, которые нужно знать о Марсе

10 вещей, которые нужно знать о Марсе

1

Маленькая планета

Если бы Солнце было высотой с типичную входную дверь, Земля была бы размером с десятицентовую монету, а Марс был бы размером с таблетку аспирина.

2

Четвертая скала

Марс вращается вокруг нашего Солнца, звезды. Марс — четвертая планета от Солнца на среднем расстоянии около 228 миллионов км (142 миллиона миль) или 1,52 а.е.

3

Более длинные дни

Один день на Марсе длится чуть больше 24 часов. Марс совершает полный оборот вокруг Солнца (год по марсианскому времени) за 687 земных суток.

4

Пересеченная местность

Марс — каменистая планета. Его твердая поверхность была изменена вулканами, ударами, ветрами, движением земной коры и химическими реакциями.

5

Принесите скафандр

Марс имеет тонкую атмосферу, состоящую в основном из двуокиси углерода (CO2), аргона (Ar), азота (N2) и небольшого количества кислорода и водяного пара.

6

Две луны

У Марса есть два спутника Фобос и Деймос.

7

без колец

Вокруг Марса нет колец.

8

Множество миссий

Эту планету посетило несколько миссий, от облетов и орбитальных аппаратов до марсоходов на поверхности. Первой успешной миссией на Марс стал облет Маринера-4 в 1965 году.

9

Тяжелое место для жизни

В настоящее время поверхность Марса не может поддерживать жизнь в том виде, в каком мы ее знаем. Текущие миссии определяют прошлый и будущий потенциал Марса для жизни.

10

Ржавая планета

Марс известен как Красная планета, потому что минералы железа в марсианской почве окисляются или ржавеют, в результате чего почва и атмосфера становятся красными.

Поп-культура

Поп-культура

Ни одна другая планета не захватила наше коллективное воображение так, как Марс.

Этот вид следов марсохода на Марсе является частью бесплатного набора постеров. Предоставлено: НАСА / Мур Бек | › Полноразмерный постер и варианты загрузки

В конце 1800-х годов, когда люди впервые заметили на поверхности Марса похожие на каналы детали, многие предположили, что там обитает разумный инопланетный вид. Это привело к многочисленным историям о марсианах, некоторые из которых вторгаются на Землю, как в радиопостановке 1938 года «Война миров ». Согласно устойчивой городской легенде, многие слушатели полагали, что эта история является реальным освещением в новостях вторжения, вызвавшего всеобщую панику.

С тех пор на Марсе происходило бессчетное количество историй, в которых исследовались возможности его марсианских обитателей. Похожие на фильм «Вспомнить все» (1990 и 2012) переносят нас на терраформированный Марс и выживающую колонию, где заканчивается воздух. Марсианская колония и Земля имеют натянутые отношения в телесериалах и романах «Пространство » .

А в романе 2014 года и его экранизации 2015 года «Марсианин» ботаник Марк Уотни застрял в одиночестве на планете и изо всех сил пытается выжить, пока спасательная миссия не сможет его найти.

Подходит для детей Марс

Марс для детей

Марс — холодный пустынный мир. Это половина размера Земли. Марс иногда называют Красной планетой. Он красный из-за ржавого железа в земле.

Как и на Земле, на Марсе есть времена года, полярные ледяные шапки, вулканы, каньоны и погода. У него очень тонкая атмосфера, состоящая из углекислого газа, азота и аргона.

На Марсе есть следы древних наводнений, но теперь вода в основном существует в ледяной грязи и тонких облаках. На некоторых склонах марсианских холмов есть свидетельства наличия в земле жидкой соленой воды.

Посетите NASA SpacePlace, чтобы узнать больше интересных для детей фактов.

NASA Space Place: Все о Марсе ›

Подробнее

Подробнее
  • Программа исследования Марса НАСА
  • НАСА: от Луны до Марса

Подробнее | Марс – Исследование Солнечной системы НАСА

Введение

Марс не место для слабонервных. Здесь сухо, каменисто и очень холодно. Четвертая планета от Солнца, Марс является одной из двух ближайших планетарных соседей Земли (другой является Венера). Марс — одна из планет, которую легче всего обнаружить на ночном небе — она выглядит как ярко-красная светящаяся точка.

Несмотря на негостеприимность к людям, роботы-исследователи, такие как новый марсоход НАСА «Настойчивость», служат первооткрывателями, которые в конечном итоге доставят людей на поверхность Красной планеты.

Дальше: исследование Марса и миссии >

Тёзка

Тёзка

Марс был назван древними римлянами в честь своего бога войны, потому что его красноватый цвет напоминал кровь. Другие цивилизации также назвали планету по этому признаку — например, египтяне называли ее «Хер Дешер», что означает «красная». Даже сегодня ее часто называют «Красной планетой», потому что минералы железа в марсианской грязи окисляются или ржавеют, в результате чего поверхность выглядит красной.

Потенциал для жизни

Потенциал для жизни

Ученые не ожидают найти на Марсе живые существа, которые в настоящее время процветают. Вместо этого они ищут признаки жизни, существовавшей давным-давно, когда Марс был теплее и был покрыт водой.

Размер и расстояние

Размер и расстояние

При радиусе 2106 миль (3390 километров) Марс примерно вдвое меньше Земли. Если бы Земля была размером с пятицентовую монету, Марс был бы размером с малину.

При среднем расстоянии 142 миллиона миль (228 миллионов километров) Марс находится в 1,5 астрономических единицах от Солнца. Одна астрономическая единица (сокращенно AU) — это расстояние от Солнца до Земли. С такого расстояния солнечному свету требуется 13 минут, чтобы добраться от Солнца до Марса.

Орбита и вращение

Орбита и вращение

Поскольку Марс вращается вокруг Солнца, он совершает один оборот за 24,6 часа, что очень похоже на один день на Земле (23,9часы). Марсианские дни называются sols — сокращение от «солнечный день». Год на Марсе длится 669,6 солов, что соответствует 687 земным дням.

Ось вращения Марса наклонена на 25 градусов по отношению к плоскости его орбиты вокруг Солнца. Это еще одно сходство с Землей, осевой наклон которой составляет 23,4 градуса. Как и на Земле, на Марсе есть отчетливые сезоны, но они длятся дольше, чем сезоны здесь, на Земле, поскольку Марсу требуется больше времени, чтобы вращаться вокруг Солнца (потому что он находится дальше). И хотя здесь, на Земле, сезоны равномерно распределены в течение года и длятся 3 месяца (или четверть года), на Марсе сезоны различаются по продолжительности из-за эллиптической яйцевидной орбиты Марса вокруг Солнца.

Весна в северном полушарии (осень в южном) — самый продолжительный сезон на 194 сола. Осень в северном полушарии (весна в южном) самая короткая — 142 дня. Северная зима / южное лето — 154 соля, а северное лето / южная зима — 178 солей.

3D-модель Марса, планеты земной группы. Авторы и права: Приложения и разработка технологий визуализации НАСА (VTAD) › Параметры загрузки

Луны

Луны

У Марса есть две маленькие луны, Фобос и Деймос, которые могут быть захвачены астероидами. Они имеют форму картофеля, потому что их масса слишком мала для того, чтобы гравитация сделала их сферическими.

Крупнейший спутник Марса Фобос, вид с орбитального аппарата Mars Reconnaissance Orbiter в 2008 году. Предоставлено: NASA/JPL-Caltech/University of Arizona | › Полное изображение и подпись

Луны получили свои имена от лошадей, которые тянули колесницу греческого бога войны Ареса.

Фобос, самая внутренняя и большая луна, покрыта сильными кратерами с глубокими канавками на поверхности. Он медленно движется к Марсу и врежется в планету или развалится примерно через 50 миллионов лет.

Деймос примерно вдвое меньше Фобоса и вращается в два с половиной раза дальше от Марса. Деймос странной формы покрыт рыхлой грязью, которая часто заполняет кратеры на его поверхности, из-за чего он кажется более гладким, чем рябый Фобос.

Иди дальше. Исследуйте спутники Марса ›

Кольца

Кольца

У Марса нет колец. Однако через 50 миллионов лет, когда Фобос врежется в Марс или развалится на части, вокруг Красной планеты может образоваться пыльное кольцо.

Формирование

Формирование

Когда Солнечная система приняла свое нынешнее расположение около 4,5 миллиардов лет назад, Марс сформировался, когда гравитация втянула закрученный газ и пыль внутрь, чтобы стать четвертой планетой от Солнца. Марс примерно вдвое меньше Земли и, как и другие планеты земной группы, имеет центральное ядро, каменистую мантию и твердую кору.

Структура

Структура

Марс имеет плотное ядро ​​в центре радиусом от 930 до 1300 миль (от 1500 до 2100 километров). Он сделан из железа, никеля и серы. Ядро окружает скалистая мантия толщиной от 770 до 1170 миль (от 1240 до 1880 километров), а над ней — кора из железа, магния, алюминия, кальция и калия. Эта кора имеет глубину от 6 до 30 миль (от 10 до 50 километров).

Поверхность

Поверхность

Красная планета на самом деле многоцветная. На поверхности мы видим такие цвета, как коричневый, золотой и коричневый. Причина, по которой Марс выглядит красноватым, связана с окислением или ржавлением железа в горных породах, реголите (марсианской «почве») и марсианской пыли. Эта пыль поднимается в атмосферу, и издалека планета кажется в основном красной.

Интересно, что хотя диаметр Марса составляет примерно половину диаметра Земли, его поверхность имеет почти такую ​​же площадь, что и суша Земли. Его вулканы, ударные кратеры, движение земной коры и атмосферные условия, такие как пыльные бури, изменили ландшафт Марса за многие годы, создав некоторые из самых интересных топографических особенностей Солнечной системы.

Большая система каньонов под названием Valles Marineris достаточно длинна, чтобы протянуться от Калифорнии до Нью-Йорка — более 3000 миль (4800 километров). Этот марсианский каньон составляет 200 миль (320 километров) в самом широком месте и 4,3 мили (7 километров) в самом глубоком месте. Это примерно в 10 раз больше Большого Каньона Земли.

Большой масштаб

В этой инфографике используются составные изображения орбитального аппарата и контур Соединенных Штатов, чтобы показать масштаб Долины Маринер. Предоставлено: НАСА/Скотт Халм | › Полное изображение и подпись

Марс является домом для самого большого вулкана в Солнечной системе, горы Олимп. Он в три раза выше земной горы Эверест, а его основание размером с штат Нью-Мексико.

Марс, по-видимому, имел водное прошлое, с древними сетями речных долин, дельтами и дном озер, а также камнями и минералами на поверхности, которые могли образоваться только в жидкой воде. Некоторые особенности предполагают, что около 3,5 миллиардов лет назад на Марсе произошли огромные наводнения.

Сегодня на Марсе есть вода, но марсианская атмосфера слишком тонкая, чтобы жидкая вода могла долго существовать на поверхности. Сегодня вода на Марсе находится в виде водяного льда прямо под поверхностью в полярных регионах, а также в виде соленой (соленой) воды, которая сезонно стекает по склонам некоторых холмов и стенкам кратеров.

Атмосфера

Атмосфера

Марс имеет тонкую атмосферу, состоящую в основном из углекислого газа, азота и аргона. Для наших глаз небо было бы туманным и красным из-за взвешенной пыли, а не знакомого голубого оттенка, который мы видим на Земле. Разреженная атмосфера Марса не обеспечивает достаточной защиты от ударов таких объектов, как метеориты, астероиды и кометы.

Температура на Марсе может достигать 70 градусов по Фаренгейту (20 градусов по Цельсию) или опускаться до -225 градусов по Фаренгейту (-153 градуса по Цельсию). А поскольку атмосфера очень тонкая, солнечное тепло легко покидает эту планету. Если бы вы стояли на поверхности Марса на экваторе в полдень, это было бы похоже на весну у ваших ног (75 градусов по Фаренгейту или 24 градуса по Цельсию) и зиму у вас в голове (32 градуса по Фаренгейту или 0 градусов по Цельсию).

Иногда ветры на Марсе бывают достаточно сильными, чтобы создавать пыльные бури, охватывающие большую часть планеты. После таких бурь могут пройти месяцы, прежде чем вся пыль осядет.

Магнитосфера

Магнитосфера

Марс сегодня не имеет глобального магнитного поля, но области марсианской коры в южном полушарии сильно намагничены, что указывает на следы магнитного поля 4 миллиарда лет назад.

Марс: все, что вам нужно знать о красной планете

Космос поддерживается своей аудиторией. Когда вы покупаете по ссылкам на нашем сайте, мы можем получать партнерскую комиссию. Вот почему вы можете доверять нам.

Марс — четвертая планета от Солнца, также известная как Красная планета. (Изображение предоставлено РистоАрнаудовым через Getty Images)

Марс, четвертая планета от Солнца, славится своим ржаво-красным цветом. Красная планета — это холодный пустынный мир с очень разреженной атмосферой. Но пыльная, безжизненная (насколько нам известно) планета далеко не унылая.

Феноменальные пыльные бури могут стать такими большими, что охватят всю планету, температура может стать настолько низкой, что углекислый газ в атмосфере конденсируется прямо в снег или иней, а марсотрясения — марсианская версия землетрясения — регулярно сотрясают обстановку.

Поэтому неудивительно, что этот маленький красный камень продолжает интриговать ученых и является одним из наиболее изученных тел в Солнечной системе , согласно NASA Science .

Связанный: Сколько времени нужно, чтобы добраться до Марса?

В соответствии с кровавым цветом Красной планеты римляне назвали ее в честь своего бога войны. По правде говоря, римляне копировали древних греков, которые также назвали планету в честь своего бога войны Ареса.

Другие цивилизации также обычно давали названия планетам в зависимости от их цвета — например, египтяне называли ее «Хер Дешер», что означает «красная», а древние китайские астрономы называли ее «огненной звездой».

Почему Марс называют Красной планетой?

Яркий цвет ржавчины, которым известен Марс, обусловлен богатыми железом минералами в его реголите — рыхлой пылью и камнем, покрывающими его поверхность. Почва Земли также представляет собой своего рода реголит, хотя и насыщенный органическими веществами. По данным НАСА, минералы железа окисляются или ржавеют, в результате чего почва становится красной.

Ландшафт Марса

Холодная разреженная атмосфера планеты означает, что жидкая вода, вероятно, не может существовать на поверхности Марса в течение сколько-нибудь заметного периода времени. Особенности, называемые повторяющимися линиями склона, могут иметь струи соленой воды, стекающей по поверхности, но это свидетельство оспаривается; некоторые ученые утверждают, что водород, обнаруженный с орбиты в этом регионе, может указывать на наличие соленых солей. Это означает, что хотя эта планета-пустыня составляет всего половину диаметра Земли, на ней столько же суши.

Красная планета является домом для самой высокой горы и самой глубокой и длинной долины в Солнечной системе. Гора Олимп имеет высоту примерно 17 миль (27 километров), что примерно в три раза выше горы Эверест, а система долин Долины Маринер, названная в честь зонда Mariner 9, обнаружившего ее в 1971 году, достигает глубины 6 миль (10 км). ) и тянется с востока на запад примерно на 2500 миль (4000 км), что составляет около одной пятой расстояния вокруг Марса и близко к ширине Австралии.

Ученые считают, что Морские Долины образовались в основном в результате растрескивания земной коры по мере ее растяжения. Отдельные каньоны в системе имеют ширину до 60 миль (100 км). Каньоны сливаются в центральной части долины Маринер в районе шириной до 370 миль (600 км). Большие каналы, выходящие из концов некоторых каньонов, и слоистые отложения внутри позволяют предположить, что каньоны когда-то могли быть заполнены жидкой водой.

Марс также имеет самые большие вулканы в Солнечной системе, Олимп Монс является одним из них. Массивный вулкан диаметром около 370 миль (600 км) достаточно широк, чтобы покрыть штат Нью-Мексико. Олимп Монс — это щитовой вулкан со склонами, которые постепенно поднимаются, как у гавайских вулканов, и был создан извержениями лавы, которая текла на большие расстояния, прежде чем затвердеть. На Марсе также есть много других форм вулканического рельефа, от небольших конусов с крутыми склонами до огромных равнин, покрытых застывшей лавой. Некоторые незначительные извержения все еще могут происходить на планете сегодня.

Связанный: Космические вулканы: Происхождение, разновидности и извержения

Гора Олимп — самый большой известный вулкан в Солнечной системе. Это цифровое мозаичное изображение вулкана было получено орбитальным аппаратом НАСА «Викинг-1». (Изображение предоставлено NASA/JPL-Caltech/USGS)

Каналы, долины и овраги встречаются по всему Марсу, и предполагается, что жидкая вода могла течь по поверхности планеты в последнее время. Некоторые каналы могут иметь ширину 60 миль (100 км) и длину 1200 миль (2000 км). Вода все еще может находиться в трещинах и порах подземных пород. Исследование, проведенное учеными в 2018 году, показало, что соленая вода под поверхностью Марса может содержать значительное количество кислорода, который может поддерживать микробную жизнь. Однако количество кислорода зависит от температуры и давления; Температура на Марсе время от времени меняется по мере смещения наклона его оси вращения.

Многие районы Марса представляют собой плоские низменные равнины. Самые низкие северные равнины — одни из самых плоских и гладких мест в Солнечной системе, потенциально созданные водой, которая когда-то текла по марсианской поверхности. Северное полушарие в основном расположено на более низкой высоте, чем южное полушарие, что позволяет предположить, что кора на севере может быть тоньше, чем на юге. Эта разница между севером и югом может быть связана с очень сильным ударом вскоре после рождения Марса.

Количество кратеров на Марсе значительно варьируется от места к месту, в зависимости от возраста поверхности. Большая часть поверхности южного полушария чрезвычайно старая, и на ней много кратеров, в том числе самая большая на планете равнина Эллада шириной 1400 миль (2300 км), в то время как поверхность северного полушария моложе и поэтому имеет меньше кратеров. Некоторые вулканы также имеют всего несколько кратеров, что говорит о том, что они извергались недавно, в результате чего образовавшаяся лава покрыла все старые кратеры. Вокруг некоторых кратеров имеются отложения обломков необычного вида, напоминающие затвердевшие селевые потоки, что может указывать на то, что ударный элемент столкнулся с подземными водами или льдом.

В 2018 году космический корабль Европейского космического агентства «Марс Экспресс» обнаружил то, что могло быть взвесь воды и зерна под ледяным плато Южным. (В некоторых отчетах оно описывается как «озеро», но неясно, сколько реголита находится в воде.) Говорят, что этот водоем имеет диаметр около 12,4 миль (20 км). Его подземное расположение напоминает аналогичные подземные озера в Антарктиде, в которых, как было обнаружено, обитают микробы. В конце года «Марс Экспресс» также обнаружил огромную ледяную зону в кратере Королева на Красной планете.

Полярные шапки Марса

Обширные отложения того, что кажется тонкослоистым нагромождением водяного льда и пыли, простираются от полюсов до 80 градусов широты в обоих марсианских полушариях. Вероятно, они были отложены атмосферой в течение длительного периода времени. Поверх большей части этих слоистых отложений в обоих полушариях лежат шапки из водяного льда, которые остаются замороженными круглый год.

В зимнее время появляются дополнительные сезонные морозные шапки. Они сделаны из твердого углекислого газа, также известного как «сухой лед», который сконденсировался из углекислого газа в атмосфере. (Думаю, воздух Марса составляет около 95% углекислого газа по объему.) В самую глубокую часть зимы этот мороз может распространяться от полюсов до широт до 45 градусов или на полпути к экватору. Согласно отчету, опубликованному в Journal of Geophysical Research-Planets, слой сухого льда имеет пушистую текстуру, похожую на свежевыпавший снег.

Климат Марса

Лед и пыль образуют марсианские полярные шапки. (Изображение предоставлено НАСА/Лаборатории реактивного движения/Университет штата Аризона, Р. Лук)

(открывается в новой вкладке)

Марс намного холоднее Земли, в значительной степени из-за его большего расстояния от Солнца. Средняя температура составляет около минус 80 градусов по Фаренгейту (минус 60 градусов по Цельсию), хотя может варьироваться от минус 195 F (минус 125 C) у полюсов зимой до 70 F (20 C) в полдень у экватора.

Атмосфера Марса, богатая углекислым газом, примерно в 100 раз менее плотная, чем в среднем Земля, но, тем не менее, она достаточно плотная, чтобы поддерживать погоду, облака и ветер. Плотность атмосферы меняется в зависимости от сезона, так как зимой углекислый газ вымерзает из марсианского воздуха. В древнем прошлом атмосфера, вероятно, была значительно толще и могла поддерживать воду, текущую по поверхности планеты. Со временем более легкие молекулы в марсианской атмосфере вырвались под давлением солнечного ветра, который повлиял на атмосферу, поскольку у Марса нет глобального магнитного поля. Сегодня этот процесс изучается миссией НАСА MAVEN (Mars Atmosphere and Volatile Evolution).

Марсианский разведывательный орбитальный аппарат НАСА впервые обнаружил снежные облака из двуокиси углерода, что сделало Марс единственным известным телом в Солнечной системе, где такая необычная зимняя погода. Красная планета также вызывает падение водяного льда из облаков.

Пыльные бури на Марсе — крупнейшие в Солнечной системе, способны покрыть всю Красную планету и длятся месяцами. Одна из теорий относительно того, почему пыльные бури на Марсе могут стать такими сильными, заключается в том, что переносимые по воздуху частицы пыли поглощают солнечный свет, нагревая марсианскую атмосферу в их окрестностях. Затем теплые карманы воздуха текут в более холодные регионы, создавая ветры. Сильные ветры поднимают с земли больше пыли, которая, в свою очередь, нагревает атмосферу, поднимая больше ветра и поднимая больше пыли.

Эти пыльные бури могут представлять серьезную опасность для роботов на поверхности Марса. Например, марсоход НАСА «Оппортьюнити» погиб после того, как в 2018 году его охватил гигантский шторм, который неделями блокировал доступ солнечного света к солнечным панелям робота.

Марсоход Curiosity НАСА сфотографировал эти дрейфующие облака 17 мая 2019 года, на 2410-й марсианский день, или сол, с помощью своих черно-белых навигационных камер. (Изображение предоставлено NASA/JPL-Caltech)

(открывается в новой вкладке)

Орбита Марса

Марс расположен дальше от Солнца, чем Земля, поэтому год на Красной планете длиннее — 687 дней по сравнению с 365 днями на нашей планете. Однако две планеты имеют одинаковую продолжительность дня; Марсу требуется около 24 часов 40 минут, чтобы совершить один оборот вокруг своей оси, по сравнению с 24 часами для Земли.

Ось Марса, как и земная, наклонена по отношению к Солнцу. Это означает, что, как и на Земле, количество солнечного света, падающего на определенные части Красной планеты, может сильно различаться в течение года, что дает Марсу сезоны.

Орбита Марса: Краткие факты

Среднее расстояние от Солнца : 141 633 260 миль (227 936 640 км). Для сравнения: в 1,524 раза больше, чем на Земле.

Перигелий (ближайшее сближение с Солнцем) : 128 400 000 миль (206 600 000 км). Для сравнения: в 1,404 раза больше, чем на Земле.

Афелий (самое дальнее расстояние от Солнца) : 154 900 000 миль (249 200 000 км). Для сравнения: в 1,638 раза больше, чем на Земле.

Однако времена года на Марсе более экстремальные, чем на Земле, потому что эллиптическая овальная орбита Красной планеты вокруг Солнца более вытянута, чем у любой другой большой планеты. Когда Марс находится ближе всего к Солнцу, его южное полушарие наклонено к нашей звезде, что дает планете короткое теплое лето, а северное полушарие переживает короткую холодную зиму. Когда Марс находится дальше всего от Солнца, северное полушарие наклонено к Солнцу, что обеспечивает долгое мягкое лето, а южное полушарие переживает долгую холодную зиму.

Наклон оси Красной планеты сильно колеблется с течением времени, потому что она не стабилизируется большой луной, такой как Земля. Эта ситуация привела к разному климату на поверхности Марса на протяжении всей его истории. Исследование 2017 года предполагает, что изменение наклона также повлияло на выброс метана в атмосферу Марса, вызывая временные периоды потепления, которые позволяли течь воде.

Размер, состав и структура

Марс имеет диаметр 4 220 миль (6 791 км) — намного меньше, чем Земля, которая составляет 7,926 миль (12 756 км) в ширину. Красная планета примерно на 10% массивнее нашего родного мира, а гравитационное притяжение на 38% сильнее. (Человек весом 100 фунтов здесь, на Земле, будет весить всего 62 фунта на Марсе, но его масса будет одинаковой на обеих планетах.)

Состав атмосферы (по объему)

95,32 % углекислого газа, 2,7 % азота, 1,6 % аргона, 0,13 % кислорода и 0,08 % монооксида углерода, с небольшими количествами воды, оксида азота, неона, водорода, дейтерия и кислорода, криптона и ксенона.

Магнитное поле

Марс потерял свое глобальное магнитное поле около 4 миллиардов лет назад, что привело к удалению большей части его атмосферы солнечным ветром. Но сегодня есть области коры планеты, которые могут быть по крайней мере в 10 раз более сильно намагничены, чем что-либо измеренное на Земле, что предполагает, что эти области являются остатками древнего глобального магнитного поля.

Химический состав

Марс, вероятно, имеет твердое ядро, состоящее из железа, никеля и серы. Мантия Марса, вероятно, похожа на земную тем, что состоит в основном из перидотита, состоящего в основном из кремния, кислорода, железа и магния. Кора, вероятно, в основном состоит из базальта вулканической породы, который также распространен в коре Земли и Луны, хотя некоторые породы коры, особенно в северном полушарии, могут быть формой андезита, вулканической породы, которая содержит больше кремний, чем базальт.

Внутренняя структура

Посадочный модуль НАСА InSight исследует внутреннюю часть Марса с момента приземления вблизи экватора планеты в ноябре 2018 года. InSight измеряет и характеризует марсотрясения, а члены миссии отслеживают колебания наклона Марса с течением времени, точно отслеживая положение спускаемого аппарата на поверхности планеты.

Эти данные открыли ключевую информацию о внутренней структуре Марса. Например, члены команды InSight недавно подсчитали, что ядро ​​планеты имеет ширину от 1110 до 1300 миль (от 1780 до 2080 км). Наблюдения InSight также показывают, что толщина коры Марса составляет в среднем от 14 до 45 миль (24 и 72 км), а мантия составляет остальную часть (неатмосферного) объема планеты.

Для сравнения: ядро ​​Земли имеет ширину около 4 400 миль (7 100 км) — больше, чем сам Марс, — а толщина мантии составляет примерно 1 800 миль (2 900 км). Земля имеет два вида коры, континентальную и океаническую, средняя толщина которых составляет около 25 миль (40 км) и 5 ​​миль (8 км) соответственно.

Спутники Марса

Два спутника Марса, Фобос и Деймос, были открыты американским астрономом Асафом Холлом в течение недели в 1877 году. Холл почти отказался от поисков спутника Марса, но его жена, Анджелина, убеждала его. Следующей ночью он обнаружил Деймос, а через шесть дней — Фобос. Он назвал луны в честь сыновей греческого бога войны Ареса — Фобос означает «страх», а Деймос — «разгром».

Изображение Фобоса, полученное марсианским разведывательным орбитальным аппаратом НАСА (Изображение предоставлено НАСА/Лаборатории реактивного движения/Университет Аризоны) покрытые пылью и рыхлыми камнями. Они крошечные по сравнению с земной Луной и имеют неправильную форму, поскольку им не хватает гравитации, чтобы принять более круглую форму. Самая широкая точка Фобоса составляет около 17 миль (27 км), а самая широкая часть Деймоса — примерно 9миль (15 км). (Луна Земли имеет ширину 2159 миль или 3475 км.)

Обе луны Марса испещрены кратерами от ударов метеоритов. Поверхность Фобоса также имеет замысловатый рисунок канавок, которые могут быть трещинами, образовавшимися после удара, образовавшего самый большой кратер Луны — дыру шириной около 6 миль (10 км), или почти половину ширины Фобоса. Два марсианских спутника всегда обращены к своей родительской планете одним и тем же лицом, точно так же, как наша Луна обращена к Земле.

Изображение Деймоса, полученное марсианским разведывательным орбитальным аппаратом НАСА (Изображение предоставлено NASA/JPL-Caltech/University of Arizona)

Остается неясным, как родились Фобос и Деймос. Это могут быть бывшие астероиды, которые были захвачены гравитационным притяжением Марса, или они могли образоваться на орбите вокруг Марса примерно в то же время, когда планета возникла. Ультрафиолетовый свет, отраженный от Фобоса, является убедительным доказательством того, что Луна является захваченным астероидом, по мнению астрономов из Университета Падуи в Италии.

Фобос постепенно движется по спирали к Марсу, с каждым столетием приближаясь к Красной планете примерно на 6 футов (1,8 метра). В течение 50 миллионов лет Фобос либо врежется в Марс, либо расколется и образует кольцо обломков вокруг планеты.

Исследования и исследования

Галилео Галилей, первый человек, наблюдавший Марс в телескоп в 1610 году. В следующем столетии астрономы обнаружили полярные ледяные шапки планеты. В 19-м и 20-м веках некоторые исследователи, наиболее известный из которых Персиваль Лоуэлл, считали, что видели на Марсе сеть длинных прямых каналов, которые намекали на возможную цивилизацию. Однако эти наблюдения оказались ошибочной интерпретацией геологических особенностей.

Несколько марсианских камней упали на Землю в течение тысячелетий, предоставив ученым редкую возможность изучить части Марса, не покидая нашу планету. Одной из самых спорных находок стал Allan Hills 84001 (ALH84001) — марсианский метеорит, который, по данным 1996, вероятно, содержит крошечные окаменелости и другие свидетельства жизни на Марсе. Другие исследователи подвергают сомнению эту гипотезу, но команда, проводившая знаменитое исследование 1996 года, твердо придерживается своей интерпретации, и дебаты по поводу ALH84001 продолжаются и сегодня.

В 2018 году отдельное исследование метеоритов показало, что органические молекулы — углеродсодержащие строительные блоки жизни, хотя и не обязательно свидетельство самой жизни — могли образоваться на Марсе в результате химических реакций, подобных батарейкам.

Роботизированный космический корабль начал наблюдения за Марсом в 1960-х годах, когда Соединенные Штаты запустили Маринер-4 в 1964 году и Маринеры 6 и 7 в 1969 году. Эти первые миссии показали, что Марс представляет собой бесплодный мир, без каких-либо признаков жизни или цивилизаций. Такие люди, как Лоуэлл представлял себе там. В 1971 году «Маринер-9» облетел Марс, нанеся на карту около 80 % планеты и обнаружив ее вулканы и большие каньоны.

Советский Союз также запускал многочисленные космические корабли «Красная планета» в 1960-х и начале 1970-х годов, но большинство из этих миссий потерпели неудачу. Марс 2 (1971) и «Марс-3» (1971 г.) работали успешно, но не смогли нанести на карту поверхность из-за пыльных бурь. Посадочный модуль НАСА «Викинг-1» приземлился на поверхности Марса в 1976 году, совершив первую успешную посадку на Красной планете. Его близнец, «Викинг-2», через шесть недель приземлился в другом районе Марса.

Посадочные аппараты «Викинг» сделали первые снимки марсианской поверхности крупным планом, но не нашли убедительных доказательств существования жизни. Тем не менее, снова возникли споры: Гил Левин, главный исследователь эксперимента по обнаружению жизни «Маркетинговый выпуск викингов», всегда утверждал, что посадочные модули шпионили доказательства микробного метаболизма в марсианской грязи. (Левин умер в июле 2021 года в возрасте 9 лет7.)

Следующими двумя кораблями, успешно достигшими Красной планеты, были посадочный модуль Mars Pathfinder и орбитальный аппарат Mars Global Surveyor. Оба корабля НАСА были запущены в 1996 году. Небольшой робот на борту Pathfinder по имени Sojourner — первый колесный вездеход. когда-либо исследовал поверхность другой планеты — рискнул исследовать поверхность планеты, анализируя горные породы в течение 95 земных дней.

В 2001 году НАСА запустило орбитальный аппарат Mars Odyssey , который обнаружил огромное количество водяного льда под марсианской поверхностью, в основном в верхних 3 футах (1 метр). Остается неясным, находится ли под ним больше воды, поскольку зонд не может видеть воду глубже.

Mars Odyssey запущен с мыса Канаверал, штат Флорида, 7 апреля 2001 г. (Изображение предоставлено НАСА)

В 2003 г. Марс прошел ближе к Земле, чем когда-либо за последние 60 000 лет. . В том же году НАСА запустило два марсохода размером с тележку для гольфа, получившие прозвища Spirit и Opportunity, которые исследовали различные области марсианской поверхности после приземления в январе 2004 года. Оба марсохода обнаружили много признаков того, что вода когда-то текла по поверхности планеты.

Spirit и Opportunity изначально выполняли трехмесячные миссии на поверхности, но оба продолжали скитаться гораздо дольше. НАСА не объявляло Spirit мертвым до 2011 года, а Opportunity все еще действовал, пока в середине 2018 года не разразилась пыльная буря.

В 2008 году НАСА отправило посадочный модуль под названием «Феникс» на далекие северные равнины Марса. Робот подтвердил наличие водяного льда в ближних недрах среди других находок.

В 2011 году марсианская научная лаборатория НАСА отправила марсоход Curiosity для изучения прошлого потенциала Марса для жизни. Нет. Спустя долгое время после приземления в кратере Гейла на Красной планете в августе 2012 года робот размером с автомобиль определил, что в древнем прошлом в этом районе находилась долгоживущая, потенциально обитаемая система озер и ручьев. Curiosity также обнаружил сложные органические молекулы и зафиксировал сезонные колебания концентрации метана в атмосфере.

У НАСА есть два других орбитальных аппарата, работающих вокруг планеты — Mars Reconnaissance Orbiter и MAVEN (Mars Atmosphere and Volatile Evolution), которые прибыли на Марс в 2006 и 2014 годах соответственно. У Европейского космического агентства (ЕКА) также есть два космических корабля на орбите планеты: Mars Express и Trace Gas Orbiter.

В сентябре 2014 года индийский марсианский орбитальный аппарат также достиг Красной планеты, став четвертой страной, успешно вышедшей на орбиту вокруг Марса.

В ноябре 2018 года НАСА высадило на поверхность стационарный аппарат под названием Mars InSight. Как отмечалось выше, InSight исследует внутреннюю структуру и состав Марса, в первую очередь, измеряя и описывая марсотрясения.

Марсоход NASA Perseverance сделал это селфи над скалой по прозвищу «Рошет» 10 сентября 2021 года. (Изображение предоставлено NASA/JPL-Caltech/MSSS) — охотящийся и собирающий образцы марсоход Perseverance в июле 2020 года. Perseverance, размером примерно с Curiosity, приземлился на дно марсианского кратера Jezero в феврале 2021 года вместе с крошечным вертолетом, демонстрирующим технологии, известным как Ingenuity.

По состоянию на сентябрь 2021 года Ingenuity совершила более дюжины полетов на Марс, доказав, что воздушное исследование планеты возможно. Perseverance задокументировала первые полеты вертолета весом 4 фунта (1,8 кг), а затем всерьез сосредоточилась на своей научной миссии. Большой марсоход уже собрал несколько образцов, часть большого тайника, который будет доставлен на Землю, возможно, уже в 2031 году в рамках совместной кампании НАСА и ЕКА.

В июле 2020 года также были запущены первая марсианская миссия Объединенных Арабских Эмиратов под названием «Надежда» и первая полностью отечественная марсианская программа Китая, Tianwen 1. Орбитальный аппарат «Надежда» прибыл на Марс в феврале 2021 года и изучает атмосферу планеты, погоду и климат.

Истории по теме:

Tianwen 1, состоящий из орбитального аппарата и дуэта спускаемого аппарата и вездехода, также вышел на орбиту Марса в феврале 2021 года. Приземлившийся элемент приземлился несколько месяцев спустя, в мае. Марсоход Tianwen 1, названный Zhurong, вскоре скатился по трапу посадочной платформы и начал исследовать марсианскую поверхность.

ЕКА также работает над созданием марсохода в рамках сотрудничества с Россией по программе «ЭкзоМарс». Этот робот по имени Розалинд Франклин должен был быть запущен в середине 2020 года, но проблемы с парашютом и другие проблемы отложили запуск до следующей возможности, в 2022 году (Марс и Земля должным образом выравниваются для межпланетных миссий только раз в 26 месяцев). Розалинда Среди прочих задач Франклин будет искать признаки прошлой жизни на Марсе. Робот будет использовать бур, чтобы углубиться в Красную планету, собирая образцы почвы примерно на 2 метра (6,5 футов) под землей.

Потерянные миссии на Марс

Марс далеко не та планета, до которой легко добраться. НАСА, Россия, Европейское космическое агентство, Китай, Япония и Советский Союз вместе потеряли много космических аппаратов в своем стремлении исследовать Красную планету. Известные примеры включают (но не ограничиваются):

1992 — NASA Mars Observer

1996 — Россия Mars 96

1998 — Mars Climate Orbiter NASA, Nozomi Япония

1999 — NASA’s Mars B003eagle Lander

3 2 посадочных модуля

2011 г. — российская миссия «Фобус-Грунт» на Фобос с китайским орбитальным аппаратом Yinghuo-1

2016 г. — испытательный посадочный модуль ЕКА «Скиапарелли»

Будущие пилотируемые миссии

Не только роботы получают билет на Марс. Группа ученых из правительственных учреждений, научных кругов и промышленности определила, что к 2030-м годам должна стать возможной пилотируемая миссия на Марс под руководством НАСА.

В конце 2017 года администрация президента Дональда Трампа поручила НАСА отправить людей обратно на Луну, прежде чем отправиться на Марс. НАСА работает над этой целью в рамках программы под названием Artemis, целью которой является обеспечение устойчивого и долгосрочного присутствия человека на Луне и вокруг нее к концу 2020-х годов. По словам представителей НАСА, уроки и навыки, извлеченные из этой лунной операции, помогут проложить путь к посадке на Марс.

За последние несколько десятилетий роботизированные миссии на Красную планету увенчались большим успехом, но доставить людей на Марс по-прежнему сложно. С современными ракетными технологиями людям потребуется не менее шести месяцев, чтобы отправиться на Марс. Таким образом, исследователи Красной планеты будут длительное время подвергаться воздействию радиации дальнего космоса и микрогравитации, которые оказывают разрушительное воздействие на организм человека. Выполнение действий в условиях умеренной гравитации на Марсе может оказаться чрезвычайно трудным после многих месяцев в условиях микрогравитации. Исследования эффектов микрогравитации продолжаются на Международной космической станции.

НАСА — не единственная организация, стремящаяся пилотировать Марс. Другие страны, в том числе Китай и Россия, также объявили о своих целях по отправке людей на Красную планету.

А Илон Маск, основатель и генеральный директор SpaceX, давно подчеркивал, что создал компанию еще в 2002 году в первую очередь для того, чтобы помочь человечеству заселить Красную планету. В настоящее время SpaceX разрабатывает и тестирует полностью многоразовую транспортную систему для дальнего космоса под названием Starship, которая, по мнению Маска, является прорывом, необходимым для того, чтобы наконец доставить людей на Марс.

Дополнительные ресурсы

Исследуйте Марс более подробно с помощью программы NASA Mars Exploration Program (откроется в новой вкладке). Узнайте больше о климате Марса с Национальной метеорологической службой (откроется в новой вкладке). Отправьте свое имя на Марс во время следующего полета НАСА на Красную планету.

Библиография

НАСА. Информационный бюллетень Марса. НАСА. Получено 11 июля 2022 г. с www.nssdc.gsfc.nasa.gov/planetary/factsheet/marsfact.html (откроется в новой вкладке)

НАСА. Марс. НАСА. Получено 11 июля 2022 г. с www.solarsystem.nasa.gov/planets/mars/overview/ (открывается в новой вкладке)

НАСА. Исследование Марса НАСА. НАСА. Получено 11 июля 2022 г. с https://mars.nasa.gov/ (открывается в новой вкладке)

НАСА. Отправьте свое имя на Марс. НАСА. Получено 11 июля 2022 г. с www.mars.nasa.gov/participate/send-your-name/future (открывается в новой вкладке)

Министерство торговли США, Северная Аравия. Планета Марс. Национальная служба погоды. Проверено 11 июля 2022 г. с www.weather.gov/fsd/mars (открывается в новой вкладке)

Присоединяйтесь к нашим космическим форумам, чтобы продолжать обсуждать последние миссии, ночное небо и многое другое! А если у вас есть новость, исправление или комментарий, сообщите нам об этом по адресу: [email protected].

Чарльз К. Чой — автор статей для Space.com и Live Science. Он охватывает все, что связано с человеческим происхождением и астрономией, а также физику, животных и общие научные темы. Чарльз имеет степень магистра гуманитарных наук Университета Миссури-Колумбия, Школу журналистики и степень бакалавра гуманитарных наук Университета Южной Флориды. Чарльз побывал на всех континентах Земли, пил прогорклый чай с маслом яка в Лхасе, плавал с морскими львами на Галапагосских островах и даже взбирался на айсберг в Антарктиде. Посетите его на http://www.sciwriter.us

Марс: все, что вам нужно знать о Красной планете

Космос поддерживается его аудиторией. Когда вы покупаете по ссылкам на нашем сайте, мы можем получать партнерскую комиссию. Вот почему вы можете доверять нам.

Марс — четвертая планета от Солнца, также известная как Красная планета. (Изображение предоставлено РистоАрнаудовым через Getty Images)

Марс, четвертая планета от Солнца, славится своим ржаво-красным цветом. Красная планета — это холодный пустынный мир с очень разреженной атмосферой. Но пыльная, безжизненная (насколько нам известно) планета далеко не унылая.

Феноменальные пыльные бури могут стать такими большими, что охватят всю планету, температура может стать настолько низкой, что углекислый газ в атмосфере конденсируется прямо в снег или иней, а марсотрясения — марсианская версия землетрясения — регулярно сотрясают обстановку.

Поэтому неудивительно, что этот маленький красный камень продолжает интриговать ученых и является одним из наиболее изученных тел в Солнечной системе , согласно NASA Science .

Связанный: Сколько времени нужно, чтобы добраться до Марса?

В соответствии с кровавым цветом Красной планеты римляне назвали ее в честь своего бога войны. По правде говоря, римляне копировали древних греков, которые также назвали планету в честь своего бога войны Ареса.

Другие цивилизации также обычно давали названия планетам в зависимости от их цвета — например, египтяне называли ее «Хер Дешер», что означает «красная», а древние китайские астрономы называли ее «огненной звездой».

Почему Марс называют Красной планетой?

Яркий цвет ржавчины, которым известен Марс, обусловлен богатыми железом минералами в его реголите — рыхлой пылью и камнем, покрывающими его поверхность. Почва Земли также представляет собой своего рода реголит, хотя и насыщенный органическими веществами. По данным НАСА, минералы железа окисляются или ржавеют, в результате чего почва становится красной.

Ландшафт Марса

Холодная разреженная атмосфера планеты означает, что жидкая вода, вероятно, не может существовать на поверхности Марса в течение сколько-нибудь заметного периода времени. Особенности, называемые повторяющимися линиями склона, могут иметь струи соленой воды, стекающей по поверхности, но это свидетельство оспаривается; некоторые ученые утверждают, что водород, обнаруженный с орбиты в этом регионе, может указывать на наличие соленых солей. Это означает, что хотя эта планета-пустыня составляет всего половину диаметра Земли, на ней столько же суши.

Красная планета является домом для самой высокой горы и самой глубокой и длинной долины в Солнечной системе. Гора Олимп имеет высоту примерно 17 миль (27 километров), что примерно в три раза выше горы Эверест, а система долин Долины Маринер, названная в честь зонда Mariner 9, обнаружившего ее в 1971 году, достигает глубины 6 миль (10 км). ) и тянется с востока на запад примерно на 2500 миль (4000 км), что составляет около одной пятой расстояния вокруг Марса и близко к ширине Австралии.

Ученые считают, что Морские Долины образовались в основном в результате растрескивания земной коры по мере ее растяжения. Отдельные каньоны в системе имеют ширину до 60 миль (100 км). Каньоны сливаются в центральной части долины Маринер в районе шириной до 370 миль (600 км). Большие каналы, выходящие из концов некоторых каньонов, и слоистые отложения внутри позволяют предположить, что каньоны когда-то могли быть заполнены жидкой водой.

Марс также имеет самые большие вулканы в Солнечной системе, Олимп Монс является одним из них. Массивный вулкан диаметром около 370 миль (600 км) достаточно широк, чтобы покрыть штат Нью-Мексико. Олимп Монс — это щитовой вулкан со склонами, которые постепенно поднимаются, как у гавайских вулканов, и был создан извержениями лавы, которая текла на большие расстояния, прежде чем затвердеть. На Марсе также есть много других форм вулканического рельефа, от небольших конусов с крутыми склонами до огромных равнин, покрытых застывшей лавой. Некоторые незначительные извержения все еще могут происходить на планете сегодня.

Связанный: Космические вулканы: Происхождение, разновидности и извержения

Гора Олимп — самый большой известный вулкан в Солнечной системе. Это цифровое мозаичное изображение вулкана было получено орбитальным аппаратом НАСА «Викинг-1». (Изображение предоставлено NASA/JPL-Caltech/USGS)

Каналы, долины и овраги встречаются по всему Марсу, и предполагается, что жидкая вода могла течь по поверхности планеты в последнее время. Некоторые каналы могут иметь ширину 60 миль (100 км) и длину 1200 миль (2000 км). Вода все еще может находиться в трещинах и порах подземных пород. Исследование, проведенное учеными в 2018 году, показало, что соленая вода под поверхностью Марса может содержать значительное количество кислорода, который может поддерживать микробную жизнь. Однако количество кислорода зависит от температуры и давления; Температура на Марсе время от времени меняется по мере смещения наклона его оси вращения.

Многие районы Марса представляют собой плоские низменные равнины. Самые низкие северные равнины — одни из самых плоских и гладких мест в Солнечной системе, потенциально созданные водой, которая когда-то текла по марсианской поверхности. Северное полушарие в основном расположено на более низкой высоте, чем южное полушарие, что позволяет предположить, что кора на севере может быть тоньше, чем на юге. Эта разница между севером и югом может быть связана с очень сильным ударом вскоре после рождения Марса.

Количество кратеров на Марсе значительно варьируется от места к месту, в зависимости от возраста поверхности. Большая часть поверхности южного полушария чрезвычайно старая, и на ней много кратеров, в том числе самая большая на планете равнина Эллада шириной 1400 миль (2300 км), в то время как поверхность северного полушария моложе и поэтому имеет меньше кратеров. Некоторые вулканы также имеют всего несколько кратеров, что говорит о том, что они извергались недавно, в результате чего образовавшаяся лава покрыла все старые кратеры. Вокруг некоторых кратеров имеются отложения обломков необычного вида, напоминающие затвердевшие селевые потоки, что может указывать на то, что ударный элемент столкнулся с подземными водами или льдом.

В 2018 году космический корабль Европейского космического агентства «Марс Экспресс» обнаружил то, что могло быть взвесь воды и зерна под ледяным плато Южным. (В некоторых отчетах оно описывается как «озеро», но неясно, сколько реголита находится в воде.) Говорят, что этот водоем имеет диаметр около 12,4 миль (20 км). Его подземное расположение напоминает аналогичные подземные озера в Антарктиде, в которых, как было обнаружено, обитают микробы. В конце года «Марс Экспресс» также обнаружил огромную ледяную зону в кратере Королева на Красной планете.

Полярные шапки Марса

Обширные отложения того, что кажется тонкослоистым нагромождением водяного льда и пыли, простираются от полюсов до 80 градусов широты в обоих марсианских полушариях. Вероятно, они были отложены атмосферой в течение длительного периода времени. Поверх большей части этих слоистых отложений в обоих полушариях лежат шапки из водяного льда, которые остаются замороженными круглый год.

В зимнее время появляются дополнительные сезонные морозные шапки. Они сделаны из твердого углекислого газа, также известного как «сухой лед», который сконденсировался из углекислого газа в атмосфере. (Думаю, воздух Марса составляет около 95% углекислого газа по объему.) В самую глубокую часть зимы этот мороз может распространяться от полюсов до широт до 45 градусов или на полпути к экватору. Согласно отчету, опубликованному в Journal of Geophysical Research-Planets, слой сухого льда имеет пушистую текстуру, похожую на свежевыпавший снег.

Климат Марса

Лед и пыль образуют марсианские полярные шапки. (Изображение предоставлено НАСА/Лаборатории реактивного движения/Университет штата Аризона, Р. Лук)

(открывается в новой вкладке)

Марс намного холоднее Земли, в значительной степени из-за его большего расстояния от Солнца. Средняя температура составляет около минус 80 градусов по Фаренгейту (минус 60 градусов по Цельсию), хотя может варьироваться от минус 195 F (минус 125 C) у полюсов зимой до 70 F (20 C) в полдень у экватора.

Атмосфера Марса, богатая углекислым газом, примерно в 100 раз менее плотная, чем в среднем Земля, но, тем не менее, она достаточно плотная, чтобы поддерживать погоду, облака и ветер. Плотность атмосферы меняется в зависимости от сезона, так как зимой углекислый газ вымерзает из марсианского воздуха. В древнем прошлом атмосфера, вероятно, была значительно толще и могла поддерживать воду, текущую по поверхности планеты. Со временем более легкие молекулы в марсианской атмосфере вырвались под давлением солнечного ветра, который повлиял на атмосферу, поскольку у Марса нет глобального магнитного поля. Сегодня этот процесс изучается миссией НАСА MAVEN (Mars Atmosphere and Volatile Evolution).

Марсианский разведывательный орбитальный аппарат НАСА впервые обнаружил снежные облака из двуокиси углерода, что сделало Марс единственным известным телом в Солнечной системе, где такая необычная зимняя погода. Красная планета также вызывает падение водяного льда из облаков.

Пыльные бури на Марсе — крупнейшие в Солнечной системе, способны покрыть всю Красную планету и длятся месяцами. Одна из теорий относительно того, почему пыльные бури на Марсе могут стать такими сильными, заключается в том, что переносимые по воздуху частицы пыли поглощают солнечный свет, нагревая марсианскую атмосферу в их окрестностях. Затем теплые карманы воздуха текут в более холодные регионы, создавая ветры. Сильные ветры поднимают с земли больше пыли, которая, в свою очередь, нагревает атмосферу, поднимая больше ветра и поднимая больше пыли.

Эти пыльные бури могут представлять серьезную опасность для роботов на поверхности Марса. Например, марсоход НАСА «Оппортьюнити» погиб после того, как в 2018 году его охватил гигантский шторм, который неделями блокировал доступ солнечного света к солнечным панелям робота.

Марсоход Curiosity НАСА сфотографировал эти дрейфующие облака 17 мая 2019 года, на 2410-й марсианский день, или сол, с помощью своих черно-белых навигационных камер. (Изображение предоставлено NASA/JPL-Caltech)

(открывается в новой вкладке)

Орбита Марса

Марс расположен дальше от Солнца, чем Земля, поэтому год на Красной планете длиннее — 687 дней по сравнению с 365 днями на нашей планете. Однако две планеты имеют одинаковую продолжительность дня; Марсу требуется около 24 часов 40 минут, чтобы совершить один оборот вокруг своей оси, по сравнению с 24 часами для Земли.

Ось Марса, как и земная, наклонена по отношению к Солнцу. Это означает, что, как и на Земле, количество солнечного света, падающего на определенные части Красной планеты, может сильно различаться в течение года, что дает Марсу сезоны.

Орбита Марса: Краткие факты

Среднее расстояние от Солнца : 141 633 260 миль (227 936 640 км). Для сравнения: в 1,524 раза больше, чем на Земле.

Перигелий (ближайшее сближение с Солнцем) : 128 400 000 миль (206 600 000 км). Для сравнения: в 1,404 раза больше, чем на Земле.

Афелий (самое дальнее расстояние от Солнца) : 154 900 000 миль (249 200 000 км). Для сравнения: в 1,638 раза больше, чем на Земле.

Однако времена года на Марсе более экстремальные, чем на Земле, потому что эллиптическая овальная орбита Красной планеты вокруг Солнца более вытянута, чем у любой другой большой планеты. Когда Марс находится ближе всего к Солнцу, его южное полушарие наклонено к нашей звезде, что дает планете короткое теплое лето, а северное полушарие переживает короткую холодную зиму. Когда Марс находится дальше всего от Солнца, северное полушарие наклонено к Солнцу, что обеспечивает долгое мягкое лето, а южное полушарие переживает долгую холодную зиму.

Наклон оси Красной планеты сильно колеблется с течением времени, потому что она не стабилизируется большой луной, такой как Земля. Эта ситуация привела к разному климату на поверхности Марса на протяжении всей его истории. Исследование 2017 года предполагает, что изменение наклона также повлияло на выброс метана в атмосферу Марса, вызывая временные периоды потепления, которые позволяли течь воде.

Размер, состав и структура

Марс имеет диаметр 4 220 миль (6 791 км) — намного меньше, чем Земля, которая составляет 7,926 миль (12 756 км) в ширину. Красная планета примерно на 10% массивнее нашего родного мира, а гравитационное притяжение на 38% сильнее. (Человек весом 100 фунтов здесь, на Земле, будет весить всего 62 фунта на Марсе, но его масса будет одинаковой на обеих планетах.)

Состав атмосферы (по объему)

95,32 % углекислого газа, 2,7 % азота, 1,6 % аргона, 0,13 % кислорода и 0,08 % монооксида углерода, с небольшими количествами воды, оксида азота, неона, водорода, дейтерия и кислорода, криптона и ксенона.

Магнитное поле

Марс потерял свое глобальное магнитное поле около 4 миллиардов лет назад, что привело к удалению большей части его атмосферы солнечным ветром. Но сегодня есть области коры планеты, которые могут быть по крайней мере в 10 раз более сильно намагничены, чем что-либо измеренное на Земле, что предполагает, что эти области являются остатками древнего глобального магнитного поля.

Химический состав

Марс, вероятно, имеет твердое ядро, состоящее из железа, никеля и серы. Мантия Марса, вероятно, похожа на земную тем, что состоит в основном из перидотита, состоящего в основном из кремния, кислорода, железа и магния. Кора, вероятно, в основном состоит из базальта вулканической породы, который также распространен в коре Земли и Луны, хотя некоторые породы коры, особенно в северном полушарии, могут быть формой андезита, вулканической породы, которая содержит больше кремний, чем базальт.

Внутренняя структура

Посадочный модуль НАСА InSight исследует внутреннюю часть Марса с момента приземления вблизи экватора планеты в ноябре 2018 года. InSight измеряет и характеризует марсотрясения, а члены миссии отслеживают колебания наклона Марса с течением времени, точно отслеживая положение спускаемого аппарата на поверхности планеты.

Эти данные открыли ключевую информацию о внутренней структуре Марса. Например, члены команды InSight недавно подсчитали, что ядро ​​планеты имеет ширину от 1110 до 1300 миль (от 1780 до 2080 км). Наблюдения InSight также показывают, что толщина коры Марса составляет в среднем от 14 до 45 миль (24 и 72 км), а мантия составляет остальную часть (неатмосферного) объема планеты.

Для сравнения: ядро ​​Земли имеет ширину около 4 400 миль (7 100 км) — больше, чем сам Марс, — а толщина мантии составляет примерно 1 800 миль (2 900 км). Земля имеет два вида коры, континентальную и океаническую, средняя толщина которых составляет около 25 миль (40 км) и 5 ​​миль (8 км) соответственно.

Спутники Марса

Два спутника Марса, Фобос и Деймос, были открыты американским астрономом Асафом Холлом в течение недели в 1877 году. Холл почти отказался от поисков спутника Марса, но его жена, Анджелина, убеждала его. Следующей ночью он обнаружил Деймос, а через шесть дней — Фобос. Он назвал луны в честь сыновей греческого бога войны Ареса — Фобос означает «страх», а Деймос — «разгром».

Изображение Фобоса, полученное марсианским разведывательным орбитальным аппаратом НАСА (Изображение предоставлено НАСА/Лаборатории реактивного движения/Университет Аризоны) покрытые пылью и рыхлыми камнями. Они крошечные по сравнению с земной Луной и имеют неправильную форму, поскольку им не хватает гравитации, чтобы принять более круглую форму. Самая широкая точка Фобоса составляет около 17 миль (27 км), а самая широкая часть Деймоса — примерно 9миль (15 км). (Луна Земли имеет ширину 2159 миль или 3475 км.)

Обе луны Марса испещрены кратерами от ударов метеоритов. Поверхность Фобоса также имеет замысловатый рисунок канавок, которые могут быть трещинами, образовавшимися после удара, образовавшего самый большой кратер Луны — дыру шириной около 6 миль (10 км), или почти половину ширины Фобоса. Два марсианских спутника всегда обращены к своей родительской планете одним и тем же лицом, точно так же, как наша Луна обращена к Земле.

Изображение Деймоса, полученное марсианским разведывательным орбитальным аппаратом НАСА (Изображение предоставлено NASA/JPL-Caltech/University of Arizona)

Остается неясным, как родились Фобос и Деймос. Это могут быть бывшие астероиды, которые были захвачены гравитационным притяжением Марса, или они могли образоваться на орбите вокруг Марса примерно в то же время, когда планета возникла. Ультрафиолетовый свет, отраженный от Фобоса, является убедительным доказательством того, что Луна является захваченным астероидом, по мнению астрономов из Университета Падуи в Италии.

Фобос постепенно движется по спирали к Марсу, с каждым столетием приближаясь к Красной планете примерно на 6 футов (1,8 метра). В течение 50 миллионов лет Фобос либо врежется в Марс, либо расколется и образует кольцо обломков вокруг планеты.

Исследования и исследования

Галилео Галилей, первый человек, наблюдавший Марс в телескоп в 1610 году. В следующем столетии астрономы обнаружили полярные ледяные шапки планеты. В 19-м и 20-м веках некоторые исследователи, наиболее известный из которых Персиваль Лоуэлл, считали, что видели на Марсе сеть длинных прямых каналов, которые намекали на возможную цивилизацию. Однако эти наблюдения оказались ошибочной интерпретацией геологических особенностей.

Несколько марсианских камней упали на Землю в течение тысячелетий, предоставив ученым редкую возможность изучить части Марса, не покидая нашу планету. Одной из самых спорных находок стал Allan Hills 84001 (ALH84001) — марсианский метеорит, который, по данным 1996, вероятно, содержит крошечные окаменелости и другие свидетельства жизни на Марсе. Другие исследователи подвергают сомнению эту гипотезу, но команда, проводившая знаменитое исследование 1996 года, твердо придерживается своей интерпретации, и дебаты по поводу ALH84001 продолжаются и сегодня.

В 2018 году отдельное исследование метеоритов показало, что органические молекулы — углеродсодержащие строительные блоки жизни, хотя и не обязательно свидетельство самой жизни — могли образоваться на Марсе в результате химических реакций, подобных батарейкам.

Роботизированный космический корабль начал наблюдения за Марсом в 1960-х годах, когда Соединенные Штаты запустили Маринер-4 в 1964 году и Маринеры 6 и 7 в 1969 году. Эти первые миссии показали, что Марс представляет собой бесплодный мир, без каких-либо признаков жизни или цивилизаций. Такие люди, как Лоуэлл представлял себе там. В 1971 году «Маринер-9» облетел Марс, нанеся на карту около 80 % планеты и обнаружив ее вулканы и большие каньоны.

Советский Союз также запускал многочисленные космические корабли «Красная планета» в 1960-х и начале 1970-х годов, но большинство из этих миссий потерпели неудачу. Марс 2 (1971) и «Марс-3» (1971 г.) работали успешно, но не смогли нанести на карту поверхность из-за пыльных бурь. Посадочный модуль НАСА «Викинг-1» приземлился на поверхности Марса в 1976 году, совершив первую успешную посадку на Красной планете. Его близнец, «Викинг-2», через шесть недель приземлился в другом районе Марса.

Посадочные аппараты «Викинг» сделали первые снимки марсианской поверхности крупным планом, но не нашли убедительных доказательств существования жизни. Тем не менее, снова возникли споры: Гил Левин, главный исследователь эксперимента по обнаружению жизни «Маркетинговый выпуск викингов», всегда утверждал, что посадочные модули шпионили доказательства микробного метаболизма в марсианской грязи. (Левин умер в июле 2021 года в возрасте 9 лет7.)

Следующими двумя кораблями, успешно достигшими Красной планеты, были посадочный модуль Mars Pathfinder и орбитальный аппарат Mars Global Surveyor. Оба корабля НАСА были запущены в 1996 году. Небольшой робот на борту Pathfinder по имени Sojourner — первый колесный вездеход. когда-либо исследовал поверхность другой планеты — рискнул исследовать поверхность планеты, анализируя горные породы в течение 95 земных дней.

В 2001 году НАСА запустило орбитальный аппарат Mars Odyssey , который обнаружил огромное количество водяного льда под марсианской поверхностью, в основном в верхних 3 футах (1 метр). Остается неясным, находится ли под ним больше воды, поскольку зонд не может видеть воду глубже.

Mars Odyssey запущен с мыса Канаверал, штат Флорида, 7 апреля 2001 г. (Изображение предоставлено НАСА)

В 2003 г. Марс прошел ближе к Земле, чем когда-либо за последние 60 000 лет. . В том же году НАСА запустило два марсохода размером с тележку для гольфа, получившие прозвища Spirit и Opportunity, которые исследовали различные области марсианской поверхности после приземления в январе 2004 года. Оба марсохода обнаружили много признаков того, что вода когда-то текла по поверхности планеты.

Spirit и Opportunity изначально выполняли трехмесячные миссии на поверхности, но оба продолжали скитаться гораздо дольше. НАСА не объявляло Spirit мертвым до 2011 года, а Opportunity все еще действовал, пока в середине 2018 года не разразилась пыльная буря.

В 2008 году НАСА отправило посадочный модуль под названием «Феникс» на далекие северные равнины Марса. Робот подтвердил наличие водяного льда в ближних недрах среди других находок.

В 2011 году марсианская научная лаборатория НАСА отправила марсоход Curiosity для изучения прошлого потенциала Марса для жизни. Нет. Спустя долгое время после приземления в кратере Гейла на Красной планете в августе 2012 года робот размером с автомобиль определил, что в древнем прошлом в этом районе находилась долгоживущая, потенциально обитаемая система озер и ручьев. Curiosity также обнаружил сложные органические молекулы и зафиксировал сезонные колебания концентрации метана в атмосфере.

У НАСА есть два других орбитальных аппарата, работающих вокруг планеты — Mars Reconnaissance Orbiter и MAVEN (Mars Atmosphere and Volatile Evolution), которые прибыли на Марс в 2006 и 2014 годах соответственно. У Европейского космического агентства (ЕКА) также есть два космических корабля на орбите планеты: Mars Express и Trace Gas Orbiter.

В сентябре 2014 года индийский марсианский орбитальный аппарат также достиг Красной планеты, став четвертой страной, успешно вышедшей на орбиту вокруг Марса.

В ноябре 2018 года НАСА высадило на поверхность стационарный аппарат под названием Mars InSight. Как отмечалось выше, InSight исследует внутреннюю структуру и состав Марса, в первую очередь, измеряя и описывая марсотрясения.

Марсоход NASA Perseverance сделал это селфи над скалой по прозвищу «Рошет» 10 сентября 2021 года. (Изображение предоставлено NASA/JPL-Caltech/MSSS) — охотящийся и собирающий образцы марсоход Perseverance в июле 2020 года. Perseverance, размером примерно с Curiosity, приземлился на дно марсианского кратера Jezero в феврале 2021 года вместе с крошечным вертолетом, демонстрирующим технологии, известным как Ingenuity.

По состоянию на сентябрь 2021 года Ingenuity совершила более дюжины полетов на Марс, доказав, что воздушное исследование планеты возможно. Perseverance задокументировала первые полеты вертолета весом 4 фунта (1,8 кг), а затем всерьез сосредоточилась на своей научной миссии. Большой марсоход уже собрал несколько образцов, часть большого тайника, который будет доставлен на Землю, возможно, уже в 2031 году в рамках совместной кампании НАСА и ЕКА.

В июле 2020 года также были запущены первая марсианская миссия Объединенных Арабских Эмиратов под названием «Надежда» и первая полностью отечественная марсианская программа Китая, Tianwen 1. Орбитальный аппарат «Надежда» прибыл на Марс в феврале 2021 года и изучает атмосферу планеты, погоду и климат.

Истории по теме:

Tianwen 1, состоящий из орбитального аппарата и дуэта спускаемого аппарата и вездехода, также вышел на орбиту Марса в феврале 2021 года. Приземлившийся элемент приземлился несколько месяцев спустя, в мае. Марсоход Tianwen 1, названный Zhurong, вскоре скатился по трапу посадочной платформы и начал исследовать марсианскую поверхность.

ЕКА также работает над созданием марсохода в рамках сотрудничества с Россией по программе «ЭкзоМарс». Этот робот по имени Розалинд Франклин должен был быть запущен в середине 2020 года, но проблемы с парашютом и другие проблемы отложили запуск до следующей возможности, в 2022 году (Марс и Земля должным образом выравниваются для межпланетных миссий только раз в 26 месяцев). Розалинда Среди прочих задач Франклин будет искать признаки прошлой жизни на Марсе. Робот будет использовать бур, чтобы углубиться в Красную планету, собирая образцы почвы примерно на 2 метра (6,5 футов) под землей.

Потерянные миссии на Марс

Марс далеко не та планета, до которой легко добраться. НАСА, Россия, Европейское космическое агентство, Китай, Япония и Советский Союз вместе потеряли много космических аппаратов в своем стремлении исследовать Красную планету. Известные примеры включают (но не ограничиваются):

1992 — NASA Mars Observer

1996 — Россия Mars 96

1998 — Mars Climate Orbiter NASA, Nozomi Япония

1999 — NASA’s Mars B003eagle Lander

3 2 посадочных модуля

2011 г. — российская миссия «Фобус-Грунт» на Фобос с китайским орбитальным аппаратом Yinghuo-1

2016 г. — испытательный посадочный модуль ЕКА «Скиапарелли»

Будущие пилотируемые миссии

Не только роботы получают билет на Марс. Группа ученых из правительственных учреждений, научных кругов и промышленности определила, что к 2030-м годам должна стать возможной пилотируемая миссия на Марс под руководством НАСА.

В конце 2017 года администрация президента Дональда Трампа поручила НАСА отправить людей обратно на Луну, прежде чем отправиться на Марс. НАСА работает над этой целью в рамках программы под названием Artemis, целью которой является обеспечение устойчивого и долгосрочного присутствия человека на Луне и вокруг нее к концу 2020-х годов. По словам представителей НАСА, уроки и навыки, извлеченные из этой лунной операции, помогут проложить путь к посадке на Марс.

За последние несколько десятилетий роботизированные миссии на Красную планету увенчались большим успехом, но доставить людей на Марс по-прежнему сложно. С современными ракетными технологиями людям потребуется не менее шести месяцев, чтобы отправиться на Марс. Таким образом, исследователи Красной планеты будут длительное время подвергаться воздействию радиации дальнего космоса и микрогравитации, которые оказывают разрушительное воздействие на организм человека. Выполнение действий в условиях умеренной гравитации на Марсе может оказаться чрезвычайно трудным после многих месяцев в условиях микрогравитации. Исследования эффектов микрогравитации продолжаются на Международной космической станции.

НАСА — не единственная организация, стремящаяся пилотировать Марс. Другие страны, в том числе Китай и Россия, также объявили о своих целях по отправке людей на Красную планету.

А Илон Маск, основатель и генеральный директор SpaceX, давно подчеркивал, что создал компанию еще в 2002 году в первую очередь для того, чтобы помочь человечеству заселить Красную планету. В настоящее время SpaceX разрабатывает и тестирует полностью многоразовую транспортную систему для дальнего космоса под названием Starship, которая, по мнению Маска, является прорывом, необходимым для того, чтобы наконец доставить людей на Марс.

Дополнительные ресурсы

Исследуйте Марс более подробно с помощью программы NASA Mars Exploration Program (откроется в новой вкладке). Узнайте больше о климате Марса с Национальной метеорологической службой (откроется в новой вкладке). Отправьте свое имя на Марс во время следующего полета НАСА на Красную планету.

Библиография

НАСА. Информационный бюллетень Марса. НАСА. Получено 11 июля 2022 г. с www.nssdc.gsfc.nasa.gov/planetary/factsheet/marsfact.html (откроется в новой вкладке)

НАСА. Марс. НАСА. Получено 11 июля 2022 г. с www.solarsystem.nasa.gov/planets/mars/overview/ (открывается в новой вкладке)

НАСА. Исследование Марса НАСА. НАСА. Получено 11 июля 2022 г. с https://mars.nasa.gov/ (открывается в новой вкладке)

НАСА. Отправьте свое имя на Марс. НАСА. Получено 11 июля 2022 г. с www.mars.nasa.gov/participate/send-your-name/future (открывается в новой вкладке)

Министерство торговли США, Северная Аравия. Планета Марс. Национальная служба погоды. Проверено 11 июля 2022 г. с www.weather.gov/fsd/mars (открывается в новой вкладке)

Присоединяйтесь к нашим космическим форумам, чтобы продолжать обсуждать последние миссии, ночное небо и многое другое! А если у вас есть новость, исправление или комментарий, сообщите нам об этом по адресу: [email protected].

Чарльз К. Чой — автор статей для Space.com и Live Science. Он охватывает все, что связано с человеческим происхождением и астрономией, а также физику, животных и общие научные темы. Чарльз имеет степень магистра гуманитарных наук Университета Миссури-Колумбия, Школу журналистики и степень бакалавра гуманитарных наук Университета Южной Флориды. Чарльз побывал на всех континентах Земли, пил прогорклый чай с маслом яка в Лхасе, плавал с морскими львами на Галапагосских островах и даже взбирался на айсберг в Антарктиде. Посетите его на http://www.sciwriter.us

Марс: все, что вам нужно знать о Красной планете

Космос поддерживается его аудиторией. Когда вы покупаете по ссылкам на нашем сайте, мы можем получать партнерскую комиссию. Вот почему вы можете доверять нам.

Марс — четвертая планета от Солнца, также известная как Красная планета. (Изображение предоставлено РистоАрнаудовым через Getty Images)

Марс, четвертая планета от Солнца, славится своим ржаво-красным цветом. Красная планета — это холодный пустынный мир с очень разреженной атмосферой. Но пыльная, безжизненная (насколько нам известно) планета далеко не унылая.

Феноменальные пыльные бури могут стать такими большими, что охватят всю планету, температура может стать настолько низкой, что углекислый газ в атмосфере конденсируется прямо в снег или иней, а марсотрясения — марсианская версия землетрясения — регулярно сотрясают обстановку.

Поэтому неудивительно, что этот маленький красный камень продолжает интриговать ученых и является одним из наиболее изученных тел в Солнечной системе , согласно NASA Science .

Связанный: Сколько времени нужно, чтобы добраться до Марса?

В соответствии с кровавым цветом Красной планеты римляне назвали ее в честь своего бога войны. По правде говоря, римляне копировали древних греков, которые также назвали планету в честь своего бога войны Ареса.

Другие цивилизации также обычно давали названия планетам в зависимости от их цвета — например, египтяне называли ее «Хер Дешер», что означает «красная», а древние китайские астрономы называли ее «огненной звездой».

Почему Марс называют Красной планетой?

Яркий цвет ржавчины, которым известен Марс, обусловлен богатыми железом минералами в его реголите — рыхлой пылью и камнем, покрывающими его поверхность. Почва Земли также представляет собой своего рода реголит, хотя и насыщенный органическими веществами. По данным НАСА, минералы железа окисляются или ржавеют, в результате чего почва становится красной.

Ландшафт Марса

Холодная разреженная атмосфера планеты означает, что жидкая вода, вероятно, не может существовать на поверхности Марса в течение сколько-нибудь заметного периода времени. Особенности, называемые повторяющимися линиями склона, могут иметь струи соленой воды, стекающей по поверхности, но это свидетельство оспаривается; некоторые ученые утверждают, что водород, обнаруженный с орбиты в этом регионе, может указывать на наличие соленых солей. Это означает, что хотя эта планета-пустыня составляет всего половину диаметра Земли, на ней столько же суши.

Красная планета является домом для самой высокой горы и самой глубокой и длинной долины в Солнечной системе. Гора Олимп имеет высоту примерно 17 миль (27 километров), что примерно в три раза выше горы Эверест, а система долин Долины Маринер, названная в честь зонда Mariner 9, обнаружившего ее в 1971 году, достигает глубины 6 миль (10 км). ) и тянется с востока на запад примерно на 2500 миль (4000 км), что составляет около одной пятой расстояния вокруг Марса и близко к ширине Австралии.

Ученые считают, что Морские Долины образовались в основном в результате растрескивания земной коры по мере ее растяжения. Отдельные каньоны в системе имеют ширину до 60 миль (100 км). Каньоны сливаются в центральной части долины Маринер в районе шириной до 370 миль (600 км). Большие каналы, выходящие из концов некоторых каньонов, и слоистые отложения внутри позволяют предположить, что каньоны когда-то могли быть заполнены жидкой водой.

Марс также имеет самые большие вулканы в Солнечной системе, Олимп Монс является одним из них. Массивный вулкан диаметром около 370 миль (600 км) достаточно широк, чтобы покрыть штат Нью-Мексико. Олимп Монс — это щитовой вулкан со склонами, которые постепенно поднимаются, как у гавайских вулканов, и был создан извержениями лавы, которая текла на большие расстояния, прежде чем затвердеть. На Марсе также есть много других форм вулканического рельефа, от небольших конусов с крутыми склонами до огромных равнин, покрытых застывшей лавой. Некоторые незначительные извержения все еще могут происходить на планете сегодня.

Связанный: Космические вулканы: Происхождение, разновидности и извержения

Гора Олимп — самый большой известный вулкан в Солнечной системе. Это цифровое мозаичное изображение вулкана было получено орбитальным аппаратом НАСА «Викинг-1». (Изображение предоставлено NASA/JPL-Caltech/USGS)

Каналы, долины и овраги встречаются по всему Марсу, и предполагается, что жидкая вода могла течь по поверхности планеты в последнее время. Некоторые каналы могут иметь ширину 60 миль (100 км) и длину 1200 миль (2000 км). Вода все еще может находиться в трещинах и порах подземных пород. Исследование, проведенное учеными в 2018 году, показало, что соленая вода под поверхностью Марса может содержать значительное количество кислорода, который может поддерживать микробную жизнь. Однако количество кислорода зависит от температуры и давления; Температура на Марсе время от времени меняется по мере смещения наклона его оси вращения.

Многие районы Марса представляют собой плоские низменные равнины. Самые низкие северные равнины — одни из самых плоских и гладких мест в Солнечной системе, потенциально созданные водой, которая когда-то текла по марсианской поверхности. Северное полушарие в основном расположено на более низкой высоте, чем южное полушарие, что позволяет предположить, что кора на севере может быть тоньше, чем на юге. Эта разница между севером и югом может быть связана с очень сильным ударом вскоре после рождения Марса.

Количество кратеров на Марсе значительно варьируется от места к месту, в зависимости от возраста поверхности. Большая часть поверхности южного полушария чрезвычайно старая, и на ней много кратеров, в том числе самая большая на планете равнина Эллада шириной 1400 миль (2300 км), в то время как поверхность северного полушария моложе и поэтому имеет меньше кратеров. Некоторые вулканы также имеют всего несколько кратеров, что говорит о том, что они извергались недавно, в результате чего образовавшаяся лава покрыла все старые кратеры. Вокруг некоторых кратеров имеются отложения обломков необычного вида, напоминающие затвердевшие селевые потоки, что может указывать на то, что ударный элемент столкнулся с подземными водами или льдом.

В 2018 году космический корабль Европейского космического агентства «Марс Экспресс» обнаружил то, что могло быть взвесь воды и зерна под ледяным плато Южным. (В некоторых отчетах оно описывается как «озеро», но неясно, сколько реголита находится в воде.) Говорят, что этот водоем имеет диаметр около 12,4 миль (20 км). Его подземное расположение напоминает аналогичные подземные озера в Антарктиде, в которых, как было обнаружено, обитают микробы. В конце года «Марс Экспресс» также обнаружил огромную ледяную зону в кратере Королева на Красной планете.

Полярные шапки Марса

Обширные отложения того, что кажется тонкослоистым нагромождением водяного льда и пыли, простираются от полюсов до 80 градусов широты в обоих марсианских полушариях. Вероятно, они были отложены атмосферой в течение длительного периода времени. Поверх большей части этих слоистых отложений в обоих полушариях лежат шапки из водяного льда, которые остаются замороженными круглый год.

В зимнее время появляются дополнительные сезонные морозные шапки. Они сделаны из твердого углекислого газа, также известного как «сухой лед», который сконденсировался из углекислого газа в атмосфере. (Думаю, воздух Марса составляет около 95% углекислого газа по объему.) В самую глубокую часть зимы этот мороз может распространяться от полюсов до широт до 45 градусов или на полпути к экватору. Согласно отчету, опубликованному в Journal of Geophysical Research-Planets, слой сухого льда имеет пушистую текстуру, похожую на свежевыпавший снег.

Климат Марса

Лед и пыль образуют марсианские полярные шапки. (Изображение предоставлено НАСА/Лаборатории реактивного движения/Университет штата Аризона, Р. Лук)

(открывается в новой вкладке)

Марс намного холоднее Земли, в значительной степени из-за его большего расстояния от Солнца. Средняя температура составляет около минус 80 градусов по Фаренгейту (минус 60 градусов по Цельсию), хотя может варьироваться от минус 195 F (минус 125 C) у полюсов зимой до 70 F (20 C) в полдень у экватора.

Атмосфера Марса, богатая углекислым газом, примерно в 100 раз менее плотная, чем в среднем Земля, но, тем не менее, она достаточно плотная, чтобы поддерживать погоду, облака и ветер. Плотность атмосферы меняется в зависимости от сезона, так как зимой углекислый газ вымерзает из марсианского воздуха. В древнем прошлом атмосфера, вероятно, была значительно толще и могла поддерживать воду, текущую по поверхности планеты. Со временем более легкие молекулы в марсианской атмосфере вырвались под давлением солнечного ветра, который повлиял на атмосферу, поскольку у Марса нет глобального магнитного поля. Сегодня этот процесс изучается миссией НАСА MAVEN (Mars Atmosphere and Volatile Evolution).

Марсианский разведывательный орбитальный аппарат НАСА впервые обнаружил снежные облака из двуокиси углерода, что сделало Марс единственным известным телом в Солнечной системе, где такая необычная зимняя погода. Красная планета также вызывает падение водяного льда из облаков.

Пыльные бури на Марсе — крупнейшие в Солнечной системе, способны покрыть всю Красную планету и длятся месяцами. Одна из теорий относительно того, почему пыльные бури на Марсе могут стать такими сильными, заключается в том, что переносимые по воздуху частицы пыли поглощают солнечный свет, нагревая марсианскую атмосферу в их окрестностях. Затем теплые карманы воздуха текут в более холодные регионы, создавая ветры. Сильные ветры поднимают с земли больше пыли, которая, в свою очередь, нагревает атмосферу, поднимая больше ветра и поднимая больше пыли.

Эти пыльные бури могут представлять серьезную опасность для роботов на поверхности Марса. Например, марсоход НАСА «Оппортьюнити» погиб после того, как в 2018 году его охватил гигантский шторм, который неделями блокировал доступ солнечного света к солнечным панелям робота.

Марсоход Curiosity НАСА сфотографировал эти дрейфующие облака 17 мая 2019 года, на 2410-й марсианский день, или сол, с помощью своих черно-белых навигационных камер. (Изображение предоставлено NASA/JPL-Caltech)

(открывается в новой вкладке)

Орбита Марса

Марс расположен дальше от Солнца, чем Земля, поэтому год на Красной планете длиннее — 687 дней по сравнению с 365 днями на нашей планете. Однако две планеты имеют одинаковую продолжительность дня; Марсу требуется около 24 часов 40 минут, чтобы совершить один оборот вокруг своей оси, по сравнению с 24 часами для Земли.

Ось Марса, как и земная, наклонена по отношению к Солнцу. Это означает, что, как и на Земле, количество солнечного света, падающего на определенные части Красной планеты, может сильно различаться в течение года, что дает Марсу сезоны.

Орбита Марса: Краткие факты

Среднее расстояние от Солнца : 141 633 260 миль (227 936 640 км). Для сравнения: в 1,524 раза больше, чем на Земле.

Перигелий (ближайшее сближение с Солнцем) : 128 400 000 миль (206 600 000 км). Для сравнения: в 1,404 раза больше, чем на Земле.

Афелий (самое дальнее расстояние от Солнца) : 154 900 000 миль (249 200 000 км). Для сравнения: в 1,638 раза больше, чем на Земле.

Однако времена года на Марсе более экстремальные, чем на Земле, потому что эллиптическая овальная орбита Красной планеты вокруг Солнца более вытянута, чем у любой другой большой планеты. Когда Марс находится ближе всего к Солнцу, его южное полушарие наклонено к нашей звезде, что дает планете короткое теплое лето, а северное полушарие переживает короткую холодную зиму. Когда Марс находится дальше всего от Солнца, северное полушарие наклонено к Солнцу, что обеспечивает долгое мягкое лето, а южное полушарие переживает долгую холодную зиму.

Наклон оси Красной планеты сильно колеблется с течением времени, потому что она не стабилизируется большой луной, такой как Земля. Эта ситуация привела к разному климату на поверхности Марса на протяжении всей его истории. Исследование 2017 года предполагает, что изменение наклона также повлияло на выброс метана в атмосферу Марса, вызывая временные периоды потепления, которые позволяли течь воде.

Размер, состав и структура

Марс имеет диаметр 4 220 миль (6 791 км) — намного меньше, чем Земля, которая составляет 7,926 миль (12 756 км) в ширину. Красная планета примерно на 10% массивнее нашего родного мира, а гравитационное притяжение на 38% сильнее. (Человек весом 100 фунтов здесь, на Земле, будет весить всего 62 фунта на Марсе, но его масса будет одинаковой на обеих планетах.)

Состав атмосферы (по объему)

95,32 % углекислого газа, 2,7 % азота, 1,6 % аргона, 0,13 % кислорода и 0,08 % монооксида углерода, с небольшими количествами воды, оксида азота, неона, водорода, дейтерия и кислорода, криптона и ксенона.

Магнитное поле

Марс потерял свое глобальное магнитное поле около 4 миллиардов лет назад, что привело к удалению большей части его атмосферы солнечным ветром. Но сегодня есть области коры планеты, которые могут быть по крайней мере в 10 раз более сильно намагничены, чем что-либо измеренное на Земле, что предполагает, что эти области являются остатками древнего глобального магнитного поля.

Химический состав

Марс, вероятно, имеет твердое ядро, состоящее из железа, никеля и серы. Мантия Марса, вероятно, похожа на земную тем, что состоит в основном из перидотита, состоящего в основном из кремния, кислорода, железа и магния. Кора, вероятно, в основном состоит из базальта вулканической породы, который также распространен в коре Земли и Луны, хотя некоторые породы коры, особенно в северном полушарии, могут быть формой андезита, вулканической породы, которая содержит больше кремний, чем базальт.

Внутренняя структура

Посадочный модуль НАСА InSight исследует внутреннюю часть Марса с момента приземления вблизи экватора планеты в ноябре 2018 года. InSight измеряет и характеризует марсотрясения, а члены миссии отслеживают колебания наклона Марса с течением времени, точно отслеживая положение спускаемого аппарата на поверхности планеты.

Эти данные открыли ключевую информацию о внутренней структуре Марса. Например, члены команды InSight недавно подсчитали, что ядро ​​планеты имеет ширину от 1110 до 1300 миль (от 1780 до 2080 км). Наблюдения InSight также показывают, что толщина коры Марса составляет в среднем от 14 до 45 миль (24 и 72 км), а мантия составляет остальную часть (неатмосферного) объема планеты.

Для сравнения: ядро ​​Земли имеет ширину около 4 400 миль (7 100 км) — больше, чем сам Марс, — а толщина мантии составляет примерно 1 800 миль (2 900 км). Земля имеет два вида коры, континентальную и океаническую, средняя толщина которых составляет около 25 миль (40 км) и 5 ​​миль (8 км) соответственно.

Спутники Марса

Два спутника Марса, Фобос и Деймос, были открыты американским астрономом Асафом Холлом в течение недели в 1877 году. Холл почти отказался от поисков спутника Марса, но его жена, Анджелина, убеждала его. Следующей ночью он обнаружил Деймос, а через шесть дней — Фобос. Он назвал луны в честь сыновей греческого бога войны Ареса — Фобос означает «страх», а Деймос — «разгром».

Изображение Фобоса, полученное марсианским разведывательным орбитальным аппаратом НАСА (Изображение предоставлено НАСА/Лаборатории реактивного движения/Университет Аризоны) покрытые пылью и рыхлыми камнями. Они крошечные по сравнению с земной Луной и имеют неправильную форму, поскольку им не хватает гравитации, чтобы принять более круглую форму. Самая широкая точка Фобоса составляет около 17 миль (27 км), а самая широкая часть Деймоса — примерно 9миль (15 км). (Луна Земли имеет ширину 2159 миль или 3475 км.)

Обе луны Марса испещрены кратерами от ударов метеоритов. Поверхность Фобоса также имеет замысловатый рисунок канавок, которые могут быть трещинами, образовавшимися после удара, образовавшего самый большой кратер Луны — дыру шириной около 6 миль (10 км), или почти половину ширины Фобоса. Два марсианских спутника всегда обращены к своей родительской планете одним и тем же лицом, точно так же, как наша Луна обращена к Земле.

Изображение Деймоса, полученное марсианским разведывательным орбитальным аппаратом НАСА (Изображение предоставлено NASA/JPL-Caltech/University of Arizona)

Остается неясным, как родились Фобос и Деймос. Это могут быть бывшие астероиды, которые были захвачены гравитационным притяжением Марса, или они могли образоваться на орбите вокруг Марса примерно в то же время, когда планета возникла. Ультрафиолетовый свет, отраженный от Фобоса, является убедительным доказательством того, что Луна является захваченным астероидом, по мнению астрономов из Университета Падуи в Италии.

Фобос постепенно движется по спирали к Марсу, с каждым столетием приближаясь к Красной планете примерно на 6 футов (1,8 метра). В течение 50 миллионов лет Фобос либо врежется в Марс, либо расколется и образует кольцо обломков вокруг планеты.

Исследования и исследования

Галилео Галилей, первый человек, наблюдавший Марс в телескоп в 1610 году. В следующем столетии астрономы обнаружили полярные ледяные шапки планеты. В 19-м и 20-м веках некоторые исследователи, наиболее известный из которых Персиваль Лоуэлл, считали, что видели на Марсе сеть длинных прямых каналов, которые намекали на возможную цивилизацию. Однако эти наблюдения оказались ошибочной интерпретацией геологических особенностей.

Несколько марсианских камней упали на Землю в течение тысячелетий, предоставив ученым редкую возможность изучить части Марса, не покидая нашу планету. Одной из самых спорных находок стал Allan Hills 84001 (ALH84001) — марсианский метеорит, который, по данным 1996, вероятно, содержит крошечные окаменелости и другие свидетельства жизни на Марсе. Другие исследователи подвергают сомнению эту гипотезу, но команда, проводившая знаменитое исследование 1996 года, твердо придерживается своей интерпретации, и дебаты по поводу ALH84001 продолжаются и сегодня.

В 2018 году отдельное исследование метеоритов показало, что органические молекулы — углеродсодержащие строительные блоки жизни, хотя и не обязательно свидетельство самой жизни — могли образоваться на Марсе в результате химических реакций, подобных батарейкам.

Роботизированный космический корабль начал наблюдения за Марсом в 1960-х годах, когда Соединенные Штаты запустили Маринер-4 в 1964 году и Маринеры 6 и 7 в 1969 году. Эти первые миссии показали, что Марс представляет собой бесплодный мир, без каких-либо признаков жизни или цивилизаций. Такие люди, как Лоуэлл представлял себе там. В 1971 году «Маринер-9» облетел Марс, нанеся на карту около 80 % планеты и обнаружив ее вулканы и большие каньоны.

Советский Союз также запускал многочисленные космические корабли «Красная планета» в 1960-х и начале 1970-х годов, но большинство из этих миссий потерпели неудачу. Марс 2 (1971) и «Марс-3» (1971 г.) работали успешно, но не смогли нанести на карту поверхность из-за пыльных бурь. Посадочный модуль НАСА «Викинг-1» приземлился на поверхности Марса в 1976 году, совершив первую успешную посадку на Красной планете. Его близнец, «Викинг-2», через шесть недель приземлился в другом районе Марса.

Посадочные аппараты «Викинг» сделали первые снимки марсианской поверхности крупным планом, но не нашли убедительных доказательств существования жизни. Тем не менее, снова возникли споры: Гил Левин, главный исследователь эксперимента по обнаружению жизни «Маркетинговый выпуск викингов», всегда утверждал, что посадочные модули шпионили доказательства микробного метаболизма в марсианской грязи. (Левин умер в июле 2021 года в возрасте 9 лет7.)

Следующими двумя кораблями, успешно достигшими Красной планеты, были посадочный модуль Mars Pathfinder и орбитальный аппарат Mars Global Surveyor. Оба корабля НАСА были запущены в 1996 году. Небольшой робот на борту Pathfinder по имени Sojourner — первый колесный вездеход. когда-либо исследовал поверхность другой планеты — рискнул исследовать поверхность планеты, анализируя горные породы в течение 95 земных дней.

В 2001 году НАСА запустило орбитальный аппарат Mars Odyssey , который обнаружил огромное количество водяного льда под марсианской поверхностью, в основном в верхних 3 футах (1 метр). Остается неясным, находится ли под ним больше воды, поскольку зонд не может видеть воду глубже.

Mars Odyssey запущен с мыса Канаверал, штат Флорида, 7 апреля 2001 г. (Изображение предоставлено НАСА)

В 2003 г. Марс прошел ближе к Земле, чем когда-либо за последние 60 000 лет. . В том же году НАСА запустило два марсохода размером с тележку для гольфа, получившие прозвища Spirit и Opportunity, которые исследовали различные области марсианской поверхности после приземления в январе 2004 года. Оба марсохода обнаружили много признаков того, что вода когда-то текла по поверхности планеты.

Spirit и Opportunity изначально выполняли трехмесячные миссии на поверхности, но оба продолжали скитаться гораздо дольше. НАСА не объявляло Spirit мертвым до 2011 года, а Opportunity все еще действовал, пока в середине 2018 года не разразилась пыльная буря.

В 2008 году НАСА отправило посадочный модуль под названием «Феникс» на далекие северные равнины Марса. Робот подтвердил наличие водяного льда в ближних недрах среди других находок.

В 2011 году марсианская научная лаборатория НАСА отправила марсоход Curiosity для изучения прошлого потенциала Марса для жизни. Нет. Спустя долгое время после приземления в кратере Гейла на Красной планете в августе 2012 года робот размером с автомобиль определил, что в древнем прошлом в этом районе находилась долгоживущая, потенциально обитаемая система озер и ручьев. Curiosity также обнаружил сложные органические молекулы и зафиксировал сезонные колебания концентрации метана в атмосфере.

У НАСА есть два других орбитальных аппарата, работающих вокруг планеты — Mars Reconnaissance Orbiter и MAVEN (Mars Atmosphere and Volatile Evolution), которые прибыли на Марс в 2006 и 2014 годах соответственно. У Европейского космического агентства (ЕКА) также есть два космических корабля на орбите планеты: Mars Express и Trace Gas Orbiter.

В сентябре 2014 года индийский марсианский орбитальный аппарат также достиг Красной планеты, став четвертой страной, успешно вышедшей на орбиту вокруг Марса.

В ноябре 2018 года НАСА высадило на поверхность стационарный аппарат под названием Mars InSight. Как отмечалось выше, InSight исследует внутреннюю структуру и состав Марса, в первую очередь, измеряя и описывая марсотрясения.

Марсоход NASA Perseverance сделал это селфи над скалой по прозвищу «Рошет» 10 сентября 2021 года. (Изображение предоставлено NASA/JPL-Caltech/MSSS) — охотящийся и собирающий образцы марсоход Perseverance в июле 2020 года. Perseverance, размером примерно с Curiosity, приземлился на дно марсианского кратера Jezero в феврале 2021 года вместе с крошечным вертолетом, демонстрирующим технологии, известным как Ingenuity.

По состоянию на сентябрь 2021 года Ingenuity совершила более дюжины полетов на Марс, доказав, что воздушное исследование планеты возможно. Perseverance задокументировала первые полеты вертолета весом 4 фунта (1,8 кг), а затем всерьез сосредоточилась на своей научной миссии. Большой марсоход уже собрал несколько образцов, часть большого тайника, который будет доставлен на Землю, возможно, уже в 2031 году в рамках совместной кампании НАСА и ЕКА.

В июле 2020 года также были запущены первая марсианская миссия Объединенных Арабских Эмиратов под названием «Надежда» и первая полностью отечественная марсианская программа Китая, Tianwen 1. Орбитальный аппарат «Надежда» прибыл на Марс в феврале 2021 года и изучает атмосферу планеты, погоду и климат.

Истории по теме:

Tianwen 1, состоящий из орбитального аппарата и дуэта спускаемого аппарата и вездехода, также вышел на орбиту Марса в феврале 2021 года. Приземлившийся элемент приземлился несколько месяцев спустя, в мае. Марсоход Tianwen 1, названный Zhurong, вскоре скатился по трапу посадочной платформы и начал исследовать марсианскую поверхность.

ЕКА также работает над созданием марсохода в рамках сотрудничества с Россией по программе «ЭкзоМарс». Этот робот по имени Розалинд Франклин должен был быть запущен в середине 2020 года, но проблемы с парашютом и другие проблемы отложили запуск до следующей возможности, в 2022 году (Марс и Земля должным образом выравниваются для межпланетных миссий только раз в 26 месяцев). Розалинда Среди прочих задач Франклин будет искать признаки прошлой жизни на Марсе. Робот будет использовать бур, чтобы углубиться в Красную планету, собирая образцы почвы примерно на 2 метра (6,5 футов) под землей.

Потерянные миссии на Марс

Марс далеко не та планета, до которой легко добраться. НАСА, Россия, Европейское космическое агентство, Китай, Япония и Советский Союз вместе потеряли много космических аппаратов в своем стремлении исследовать Красную планету. Известные примеры включают (но не ограничиваются):

1992 — NASA Mars Observer

1996 — Россия Mars 96

1998 — Mars Climate Orbiter NASA, Nozomi Япония

1999 — NASA’s Mars B003eagle Lander

3 2 посадочных модуля

2011 г. — российская миссия «Фобус-Грунт» на Фобос с китайским орбитальным аппаратом Yinghuo-1

2016 г. — испытательный посадочный модуль ЕКА «Скиапарелли»

Будущие пилотируемые миссии

Не только роботы получают билет на Марс. Группа ученых из правительственных учреждений, научных кругов и промышленности определила, что к 2030-м годам должна стать возможной пилотируемая миссия на Марс под руководством НАСА.

В конце 2017 года администрация президента Дональда Трампа поручила НАСА отправить людей обратно на Луну, прежде чем отправиться на Марс. НАСА работает над этой целью в рамках программы под названием Artemis, целью которой является обеспечение устойчивого и долгосрочного присутствия человека на Луне и вокруг нее к концу 2020-х годов. По словам представителей НАСА, уроки и навыки, извлеченные из этой лунной операции, помогут проложить путь к посадке на Марс.

За последние несколько десятилетий роботизированные миссии на Красную планету увенчались большим успехом, но доставить людей на Марс по-прежнему сложно. С современными ракетными технологиями людям потребуется не менее шести месяцев, чтобы отправиться на Марс. Таким образом, исследователи Красной планеты будут длительное время подвергаться воздействию радиации дальнего космоса и микрогравитации, которые оказывают разрушительное воздействие на организм человека. Выполнение действий в условиях умеренной гравитации на Марсе может оказаться чрезвычайно трудным после многих месяцев в условиях микрогравитации. Исследования эффектов микрогравитации продолжаются на Международной космической станции.

НАСА — не единственная организация, стремящаяся пилотировать Марс. Другие страны, в том числе Китай и Россия, также объявили о своих целях по отправке людей на Красную планету.

А Илон Маск, основатель и генеральный директор SpaceX, давно подчеркивал, что создал компанию еще в 2002 году в первую очередь для того, чтобы помочь человечеству заселить Красную планету. В настоящее время SpaceX разрабатывает и тестирует полностью многоразовую транспортную систему для дальнего космоса под названием Starship, которая, по мнению Маска, является прорывом, необходимым для того, чтобы наконец доставить людей на Марс.

Дополнительные ресурсы

Исследуйте Марс более подробно с помощью программы NASA Mars Exploration Program (откроется в новой вкладке). Узнайте больше о климате Марса с Национальной метеорологической службой (откроется в новой вкладке). Отправьте свое имя на Марс во время следующего полета НАСА на Красную планету.

Библиография

НАСА. Информационный бюллетень Марса. НАСА. Получено 11 июля 2022 г. с www.nssdc.gsfc.nasa.gov/planetary/factsheet/marsfact.html (откроется в новой вкладке)

НАСА. Марс. НАСА. Получено 11 июля 2022 г. с www.solarsystem.nasa.gov/planets/mars/overview/ (открывается в новой вкладке)

НАСА. Исследование Марса НАСА. НАСА. Получено 11 июля 2022 г. с https://mars.nasa.gov/ (открывается в новой вкладке)

НАСА. Отправьте свое имя на Марс. НАСА. Получено 11 июля 2022 г. с www.mars.nasa.gov/participate/send-your-name/future (открывается в новой вкладке)

Министерство торговли США, Северная Аравия. Планета Марс. Национальная служба погоды. Проверено 11 июля 2022 г. с www.weather.gov/fsd/mars (открывается в новой вкладке)

Присоединяйтесь к нашим космическим форумам, чтобы продолжать обсуждать последние миссии, ночное небо и многое другое! А если у вас есть новость, исправление или комментарий, сообщите нам об этом по адресу: [email protected].

Чарльз К. Чой — автор статей для Space.com и Live Science. Он охватывает все, что связано с человеческим происхождением и астрономией, а также физику, животных и общие научные темы. Чарльз имеет степень магистра гуманитарных наук Университета Миссури-Колумбия, Школу журналистики и степень бакалавра гуманитарных наук Университета Южной Флориды. Чарльз побывал на всех континентах Земли, пил прогорклый чай с маслом яка в Лхасе, плавал с морскими львами на Галапагосских островах и даже взбирался на айсберг в Антарктиде. Посетите его на http://www.sciwriter.us

Марс: все, что вам нужно знать о Красной планете

Космос поддерживается его аудиторией. Когда вы покупаете по ссылкам на нашем сайте, мы можем получать партнерскую комиссию. Вот почему вы можете доверять нам.

Марс — четвертая планета от Солнца, также известная как Красная планета. (Изображение предоставлено РистоАрнаудовым через Getty Images)

Марс, четвертая планета от Солнца, славится своим ржаво-красным цветом. Красная планета — это холодный пустынный мир с очень разреженной атмосферой. Но пыльная, безжизненная (насколько нам известно) планета далеко не унылая.

Феноменальные пыльные бури могут стать такими большими, что охватят всю планету, температура может стать настолько низкой, что углекислый газ в атмосфере конденсируется прямо в снег или иней, а марсотрясения — марсианская версия землетрясения — регулярно сотрясают обстановку.

Поэтому неудивительно, что этот маленький красный камень продолжает интриговать ученых и является одним из наиболее изученных тел в Солнечной системе , согласно NASA Science .

Связанный: Сколько времени нужно, чтобы добраться до Марса?

В соответствии с кровавым цветом Красной планеты римляне назвали ее в честь своего бога войны. По правде говоря, римляне копировали древних греков, которые также назвали планету в честь своего бога войны Ареса.

Другие цивилизации также обычно давали названия планетам в зависимости от их цвета — например, египтяне называли ее «Хер Дешер», что означает «красная», а древние китайские астрономы называли ее «огненной звездой».

Почему Марс называют Красной планетой?

Яркий цвет ржавчины, которым известен Марс, обусловлен богатыми железом минералами в его реголите — рыхлой пылью и камнем, покрывающими его поверхность. Почва Земли также представляет собой своего рода реголит, хотя и насыщенный органическими веществами. По данным НАСА, минералы железа окисляются или ржавеют, в результате чего почва становится красной.

Ландшафт Марса

Холодная разреженная атмосфера планеты означает, что жидкая вода, вероятно, не может существовать на поверхности Марса в течение сколько-нибудь заметного периода времени. Особенности, называемые повторяющимися линиями склона, могут иметь струи соленой воды, стекающей по поверхности, но это свидетельство оспаривается; некоторые ученые утверждают, что водород, обнаруженный с орбиты в этом регионе, может указывать на наличие соленых солей. Это означает, что хотя эта планета-пустыня составляет всего половину диаметра Земли, на ней столько же суши.

Красная планета является домом для самой высокой горы и самой глубокой и длинной долины в Солнечной системе. Гора Олимп имеет высоту примерно 17 миль (27 километров), что примерно в три раза выше горы Эверест, а система долин Долины Маринер, названная в честь зонда Mariner 9, обнаружившего ее в 1971 году, достигает глубины 6 миль (10 км). ) и тянется с востока на запад примерно на 2500 миль (4000 км), что составляет около одной пятой расстояния вокруг Марса и близко к ширине Австралии.

Ученые считают, что Морские Долины образовались в основном в результате растрескивания земной коры по мере ее растяжения. Отдельные каньоны в системе имеют ширину до 60 миль (100 км). Каньоны сливаются в центральной части долины Маринер в районе шириной до 370 миль (600 км). Большие каналы, выходящие из концов некоторых каньонов, и слоистые отложения внутри позволяют предположить, что каньоны когда-то могли быть заполнены жидкой водой.

Марс также имеет самые большие вулканы в Солнечной системе, Олимп Монс является одним из них. Массивный вулкан диаметром около 370 миль (600 км) достаточно широк, чтобы покрыть штат Нью-Мексико. Олимп Монс — это щитовой вулкан со склонами, которые постепенно поднимаются, как у гавайских вулканов, и был создан извержениями лавы, которая текла на большие расстояния, прежде чем затвердеть. На Марсе также есть много других форм вулканического рельефа, от небольших конусов с крутыми склонами до огромных равнин, покрытых застывшей лавой. Некоторые незначительные извержения все еще могут происходить на планете сегодня.

Связанный: Космические вулканы: Происхождение, разновидности и извержения

Гора Олимп — самый большой известный вулкан в Солнечной системе. Это цифровое мозаичное изображение вулкана было получено орбитальным аппаратом НАСА «Викинг-1». (Изображение предоставлено NASA/JPL-Caltech/USGS)

Каналы, долины и овраги встречаются по всему Марсу, и предполагается, что жидкая вода могла течь по поверхности планеты в последнее время. Некоторые каналы могут иметь ширину 60 миль (100 км) и длину 1200 миль (2000 км). Вода все еще может находиться в трещинах и порах подземных пород. Исследование, проведенное учеными в 2018 году, показало, что соленая вода под поверхностью Марса может содержать значительное количество кислорода, который может поддерживать микробную жизнь. Однако количество кислорода зависит от температуры и давления; Температура на Марсе время от времени меняется по мере смещения наклона его оси вращения.

Многие районы Марса представляют собой плоские низменные равнины. Самые низкие северные равнины — одни из самых плоских и гладких мест в Солнечной системе, потенциально созданные водой, которая когда-то текла по марсианской поверхности. Северное полушарие в основном расположено на более низкой высоте, чем южное полушарие, что позволяет предположить, что кора на севере может быть тоньше, чем на юге. Эта разница между севером и югом может быть связана с очень сильным ударом вскоре после рождения Марса.

Количество кратеров на Марсе значительно варьируется от места к месту, в зависимости от возраста поверхности. Большая часть поверхности южного полушария чрезвычайно старая, и на ней много кратеров, в том числе самая большая на планете равнина Эллада шириной 1400 миль (2300 км), в то время как поверхность северного полушария моложе и поэтому имеет меньше кратеров. Некоторые вулканы также имеют всего несколько кратеров, что говорит о том, что они извергались недавно, в результате чего образовавшаяся лава покрыла все старые кратеры. Вокруг некоторых кратеров имеются отложения обломков необычного вида, напоминающие затвердевшие селевые потоки, что может указывать на то, что ударный элемент столкнулся с подземными водами или льдом.

В 2018 году космический корабль Европейского космического агентства «Марс Экспресс» обнаружил то, что могло быть взвесь воды и зерна под ледяным плато Южным. (В некоторых отчетах оно описывается как «озеро», но неясно, сколько реголита находится в воде.) Говорят, что этот водоем имеет диаметр около 12,4 миль (20 км). Его подземное расположение напоминает аналогичные подземные озера в Антарктиде, в которых, как было обнаружено, обитают микробы. В конце года «Марс Экспресс» также обнаружил огромную ледяную зону в кратере Королева на Красной планете.

Полярные шапки Марса

Обширные отложения того, что кажется тонкослоистым нагромождением водяного льда и пыли, простираются от полюсов до 80 градусов широты в обоих марсианских полушариях. Вероятно, они были отложены атмосферой в течение длительного периода времени. Поверх большей части этих слоистых отложений в обоих полушариях лежат шапки из водяного льда, которые остаются замороженными круглый год.

В зимнее время появляются дополнительные сезонные морозные шапки. Они сделаны из твердого углекислого газа, также известного как «сухой лед», который сконденсировался из углекислого газа в атмосфере. (Думаю, воздух Марса составляет около 95% углекислого газа по объему.) В самую глубокую часть зимы этот мороз может распространяться от полюсов до широт до 45 градусов или на полпути к экватору. Согласно отчету, опубликованному в Journal of Geophysical Research-Planets, слой сухого льда имеет пушистую текстуру, похожую на свежевыпавший снег.

Климат Марса

Лед и пыль образуют марсианские полярные шапки. (Изображение предоставлено НАСА/Лаборатории реактивного движения/Университет штата Аризона, Р. Лук)

(открывается в новой вкладке)

Марс намного холоднее Земли, в значительной степени из-за его большего расстояния от Солнца. Средняя температура составляет около минус 80 градусов по Фаренгейту (минус 60 градусов по Цельсию), хотя может варьироваться от минус 195 F (минус 125 C) у полюсов зимой до 70 F (20 C) в полдень у экватора.

Атмосфера Марса, богатая углекислым газом, примерно в 100 раз менее плотная, чем в среднем Земля, но, тем не менее, она достаточно плотная, чтобы поддерживать погоду, облака и ветер. Плотность атмосферы меняется в зависимости от сезона, так как зимой углекислый газ вымерзает из марсианского воздуха. В древнем прошлом атмосфера, вероятно, была значительно толще и могла поддерживать воду, текущую по поверхности планеты. Со временем более легкие молекулы в марсианской атмосфере вырвались под давлением солнечного ветра, который повлиял на атмосферу, поскольку у Марса нет глобального магнитного поля. Сегодня этот процесс изучается миссией НАСА MAVEN (Mars Atmosphere and Volatile Evolution).

Марсианский разведывательный орбитальный аппарат НАСА впервые обнаружил снежные облака из двуокиси углерода, что сделало Марс единственным известным телом в Солнечной системе, где такая необычная зимняя погода. Красная планета также вызывает падение водяного льда из облаков.

Пыльные бури на Марсе — крупнейшие в Солнечной системе, способны покрыть всю Красную планету и длятся месяцами. Одна из теорий относительно того, почему пыльные бури на Марсе могут стать такими сильными, заключается в том, что переносимые по воздуху частицы пыли поглощают солнечный свет, нагревая марсианскую атмосферу в их окрестностях. Затем теплые карманы воздуха текут в более холодные регионы, создавая ветры. Сильные ветры поднимают с земли больше пыли, которая, в свою очередь, нагревает атмосферу, поднимая больше ветра и поднимая больше пыли.

Эти пыльные бури могут представлять серьезную опасность для роботов на поверхности Марса. Например, марсоход НАСА «Оппортьюнити» погиб после того, как в 2018 году его охватил гигантский шторм, который неделями блокировал доступ солнечного света к солнечным панелям робота.

Марсоход Curiosity НАСА сфотографировал эти дрейфующие облака 17 мая 2019 года, на 2410-й марсианский день, или сол, с помощью своих черно-белых навигационных камер. (Изображение предоставлено NASA/JPL-Caltech)

(открывается в новой вкладке)

Орбита Марса

Марс расположен дальше от Солнца, чем Земля, поэтому год на Красной планете длиннее — 687 дней по сравнению с 365 днями на нашей планете. Однако две планеты имеют одинаковую продолжительность дня; Марсу требуется около 24 часов 40 минут, чтобы совершить один оборот вокруг своей оси, по сравнению с 24 часами для Земли.

Ось Марса, как и земная, наклонена по отношению к Солнцу. Это означает, что, как и на Земле, количество солнечного света, падающего на определенные части Красной планеты, может сильно различаться в течение года, что дает Марсу сезоны.

Орбита Марса: Краткие факты

Среднее расстояние от Солнца : 141 633 260 миль (227 936 640 км). Для сравнения: в 1,524 раза больше, чем на Земле.

Перигелий (ближайшее сближение с Солнцем) : 128 400 000 миль (206 600 000 км). Для сравнения: в 1,404 раза больше, чем на Земле.

Афелий (самое дальнее расстояние от Солнца) : 154 900 000 миль (249 200 000 км). Для сравнения: в 1,638 раза больше, чем на Земле.

Однако времена года на Марсе более экстремальные, чем на Земле, потому что эллиптическая овальная орбита Красной планеты вокруг Солнца более вытянута, чем у любой другой большой планеты. Когда Марс находится ближе всего к Солнцу, его южное полушарие наклонено к нашей звезде, что дает планете короткое теплое лето, а северное полушарие переживает короткую холодную зиму. Когда Марс находится дальше всего от Солнца, северное полушарие наклонено к Солнцу, что обеспечивает долгое мягкое лето, а южное полушарие переживает долгую холодную зиму.

Наклон оси Красной планеты сильно колеблется с течением времени, потому что она не стабилизируется большой луной, такой как Земля. Эта ситуация привела к разному климату на поверхности Марса на протяжении всей его истории. Исследование 2017 года предполагает, что изменение наклона также повлияло на выброс метана в атмосферу Марса, вызывая временные периоды потепления, которые позволяли течь воде.

Размер, состав и структура

Марс имеет диаметр 4 220 миль (6 791 км) — намного меньше, чем Земля, которая составляет 7,926 миль (12 756 км) в ширину. Красная планета примерно на 10% массивнее нашего родного мира, а гравитационное притяжение на 38% сильнее. (Человек весом 100 фунтов здесь, на Земле, будет весить всего 62 фунта на Марсе, но его масса будет одинаковой на обеих планетах.)

Состав атмосферы (по объему)

95,32 % углекислого газа, 2,7 % азота, 1,6 % аргона, 0,13 % кислорода и 0,08 % монооксида углерода, с небольшими количествами воды, оксида азота, неона, водорода, дейтерия и кислорода, криптона и ксенона.

Магнитное поле

Марс потерял свое глобальное магнитное поле около 4 миллиардов лет назад, что привело к удалению большей части его атмосферы солнечным ветром. Но сегодня есть области коры планеты, которые могут быть по крайней мере в 10 раз более сильно намагничены, чем что-либо измеренное на Земле, что предполагает, что эти области являются остатками древнего глобального магнитного поля.

Химический состав

Марс, вероятно, имеет твердое ядро, состоящее из железа, никеля и серы. Мантия Марса, вероятно, похожа на земную тем, что состоит в основном из перидотита, состоящего в основном из кремния, кислорода, железа и магния. Кора, вероятно, в основном состоит из базальта вулканической породы, который также распространен в коре Земли и Луны, хотя некоторые породы коры, особенно в северном полушарии, могут быть формой андезита, вулканической породы, которая содержит больше кремний, чем базальт.

Внутренняя структура

Посадочный модуль НАСА InSight исследует внутреннюю часть Марса с момента приземления вблизи экватора планеты в ноябре 2018 года. InSight измеряет и характеризует марсотрясения, а члены миссии отслеживают колебания наклона Марса с течением времени, точно отслеживая положение спускаемого аппарата на поверхности планеты.

Эти данные открыли ключевую информацию о внутренней структуре Марса. Например, члены команды InSight недавно подсчитали, что ядро ​​планеты имеет ширину от 1110 до 1300 миль (от 1780 до 2080 км). Наблюдения InSight также показывают, что толщина коры Марса составляет в среднем от 14 до 45 миль (24 и 72 км), а мантия составляет остальную часть (неатмосферного) объема планеты.

Для сравнения: ядро ​​Земли имеет ширину около 4 400 миль (7 100 км) — больше, чем сам Марс, — а толщина мантии составляет примерно 1 800 миль (2 900 км). Земля имеет два вида коры, континентальную и океаническую, средняя толщина которых составляет около 25 миль (40 км) и 5 ​​миль (8 км) соответственно.

Спутники Марса

Два спутника Марса, Фобос и Деймос, были открыты американским астрономом Асафом Холлом в течение недели в 1877 году. Холл почти отказался от поисков спутника Марса, но его жена, Анджелина, убеждала его. Следующей ночью он обнаружил Деймос, а через шесть дней — Фобос. Он назвал луны в честь сыновей греческого бога войны Ареса — Фобос означает «страх», а Деймос — «разгром».

Изображение Фобоса, полученное марсианским разведывательным орбитальным аппаратом НАСА (Изображение предоставлено НАСА/Лаборатории реактивного движения/Университет Аризоны) покрытые пылью и рыхлыми камнями. Они крошечные по сравнению с земной Луной и имеют неправильную форму, поскольку им не хватает гравитации, чтобы принять более круглую форму. Самая широкая точка Фобоса составляет около 17 миль (27 км), а самая широкая часть Деймоса — примерно 9миль (15 км). (Луна Земли имеет ширину 2159 миль или 3475 км.)

Обе луны Марса испещрены кратерами от ударов метеоритов. Поверхность Фобоса также имеет замысловатый рисунок канавок, которые могут быть трещинами, образовавшимися после удара, образовавшего самый большой кратер Луны — дыру шириной около 6 миль (10 км), или почти половину ширины Фобоса. Два марсианских спутника всегда обращены к своей родительской планете одним и тем же лицом, точно так же, как наша Луна обращена к Земле.

Изображение Деймоса, полученное марсианским разведывательным орбитальным аппаратом НАСА (Изображение предоставлено NASA/JPL-Caltech/University of Arizona)

Остается неясным, как родились Фобос и Деймос. Это могут быть бывшие астероиды, которые были захвачены гравитационным притяжением Марса, или они могли образоваться на орбите вокруг Марса примерно в то же время, когда планета возникла. Ультрафиолетовый свет, отраженный от Фобоса, является убедительным доказательством того, что Луна является захваченным астероидом, по мнению астрономов из Университета Падуи в Италии.

Фобос постепенно движется по спирали к Марсу, с каждым столетием приближаясь к Красной планете примерно на 6 футов (1,8 метра). В течение 50 миллионов лет Фобос либо врежется в Марс, либо расколется и образует кольцо обломков вокруг планеты.

Исследования и исследования

Галилео Галилей, первый человек, наблюдавший Марс в телескоп в 1610 году. В следующем столетии астрономы обнаружили полярные ледяные шапки планеты. В 19-м и 20-м веках некоторые исследователи, наиболее известный из которых Персиваль Лоуэлл, считали, что видели на Марсе сеть длинных прямых каналов, которые намекали на возможную цивилизацию. Однако эти наблюдения оказались ошибочной интерпретацией геологических особенностей.

Несколько марсианских камней упали на Землю в течение тысячелетий, предоставив ученым редкую возможность изучить части Марса, не покидая нашу планету. Одной из самых спорных находок стал Allan Hills 84001 (ALH84001) — марсианский метеорит, который, по данным 1996, вероятно, содержит крошечные окаменелости и другие свидетельства жизни на Марсе. Другие исследователи подвергают сомнению эту гипотезу, но команда, проводившая знаменитое исследование 1996 года, твердо придерживается своей интерпретации, и дебаты по поводу ALH84001 продолжаются и сегодня.

В 2018 году отдельное исследование метеоритов показало, что органические молекулы — углеродсодержащие строительные блоки жизни, хотя и не обязательно свидетельство самой жизни — могли образоваться на Марсе в результате химических реакций, подобных батарейкам.

Роботизированный космический корабль начал наблюдения за Марсом в 1960-х годах, когда Соединенные Штаты запустили Маринер-4 в 1964 году и Маринеры 6 и 7 в 1969 году. Эти первые миссии показали, что Марс представляет собой бесплодный мир, без каких-либо признаков жизни или цивилизаций. Такие люди, как Лоуэлл представлял себе там. В 1971 году «Маринер-9» облетел Марс, нанеся на карту около 80 % планеты и обнаружив ее вулканы и большие каньоны.

Советский Союз также запускал многочисленные космические корабли «Красная планета» в 1960-х и начале 1970-х годов, но большинство из этих миссий потерпели неудачу. Марс 2 (1971) и «Марс-3» (1971 г.) работали успешно, но не смогли нанести на карту поверхность из-за пыльных бурь. Посадочный модуль НАСА «Викинг-1» приземлился на поверхности Марса в 1976 году, совершив первую успешную посадку на Красной планете. Его близнец, «Викинг-2», через шесть недель приземлился в другом районе Марса.

Посадочные аппараты «Викинг» сделали первые снимки марсианской поверхности крупным планом, но не нашли убедительных доказательств существования жизни. Тем не менее, снова возникли споры: Гил Левин, главный исследователь эксперимента по обнаружению жизни «Маркетинговый выпуск викингов», всегда утверждал, что посадочные модули шпионили доказательства микробного метаболизма в марсианской грязи. (Левин умер в июле 2021 года в возрасте 9 лет7.)

Следующими двумя кораблями, успешно достигшими Красной планеты, были посадочный модуль Mars Pathfinder и орбитальный аппарат Mars Global Surveyor. Оба корабля НАСА были запущены в 1996 году. Небольшой робот на борту Pathfinder по имени Sojourner — первый колесный вездеход. когда-либо исследовал поверхность другой планеты — рискнул исследовать поверхность планеты, анализируя горные породы в течение 95 земных дней.

В 2001 году НАСА запустило орбитальный аппарат Mars Odyssey , который обнаружил огромное количество водяного льда под марсианской поверхностью, в основном в верхних 3 футах (1 метр). Остается неясным, находится ли под ним больше воды, поскольку зонд не может видеть воду глубже.

Mars Odyssey запущен с мыса Канаверал, штат Флорида, 7 апреля 2001 г. (Изображение предоставлено НАСА)

В 2003 г. Марс прошел ближе к Земле, чем когда-либо за последние 60 000 лет. . В том же году НАСА запустило два марсохода размером с тележку для гольфа, получившие прозвища Spirit и Opportunity, которые исследовали различные области марсианской поверхности после приземления в январе 2004 года. Оба марсохода обнаружили много признаков того, что вода когда-то текла по поверхности планеты.

Spirit и Opportunity изначально выполняли трехмесячные миссии на поверхности, но оба продолжали скитаться гораздо дольше. НАСА не объявляло Spirit мертвым до 2011 года, а Opportunity все еще действовал, пока в середине 2018 года не разразилась пыльная буря.

В 2008 году НАСА отправило посадочный модуль под названием «Феникс» на далекие северные равнины Марса. Робот подтвердил наличие водяного льда в ближних недрах среди других находок.

В 2011 году марсианская научная лаборатория НАСА отправила марсоход Curiosity для изучения прошлого потенциала Марса для жизни. Нет. Спустя долгое время после приземления в кратере Гейла на Красной планете в августе 2012 года робот размером с автомобиль определил, что в древнем прошлом в этом районе находилась долгоживущая, потенциально обитаемая система озер и ручьев. Curiosity также обнаружил сложные органические молекулы и зафиксировал сезонные колебания концентрации метана в атмосфере.

У НАСА есть два других орбитальных аппарата, работающих вокруг планеты — Mars Reconnaissance Orbiter и MAVEN (Mars Atmosphere and Volatile Evolution), которые прибыли на Марс в 2006 и 2014 годах соответственно. У Европейского космического агентства (ЕКА) также есть два космических корабля на орбите планеты: Mars Express и Trace Gas Orbiter.

В сентябре 2014 года индийский марсианский орбитальный аппарат также достиг Красной планеты, став четвертой страной, успешно вышедшей на орбиту вокруг Марса.

В ноябре 2018 года НАСА высадило на поверхность стационарный аппарат под названием Mars InSight. Как отмечалось выше, InSight исследует внутреннюю структуру и состав Марса, в первую очередь, измеряя и описывая марсотрясения.

Марсоход NASA Perseverance сделал это селфи над скалой по прозвищу «Рошет» 10 сентября 2021 года. (Изображение предоставлено NASA/JPL-Caltech/MSSS) — охотящийся и собирающий образцы марсоход Perseverance в июле 2020 года. Perseverance, размером примерно с Curiosity, приземлился на дно марсианского кратера Jezero в феврале 2021 года вместе с крошечным вертолетом, демонстрирующим технологии, известным как Ingenuity.

По состоянию на сентябрь 2021 года Ingenuity совершила более дюжины полетов на Марс, доказав, что воздушное исследование планеты возможно. Perseverance задокументировала первые полеты вертолета весом 4 фунта (1,8 кг), а затем всерьез сосредоточилась на своей научной миссии. Большой марсоход уже собрал несколько образцов, часть большого тайника, который будет доставлен на Землю, возможно, уже в 2031 году в рамках совместной кампании НАСА и ЕКА.

В июле 2020 года также были запущены первая марсианская миссия Объединенных Арабских Эмиратов под названием «Надежда» и первая полностью отечественная марсианская программа Китая, Tianwen 1. Орбитальный аппарат «Надежда» прибыл на Марс в феврале 2021 года и изучает атмосферу планеты, погоду и климат.

Истории по теме:

Tianwen 1, состоящий из орбитального аппарата и дуэта спускаемого аппарата и вездехода, также вышел на орбиту Марса в феврале 2021 года. Приземлившийся элемент приземлился несколько месяцев спустя, в мае. Марсоход Tianwen 1, названный Zhurong, вскоре скатился по трапу посадочной платформы и начал исследовать марсианскую поверхность.

ЕКА также работает над созданием марсохода в рамках сотрудничества с Россией по программе «ЭкзоМарс». Этот робот по имени Розалинд Франклин должен был быть запущен в середине 2020 года, но проблемы с парашютом и другие проблемы отложили запуск до следующей возможности, в 2022 году (Марс и Земля должным образом выравниваются для межпланетных миссий только раз в 26 месяцев). Розалинда Среди прочих задач Франклин будет искать признаки прошлой жизни на Марсе. Робот будет использовать бур, чтобы углубиться в Красную планету, собирая образцы почвы примерно на 2 метра (6,5 футов) под землей.

Потерянные миссии на Марс

Марс далеко не та планета, до которой легко добраться. НАСА, Россия, Европейское космическое агентство, Китай, Япония и Советский Союз вместе потеряли много космических аппаратов в своем стремлении исследовать Красную планету. Известные примеры включают (но не ограничиваются):

1992 — NASA Mars Observer

1996 — Россия Mars 96

1998 — Mars Climate Orbiter NASA, Nozomi Япония

1999 — NASA’s Mars B003eagle Lander

3 2 посадочных модуля

2011 г. — российская миссия «Фобус-Грунт» на Фобос с китайским орбитальным аппаратом Yinghuo-1

2016 г. — испытательный посадочный модуль ЕКА «Скиапарелли»

Будущие пилотируемые миссии

Не только роботы получают билет на Марс. Группа ученых из правительственных учреждений, научных кругов и промышленности определила, что к 2030-м годам должна стать возможной пилотируемая миссия на Марс под руководством НАСА.

В конце 2017 года администрация президента Дональда Трампа поручила НАСА отправить людей обратно на Луну, прежде чем отправиться на Марс. НАСА работает над этой целью в рамках программы под названием Artemis, целью которой является обеспечение устойчивого и долгосрочного присутствия человека на Луне и вокруг нее к концу 2020-х годов. По словам представителей НАСА, уроки и навыки, извлеченные из этой лунной операции, помогут проложить путь к посадке на Марс.

За последние несколько десятилетий роботизированные миссии на Красную планету увенчались большим успехом, но доставить людей на Марс по-прежнему сложно. С современными ракетными технологиями людям потребуется не менее шести месяцев, чтобы отправиться на Марс. Таким образом, исследователи Красной планеты будут длительное время подвергаться воздействию радиации дальнего космоса и микрогравитации, которые оказывают разрушительное воздействие на организм человека. Выполнение действий в условиях умеренной гравитации на Марсе может оказаться чрезвычайно трудным после многих месяцев в условиях микрогравитации. Исследования эффектов микрогравитации продолжаются на Международной космической станции.

НАСА — не единственная организация, стремящаяся пилотировать Марс. Другие страны, в том числе Китай и Россия, также объявили о своих целях по отправке людей на Красную планету.

А Илон Маск, основатель и генеральный директор SpaceX, давно подчеркивал, что создал компанию еще в 2002 году в первую очередь для того, чтобы помочь человечеству заселить Красную планету. В настоящее время SpaceX разрабатывает и тестирует полностью многоразовую транспортную систему для дальнего космоса под названием Starship, которая, по мнению Маска, является прорывом, необходимым для того, чтобы наконец доставить людей на Марс.

Дополнительные ресурсы

Исследуйте Марс более подробно с помощью программы NASA Mars Exploration Program (откроется в новой вкладке). Узнайте больше о климате Марса с Национальной метеорологической службой (откроется в новой вкладке). Отправьте свое имя на Марс во время следующего полета НАСА на Красную планету.

Библиография

НАСА. Информационный бюллетень Марса. НАСА. Получено 11 июля 2022 г. с www.nssdc.gsfc.nasa.gov/planetary/factsheet/marsfact.html (откроется в новой вкладке)

НАСА. Марс. НАСА. Получено 11 июля 2022 г. с www.solarsystem.nasa.gov/planets/mars/overview/ (открывается в новой вкладке)

НАСА. Исследование Марса НАСА. НАСА. Получено 11 июля 2022 г. с https://mars.nasa.gov/ (открывается в новой вкладке)

НАСА. Отправьте свое имя на Марс. НАСА. Получено 11 июля 2022 г. с www.mars.nasa.gov/participate/send-your-name/future (открывается в новой вкладке)

Министерство торговли США, Северная Аравия. Планета Марс. Национальная служба погоды. Проверено 11 июля 2022 г. с www.weather.gov/fsd/mars (открывается в новой вкладке)

Присоединяйтесь к нашим космическим форумам, чтобы продолжать обсуждать последние миссии, ночное небо и многое другое! А если у вас есть новость, исправление или комментарий, сообщите нам об этом по адресу: [email protected].

Чарльз К. Чой — автор статей для Space.com и Live Science. Он охватывает все, что связано с человеческим происхождением и астрономией, а также физику, животных и общие научные темы.